SAMPLE TEST, FORM A PART 1 — ENGLISH LANGUAGE ARTS Suggested Time — 90 Minutes 57 QUESTIONS REVISING/EDITING QUESTIONS 1–20 IMPORTANT NOTE The Revising/Editing section (Questions 1-20) is in two parts: Part A and Part B. REVISING/EDITING Part A DIRECTIONS: Read and answer each of the following questions. You will be asked to recognize and correct errors in sentences or short paragraphs. Mark the best answer for each question. 1. Read this sentence. During a nightly-news segment about a cooking contest, a reporter talked to some people who did the best in the contest. Which of these is the most precise revision for the words talked to some people who did the best in the contest? A. conversed with some of the people who won the contest B. spoke to the three contestants who did well C. discussed the contest with some of the winners D. interviewed the top three contestants Form A 40 CONTINUE ON TO THE NEXT PAGE 2. Read this paragraph. (1) When coal was used to heat homes, it frequently left a soot stain on the walls. (2) Brothers Cleo and Noah McVicker, who owned a cleaning product company, created a doughy substance to help people remove this soot. (3) Over time, as natural gas becomes more common, people had little need for soot cleansers, and the McVickers’ family company struggled to stay in business. (4) Then one day, Joe McVicker, Cleo’s son, learned that his sister-in-law had been using the substance for art projects in her classroom, so he remarketed the product as the toy known today as Play-Doh. Which sentence should be revised to correct an inappropriate shift in verb tense? E. sentence 1 F. sentence 2 G. sentence 3 H. sentence 4 3. Read this sentence. The animal shelter, which is located on Midway Street, is looking for volunteers to help with a variety of tasks, such as walking dogs cleaning kennels, hand-feeding newborn kittens, and supporting the pet adoption process. Which edit should be made in this sentence? A. delete the comma after shelter B. insert a comma after volunteers C. insert a comma after dogs D. delete the comma after kennels Form A 41 CONTINUE ON TO THE NEXT PAGE 4. Read these sentences. (1) Flyby missions near Jupiter have been happening since 1973. (2) Flyby missions allow scientists to collect data about Jupiter and its moons. What is the best way to combine these sentences to clarify the relationship between the ideas? E. Flyby missions near Jupiter, which allow scientists to collect data about the planet and its moons, have been happening since 1973. F. Although there have been flyby missions near Jupiter since 1973, they have allowed scientists to collect data about the planet and its moons. G. Flyby missions near Jupiter allow scientists to collect data about the planet and its moons, which have been happening since 1973. H. Since 1973 flyby missions have been happening near Jupiter, but they allow scientists to collect data about the planet and its moons. 5. Read this paragraph. (1) Eliza and Brianna have been singing in their school chorus since they were in fourth grade. (2) The girls always sing a duet at the school talent show, and they take turns singing the national anthem before school sporting events. (3) Outside of school, she also sings in a choir made up of young and old members of her community. (4) Both girls hope that they will be able to continue singing for many more years. Which sentence should be revised to correct a vague pronoun? A. sentence 1 B. sentence 2 C. sentence 3 D. sentence 4 6. Read this sentence. In 1962 the agile athletic Wilt Chamberlain became the first and only professional basketball player in the United States to score 100 points in a single game. Which edit should be made to correct this sentence? E. insert a comma after agile F. insert a comma after Chamberlain G. insert a comma after only H. insert a comma after States Form A 42 CONTINUE ON TO THE NEXT PAGE REVISING/EDITING Part B DIRECTIONS: Read the passage below and answer the questions following it. You will be asked to improve the writing quality of the passage and to correct errors so that the passage follows the conventions of standard written English. You may reread the passage if you need to. Mark the best answer for each question. Unlock, Ride, Return (1) In metropolitan areas around the world, millions of cars, trucks, and taxis pack the streets every day, causing headaches for commuters and polluting the environment. (2) Public transportation eases some of this congestion, but crowding and potential delays are still an issue for many travelers. (3) In recent years, another transportation option has been gaining momentum in some cities. (4) This option is all about sharing bicycles, which is a creative and new idea for some. (5) The bike share concept is fairly simple. (6) Bike stations are set up at multiple locations in a city. (7) Frequent users can purchase a membership pass, while less-frequent users or tourists can buy a daily permit. (8) Many city bikers prefer bike sharing over ownership. (9) They are not responsible for the bike’s storage or its maintenance. (10) Tourists also benefit from having an affordable way to experience the sites of a city. (11) The largest bike sharing program in the United States today is in New York City. (12) Known as Citi Bike, the program was launched in 2013 and now boasts 10,000 bikes spread across 600 stations in Manhattan, Brooklyn, and Queens. (13) Similarly, in Hangzhou, China, a city of 7 million residents, there are approximately 75,000 bikes offered across 2,700 stations. (14) In 2016, people used Citi Bike for a lot of trips, which turned out to be a huge increase in the number of people taking rides from the previous year. (15) These numbers are likely to increase further, since the program is set to expand into additional neighborhoods soon. (16) According to Citi Bike, the bike sharing concept has gained rapid popularity because it is “faster than walking, cheaper than a taxi, and more fun than the subway.” (17) New York City officials estimated that in 2016 the bike share program had kept nearly 5,000 tons of carbon dioxide out of the city’s air. (18) Officials in Montreal, Canada, and Lyon, France, have noted similar impacts on air quality. (19) As populations grow, fuel costs increase, and environmental concerns escalate, people will continue to search for more economical and environmentally friendly ways to travel. (20) Bike sharing is a new mode of transportation. 7. Which sentence should replace sentence 4 to more clearly introduce the topic of this passage? A. Bike share programs have become an accessible and environmentally friendly solution for many urban travelers. B. Bike share programs are successful because both residents and tourists can use the bicycles to travel within a city. C. Bike share programs offer an alternative form of transportation in urban areas that is faster than using a car or taxi. D. Bike share programs have created an affordable mode of transportation for people in large cities throughout the world. Form A 43 CONTINUE ON TO THE NEXT PAGE 8. Which sentence can best follow sentence 6 to help develop the ideas in the second paragraph (sentences 5–10)? E. Bike sharing stations that are located near tourist destinations or major transportation hubs are the most popular. F. A person obtains a bike from one station, rides it for a period of time, and then returns the bike to the original station or to another station within the system. G. A mechanism called a dock keeps each bicycle securely locked to the station until a user comes along, releases the lock, and starts a ride. H. Bike sharing stations have been built in many of the largest cities in the United States, including Boston, San Francisco, Chicago, Minneapolis, and Denver. 9. What is the best way to combine sentences 8 and 9 to clarify the relationship between the ideas? A. Although they prefer bike sharing over ownership, many city bikers are not responsible for the bike’s storage or its maintenance. B. Many city bikers prefer bike sharing over ownership, and they are not responsible for the bike’s storage or its maintenance. C. Since many city bikers prefer bike sharing over ownership, they are not responsible for the bike’s storage or its maintenance. D. Many city bikers prefer bike sharing over ownership because they are not responsible for the bike’s storage or its maintenance. 10. Which revision of sentence 14 uses the most precise language? E. In 2016, data revealed that Citi Bike was used for millions of trips, which shows that the program had a 40 percent increase in ridership from the previous year. F. In 2016, nearly 14 million trips were recorded by Citi Bike, which adds up to a huge increase in ridership compared with the previous year. G. In 2016, Citi Bike documented nearly 14 million trips, which amounted to an astonishing 40 percent increase in ridership from the previous year. H. In 2016, Citi Bike saw several million trips, which turned out to be a big increase in ridership from the previous year. 11. Which sentence should be added before sentence 17 to improve the organization of the fourth paragraph (sentences 17–18)? A. Experts increasingly want to discuss the positive effects of bike sharing programs, such as Citi Bike, in big cities. B. While bike sharing programs offer clear benefits to riders, some people cite another reason for their support of the bike share model. C. If bike sharing programs are going to continue to increase in popularity, it is important for communities to discuss all the benefits these programs offer. D. Citi Bike is an innovative program that can cite positive statistics about ridership from one year to another. Form A 44 CONTINUE ON TO THE NEXT PAGE 12. Which sentence could best follow sentence 18 and support the main point of the fourth paragraph (sentences 17–18)? E. Bike share programs are developed in cities mainly to improve air quality. F. Participating in a bike share program is the main way travelers can improve air quality in cities. G. Cities may begin to experience improved air quality as more travelers use bike share programs. H. Bike share programs may be more effective at improving air quality in some cities than they are in other cities. 13. Which sentence is irrelevant to the ideas in the third paragraph (sentences 11–16) and should be deleted? A. sentence 12 B. sentence 13 C. sentence 15 D. sentence 16 14. Which concluding sentence should replace sentence 20 to better support the information presented in the passage? E. Over time, bike sharing may become a routine part of modern urban life. F. Even small- or medium-sized cities can benefit from implementing a bike share program. G. Compared with other solutions, bike sharing seems to have the most potential. H. Ultimately, bike sharing is an interesting and unique way for tourists to explore a city. Form A 45 CONTINUE ON TO THE NEXT PAGE Pursuing a Hobby (1) A hobby is an activity or interest that a person pursues for pleasure or relaxation. (2) For some it is a sport or a game, while for others it is an art, a craft, or a volunteer opportunity. (3) Becoming involved in a hobby can seem difficult and time consuming, but that should not stop people from pursuing one. (4) Hobbies can be an outlet for the stress of everyday life. (5) School, work, family responsibilities, and relationships can all compete for a person’s time and attention. (6) Many people exhibit psychological symptoms of stress, such as boredom, tension, and anxiety. (7) Others report physical symptoms, including low energy, headaches, and insomnia. (8) Reading books, creating works of art, or playing games can give the human mind a reprieve from stress. (9) But stress relief is not the only benefit of pursuing a hobby. (10) People who regularly pursue a hobby spend time in what is called active leisure. (11) Active leisure involves doing an activity that is relaxing but that also expends some mental or physical energy, such as following a pattern to knit a scarf, analyzing statistics about a favorite sports team, or doing light noncompetitive exercise. (12) During active leisure, people may experience what experts call flow, or a state of effortless concentration. (13) Pursuing a stimulating hobby can help a person find flow, which psychologists believe is more relaxing and restorative than passive leisure activities, such as watching television. (14) Many successful businesspeople and celebrities have said that they pursue hobbies in their free time. (15) Another benefit of hobbies is that they can encourage positive social interaction among people with similar interests. (16) A hobbyist might decide to do something to get better at a hobby or go to places with other people interested in the hobby. (17) Making connections and having discussions with fellow hobbyists can enhance a person’s knowledge about a hobby while fostering new friendships. (18) Free time is a precious commodity, and spending it engaged in a hobby has many advantages. (19) Finding an enjoyable hobby may take effort, but the physical, mental, and social effects of engaging in a hobby are overwhelmingly positive. 15. Which sentence should follow sentence 3 to state the main claim of the passage? A. Hobbies offer a variety of benefits, and people should set aside time to pursue them. B. People should select a hobby carefully before investing the time and resources. C. Hobbies are a productive way to spend free time, so people should pursue them. D. People should find hobbies that bring them pleasure and enjoyment. 16. Which transition word or phrase should be added to the beginning of sentence 6? E. For example F. Indeed G. As a result H. Also Form A 46 CONTINUE ON TO THE NEXT PAGE 17. Read this sentence. Active hobbies, such as jogging or yoga, can also provide relief from some of the effects of stress, because they prompt the body to release chemicals called endorphins, which can promote positive feelings. Where should this sentence be added to best support the ideas in the second paragraph (sentences 4–9)? A. between sentences 6 and 7 B. between sentences 7 and 8 C. between sentences 8 and 9 D. at the end of the paragraph (after sentence 9) 18. Which revision of sentence 16 uses the most precise language? E. A hobbyist might try to learn more about a hobby or go to events with other people who also like the same hobby. F. A hobbyist might enroll in a course related to the hobby or attend a convention with other people who enjoy the hobby. G. A hobbyist might try to find new information about a hobby or go to places where other people are involved with the hobby. H. A hobbyist might want to expand his or her knowledge of a hobby or do an activity with other people who pursue the same hobby. 19. Which sentence would best follow sentence 17 to support the ideas in the fourth paragraph (sentences 15–17)? A. Meaningful friendships are associated with an increased sense of self-confidence and happiness. B. Participating in a hobby with friends is usually more enjoyable than pursuing a hobby alone. C. Friends who enjoy pursing a hobby together will likely enjoy pursuing other hobbies together. D. Forming deep and rich friendships can become more challenging as people get older. 20. Which sentence is irrelevant to the ideas presented in the third paragraph (sentences 10–14) and should be deleted? E. sentence 10 F. sentence 11 G. sentence 13 H. sentence 14 Form A 47 CONTINUE ON TO THE NEXT PAGE READING COMPREHENSION QUESTIONS 21–57 DIRECTIONS: Read the passage below and answer the questions following it. Base your answers on information contained only in the passage. You may reread a passage if you need to. Mark the best answer for each question. 5 10 15 20 25 30 35 The eruption of the Philippine volcano Mount Pinatubo in June 1991 sent a huge cloud of gas and dust encircling the globe. The dust and ash from Mount Pinatubo was blamed for a two-year decrease in global temperature, changes in weather patterns, and damage to the ozone layer. The situation brings to mind a meteorological event that occurred 175 years earlier. At that time, harsh weather conditions plagued much of eastern North America and, to a lesser extent, northern Europe. 40 45 50 April 1816 brought typical spring weather to upstate New York and New England; trees budded, and farmers prepared to plow and plant. In May, however, the expected warm temperatures failed to arrive. Most people remained optimistic, waiting for the summer that was “just around the corner.” They waited in vain. June ushered in what modern meteorologists call “The Year Without a Summer.” During the first week of June, ten inches of snow fell on New England. Throughout the month, temperatures rarely rose above the 30s. Many farmers replanted crops several times, only to see them stunted or destroyed by sleet, hail, and icy winds. July and August brought little improvement. During most days the temperature stayed in the 40s. Farmers’ diaries document the farmers’ daily struggles with near-freezing temperatures, failing crops, and dying farm animals. The few crops that managed to survive were killed by frost in midSeptember. Winter came early in New England and was unusually severe. Even the South was affected; on July 4, the high temperature for Savannah, Georgia, was only 46 degrees! Form A 55 60 65 70 48 Some religious leaders warned their congregations that the unusual weather meant that the end of the world was drawing near. Other leaders attributed the cool weather to unusual sunspot activity. The proliferation of the newly invented lightning rod was also blamed. Some people believed that lightning rods had interrupted the natural temperature balance of Earth, causing the cooler temperatures. It was not until October that the first plausible explanation for “The Year Without a Summer” was suggested. A German astronomer, Friedrich Bessel, reported seeing thick clouds of dust in the upper atmosphere. He theorized that these dust particles screened portions of Earth from the warming rays of the sun. It was discovered that in April 1815, Mount Tambora, an Indonesian volcano, had erupted with such force that it had sent an estimated 100 cubic miles of fine dust into the atmosphere. Witnesses to the eruption reported that the sky remained dark for two days. The dust then rose high into the stratosphere, where it encircled the world for several years to come. Skeptics in 1816 doubted that a faraway volcano could steal their summer. However, most present-day researchers believe Bessel’s explanation to be generally correct, demonstrating the global nature of weather. The dust in the atmosphere eventually settled, and the spring of 1817 was back to normal. CONTINUE ON TO THE NEXT PAGE 21. Which of the following best tells what this passage is about? 24. Which of the following is implied by the phrase “the global nature of weather” (line 70)? A. the belief of some religious leaders that the end of the world was coming in 1816 B. a summer of strange weather and its probable cause C. the importance of summer weather to agriculture in New England D. a comparison of the weather of 1816 and 1991 E. Understanding weather events around the world is important for making weather predictions. F. Extreme weather conditions in some parts of the world can have a lasting impact on a geographical area. G. Natural disasters tend to occur in different parts of the world at the same time. H. Conditions in one part of the world can affect weather in another part of the world. 22. What is the most likely reason farmers persisted in replanting their crops? E. They believed that the cold weather could not continue all summer long. F. They thought that crops would be able to survive even though the weather remained cold. G. The weather had warmed up by July. H. They thought the June snowfalls would provide needed moisture. 25. The author includes the details about the eruption of Mount Tambora in lines 57–65 in order to A. emphasize that the aftermath of the eruption still affects the environment today. B. highlight the severe impact that the eruption had on the atmosphere. C. provide details about what happens during a volcanic eruption. D. suggest that weather events around the world caused the eruption. 23. In the winter that followed the summer of 1816, New Englanders most likely experienced A. new weather events that they had not encountered before. B. temperatures that were warmer than usual for that time of year. C. shortages of fruits, vegetables, and other essential crops. D. difficulty adjusting to a different time line for planting crops. Form A 49 CONTINUE ON TO THE NEXT PAGE 27. How does the third paragraph contribute to the passage? 26. The cold summer of 1816 was most likely caused by E. F. G. H. Form A A. It presents the most probable cause of the 1816 weather. B. It shows how nineteenth-century people explained the 1816 weather. C. It presents a theory about the 1816 weather that some skeptics doubted. D. It includes eyewitness reports to describe the source of the 1816 weather. unusual sunspot activity. the excessive use of lightning rods. damage to the ozone layer. an increase in dust in the atmosphere. 50 CONTINUE ON TO THE NEXT PAGE 5 10 The British novelist Charles Dickens is well known for the colorful and eccentric characters he created in his many novels. But one of his books, David Copperfield, seems to have a great deal to do with fact as well as fiction. After attempting to write his autobiography, Dickens abandoned the project and began to work on a novel, the plot of which was loosely based on his own boyhood experiences. Apparently, it was easier for him to weave the events of his own life into the fiction of David Copperfield than to write about them in nonfiction. 40 45 50 15 20 25 30 35 Some of Dickens’s most troubling memories involved a job he held in 1824 as a 12-yearold child. Because his family was deeply in debt, he was forced to quit school and go to work in a London factory, pasting labels on pots of shoe polish. Dickens lived in a boardinghouse, using his meager wages to support himself and to help pay his family’s debts. He worked in the dreary, run-down factory six days a week from 8:00 a.m. to 8:00 p.m. Such long hours were not unusual at the time, for children or adults, but Dickens was miserable during the entire four months he spent working at the factory. 55 60 Even when the family finances improved, Dickens continued to work at the factory until his father quarreled with Dickens’s boss, who promptly dismissed the son. Dickens was upset at being fired but relieved to be out of the factory. Thus he felt betrayed when his mother, anxious for the boy’s weekly wage, succeeded in making peace and getting Dickens’s job back for him. The father, however, now sided with his son, and the boy Form A 65 51 was sent back to school. “I know how these things have worked together to make me what I am,” Dickens later wrote, but he never forgot that his mother was eager for him to return to work. As an adult, Dickens always remembered the shame and humiliation he felt during those months at the factory. For years afterward, whenever in London, he could not go near the sites of the factory and boardinghouse, going out of his way to avoid those painful reminders of his past. In fact, Dickens never told his wife and children about his childhood work experience. It was only after his death that they heard of it from a family friend whom Dickens had confided in. Instead, Dickens expressed his feelings by giving his fictional “other self,” David Copperfield, a job similar to the one he had so hated. In the novel, ten-year-old David is forced by his harsh stepfather to work as a bottle washer in a factory. Young David, who “suffered exquisitely” as a child manual laborer, was apparently Dickens’s way of dealing with his own past. David Copperfield was to become Dickens’s most popular novel, and Dickens himself called it his “favorite child.” CONTINUE ON TO THE NEXT PAGE 31. Which of the following is the most likely reason that Dickens wrote David Copperfield? 28. Which of the following best tells what this passage is about? E. Dickens’s motivations for becoming a writer F. Dickens’s childhood relationship with his parents G. the autobiographical basis for Dickens’s David Copperfield H. the characters created by Dickens for David Copperfield A. He needed money from the novel to help pay his family’s debts. B. He needed a way to cope with the difficulties of his childhood. C. He wanted to avoid telling his children about his experiences in the factory. D. He wanted to explain how his childhood had influenced his adult life. 29. In the fourth paragraph, the author includes details about Dickens’s experiences as an adult in order to 32. Based on the details in the third paragraph, what can be concluded about the relationship between Dickens and his mother? A. highlight that the time Dickens spent working in the factory affected him throughout his life. B. emphasize that Dickens made sure he would never have to work in a factory again. C. highlight that Dickens did not readily share details about his childhood with others. D. emphasize that Dickens did not want his own children to experience hardships. E. He avoided her after he was allowed to return to school. F. He resented her for putting the need for his wages above his happiness. G. He hoped she would understand why attending school was important to him. H. He recognized that she had the authority to negotiate on his behalf. 33. What most directly enabled Charles Dickens to return to school? 30. The details in the second and third paragraphs suggest that young Dickens A. the discussion between his mother and his boss B. the argument between his father and his boss C. getting fired from the factory D. getting support from his father E. struggled to perform his job in the factory. F. worried about the financial situation of his family. G. wished he had a better relationship with his parents. H. preferred attending school to working in a factory. Form A 52 CONTINUE ON TO THE NEXT PAGE 5 10 When you eat an orange, your experience of its flavor comes from the combination of its aroma and its taste. Taste buds, the sensory receptors on the tongue, convey information to the brain about chemicals in food while the food dissolves in saliva. The sense of smell comes into play when the olfactory nerve in the nasal passages senses even very low concentrations of food chemicals in gaseous form. The sense of smell has a larger role in tasting flavors than most people realize— that is, until they have a stuffy nose and nothing tastes good. 40 45 50 15 20 25 If taste and smell depend on our detection of food chemicals, one might expect that chemists would be able to duplicate the flavors of foods. In fact, a surprising number of popular food flavors can now be reproduced in the laboratory, and even more are on the way. Orange, perhaps the most popular flavor worldwide, has been reproduced successfully. So have some national favorites, including cashew (Latin America), paprika (Hungary), and fruit-flavored Jamaica (Mexico). Synthetic flavors are not limited to flavoring food; they are also added to mouthwashes, toothpastes, beverages, and other consumer products. 55 60 65 30 35 Only a small proportion of the chemical components occurring naturally in foods actually contributes to their flavor. To identify these critical components, scientists use a gas chromatograph to separate a food into its basic chemical constituents. Flavor experts, called flavorists, then attempt to isolate those chemicals that are essential to the distinctive flavor of a food. Mechanical Form A 70 53 techniques have been developed to capture the aromas of food as it is being prepared— such as the smell of baking bread—and distill the essential chemicals from these essences. Flavorists use their highly developed senses of taste and smell to attempt to produce acceptable flavorings that are chemically identical to, but purer than, flavors that are naturally present in unprocessed food. Although American consumers claim to want “natural” flavors in their food, taste tests demonstrate that they often prefer their synthetically produced counterparts. Artificial flavors tend to be stronger and less subtle than natural flavors. For example, many Americans prefer a soft drink created with artificial flavors, such as orange soda, over an “all-natural” soda flavored with real oranges, which may taste weak in comparison. In fact, some flavorists worry that consumers will develop such a strong taste for artificial flavors that natural flavorings, usually more expensive than their artificial counterparts, will become scarce. Researchers have not always been successful in their efforts to duplicate natural flavors. Some popular flavors, such as coffee, strawberry, and chocolate, have proven virtually impossible to reproduce. The difficulty in creating a flavor like chocolate, experts say, is its complexity—a mysterious combination of sweet and bitter that excites the taste buds in an unusual and satisfying way. CONTINUE ON TO THE NEXT PAGE 34. Which of the following best tells what this passage is about? E. F. G. H. 37. Why does the author mention orange soda in the fourth paragraph? A. to suggest that consumer preferences for natural or artificial flavors vary B. to explain why natural flavors are more expensive than artificial substitutes C. to demonstrate that consumers sometimes prefer artificial flavors to natural flavors D. to give an example of a natural flavor that may become difficult to find in the future the relationship between smell and taste the science of how taste buds work the analysis and creation of flavors the difficulty of producing artificial flavors 35. What is the principal goal of the scientific research described in the third paragraph? A. to predict consumer taste preferences in food B. to develop food with strong flavors C. to produce synthetic equivalents of natural food flavors D. to improve the natural flavors in unprocessed food 38. The author describes the role that the sense of smell plays when experiencing a flavor (lines 6–13) in order to E. emphasize that it is easier for a person to sense the aroma of a flavor than its taste. F. highlight that the aroma of a flavor influences how a person perceives its taste. G. emphasize that synthetic flavors need to have both an aroma and a taste. H. highlight that synthetic flavors tend to have a stronger aroma than taste. 36. Which of the following can be concluded from the collection of aromas during food preparation? E. Creating artificial flavors from captured aromas is a difficult process. F. Certain chemical components of a food’s flavor are present in its aroma. G. Most people cannot tell the difference between natural flavors and artificial flavors. H. The natural flavor of a food is usually enhanced during the cooking process. Form A 39. The author includes details about the uses for synthetic flavors in lines 25–28 in order to A. show that synthetic flavors are easy to create. B. demonstrate that many items are made with the same synthetic flavor. C. suggest that synthetic flavors are healthier than natural flavors. D. highlight that synthetic flavors are found in a variety of everyday items. 54 CONTINUE ON TO THE NEXT PAGE 5 10 15 20 25 30 35 40 45 The African country of Zimbabwe took its name from the Shona word meaning “stone enclosures” or “venerated houses.” In fact, today dozens of stone ruins are scattered throughout Zimbabwe and other areas in southeastern Africa. One of these ruins, known as Great Zimbabwe, was once a fabled city that inspired tales that circulated throughout Europe. Where was this remarkable city, and who had built it? For centuries the mystery occupied the minds of explorers and treasure seekers. 50 55 The first reports to Europeans of Great Zimbabwe were spread a thousand years ago by Arab traders sailing between the Middle East and the east coast of Africa. They told of the fabulous wealth of a mysterious stone city in the African interior. In their tales, that city became associated with their understanding of Middle Eastern history— the Queen of Sheba, King Solomon, and his legendary gold mines, long since lost to the world. By the sixteenth century, Portuguese explorers regularly visited East Africa, searching for “King Solomon’s gold,” but they never found Great Zimbabwe. In 1552, a Portuguese historian, João de Barros, recorded a story told by the Arabs about a city with a “square fortress of masonry within and without, built of stones of marvelous size, and there appears to be no mortar joining them.” 75 In the twentieth century, after excavating areas that had not been disturbed, David Randall-MacIver, a Scottish Egyptologist, and Gertrude Caton-Thompson, an English archaeologist, concluded that the ruins were unmistakably African in origin. Great Zimbabwe was most likely built during the fourteenth or fifteenth century by the ancestors of the present-day Shona people. Recent carbon-14 dating supports their conclusion. Great Zimbabwe was once home to an estimated 20,000 people, the center of a great Shona kingdom. Wealthy Shona kings traded their ivory and gold in coastal towns for other goods, thus accounting for the discovery of beads and other foreign wares in the ruins. 80 One mystery of Great Zimbabwe had been solved. Another mystery remains: why was the settlement at Great Zimbabwe abandoned, leaving the magnificent stone architecture to fall into ruins? 60 65 70 In fact, Great Zimbabwe was a marvel. In one area, a massive wall, over thirty feet high and twenty feet thick, created a great enclosure. Another area contained a fortresslike series of walls, corridors, and steps built into the bluff above. Throughout the city, each stone was precisely fitted to the others without the use of mortar. built by the Queen of Sheba. British authorities sent a British journalist, Richard Hall, to Great Zimbabwe to investigate Mauch’s report. Archaeology was still in its infancy, and Hall, convinced that the structures had been built by ancient people from the Middle East, dug up and discarded archaeological deposits that would have revealed much about the true history of Great Zimbabwe. Later European excavations destroyed even more valuable evidence. In the 1870s, a German geologist, Karl Mauch, was the first European to see Great Zimbabwe, by then in ruins. Mauch realized that he had “rediscovered” the fabled city from de Barros’s story. He jumped to the conclusion that Great Zimbabwe had been Form A 55 CONTINUE ON TO THE NEXT PAGE 43. Which of the following statements about the Shona people is best supported by the passage? 40. Which statement best describes the central idea of the passage? E. Great Zimbabwe was an enormous stone city thought to be home to some of the greatest treasure of ancient history. F. Mysteries related to Great Zimbabwe continue to interest historians and explorers even though archaeologists have confirmed its origins. G. The history of Great Zimbabwe was subject to much speculation until modern archaeologists definitively determined its origins. H. Early missteps in the study and excavation of the Great Zimbabwe ruins led to the loss of valuable evidence about the city. A. They live along the east coast of Africa. B. They are descendants of the people who built Great Zimbabwe. C. They lived in the Middle East before settling in Africa. D. They were once ruled by King Solomon and the Queen of Sheba. 44. The conclusions of David Randall-MacIver and Gertrude Caton-Thompson were significant mostly because they E. proved that Great Zimbabwe was much older than previously thought. F. questioned why the Shona people left Great Zimbabwe. G. supported the idea that ancient Shona society was robust. H. revealed that Great Zimbabwe was created by the ancient Shona people. 41. What was the main way that Karl Mauch’s conclusions about Great Zimbabwe (lines 45–47) affected later archaeological investigations? A. Archaeologists from all over Europe became interested in excavating the area. B. Archaeologists made assumptions about the history of the ruins before excavating. C. Archaeologists started to believe that many of the past accounts recorded about the ruins were true. D. Archaeologists realized it was unlikely that an ancient culture could build such a grand structure. 45. Which of the following best describes the relationship of Portuguese explorers to Great Zimbabwe? A. They searched for it but were never able to find it. B. They hoped it was as grand as they had been told by traders. C. They knew it was unlikely they would find the stone city. D. They had to rely on information from others in order to search for it. 42. What was “one mystery of Great Zimbabwe” (line 76) that had been solved? E. when the settlement was abandoned and why F. why there was no ivory or gold found in the ruins G. why the ruins remained undiscovered until the 1870s H. who had built the settlement and when Form A 56 CONTINUE ON TO THE NEXT PAGE 5 10 15 20 25 30 35 In many cultures, the ugly physical appearance of the bat has given it an unearned reputation as an evil and vicious bearer of diseases. Many people, for example, believe that little brown bats carry rabies. In fact, they are no more likely to transmit the disease than other animals, such as dogs. Brown bats actually help prevent disease, not spread it. The basis of their diet is the mosquito, an insect that transmits more diseases than all the bats in the world combined. 40 45 A group of bat species known as flying foxes or fruit bats serves another important purpose as a critical link in the reproduction of many tropical trees and shrubs. In the tropical rain forests of Africa, Asia, and Australia, plants such as avocados, date trees, cashews, and mangoes rely in part on flying foxes for pollination. One of Africa’s most valuable hardwood trees, the iroko, is entirely dependent on this type of bat for pollination. Flying foxes feed on flowers, fruit, and nectar, flying from one plant to another and pollinating the plants as they go, much as bees do in other parts of the world. Because they are sloppy eaters, flying foxes drop fruit as they go, dispersing the seeds. They can travel great distances and convey pollen and seeds far from their origins, thereby maintaining the genetic biodiversity within a plant species. 50 55 60 65 depend on them for food and shelter—would be threatened to the point of extinction. Areas outside the rain forests would be affected as well, since the rain forests’ lush vegetation replenishes the oxygen in the global atmosphere. Unfortunately, many people are determined to get rid of bats. Flying foxes are at particular risk. In the wild, they feed on wild fruit, but when their rain forest habitat is reduced by conversion into farmland or residential areas, they occasionally raid cultivated fruit trees, spoiling the crops. Several flying fox species have been hunted to extinction, while others are seriously endangered. Conservation groups and government agencies in many countries are attempting to change people’s attitudes toward bats. When people learn that bats pollinate the trees and crops that provide their livelihood, they are more likely to appreciate and protect the bats in their area. There are also effective, nonharmful ways to deal with troublesome bats. Orchard owners can cover their trees with netting to discourage the bats, and there are humane methods for moving bats from places where they are not wanted. For the sake of the rain forests and for life-forms everywhere that depend on them, it is urgent that people apply a new twist to an old adage and realize that ugliness is only skin deep. Because of the importance of bats’ role in pollination and seed distribution, scientists consider them a keystone in the ecosystems of tropical rain forests. Without bats, many bat-pollinated plants—and the animals that Form A 57 CONTINUE ON TO THE NEXT PAGE 49. Which evidence best supports the claim that the bat is important to the survival of certain species? 46. Which statement best reflects the central idea of this passage? E. Bats provide numerous benefits to the environment and should be protected. F. Bats are still considered pests even though people understand that bats are helpful. G. Bats help limit the spread of disease by eating insects that carry diseases. H. Bats are responsible for pollinating a variety of rain forest plants across great distances. A. the details about how bats pollinate the iroko tree B. the information about how bats help maintain genetic diversity C. the details about how bats sometimes eat cultivated fruit trees D. the information about how bats consume mainly mosquitoes 50. In the third paragraph, the author includes the details about the bat’s role as a keystone species in order to 47. Which of the following best describes animal species that function as a “keystone” (line 35)? E. convey that bats help distribute seeds throughout the rain forest. F. show that bats provide food and shelter for other animals. G. emphasize that threats to bats would affect the rain forest ecosystem. H. highlight that bats help maintain the levels of oxygen in the atmosphere. A. They can be threatened with extinction. B. They are a food source for other animals. C. They pollinate many different types of plants. D. They help maintain the balance of their environment. 48. Which statement best describes the author’s opinion about bat conservation? 51. What does the author intend to convey by the statement “ugliness is only skin deep” (line 69)? E. Allowing bats to occasionally eat crops and fruit from cultivated trees is a good way to prevent bats from becoming endangered. F. Because bats can support the growth of many different species, they should be relocated to areas with struggling ecosystems. G. Although bats may seem to be a nuisance, it is vitally important that they are protected and managed in humane ways. H. Protecting the bat population takes priority over concerns about bats ruining cultivated trees and crops. Form A A. Certain species of bats have a less appealing physical appearance than other species of bats. B. People should avoid making assumptions about bats based on the animals’ physical appearance. C. People should avoid removing bats from certain areas because of the physical appearance of bats. D. Efforts to conserve bats are worthwhile even though people dislike the animals’ physical appearance. 58 CONTINUE ON TO THE NEXT PAGE 5 10 15 20 25 30 35 Imagine living in a society where ordinary people could be punished for what they choose to read and write. For much of the twentieth century, such a closed society existed in Russia and the rest of the Soviet Union. The Soviet government tried to dominate its citizens’ activities and ideas by controlling the information that they received. Government censors examined books, films, and newscasts and banned anything they considered objectionable. They censored criticism of the Soviet government, news from the outside world, and anything that complimented Soviet enemies. 40 45 50 The Soviet government’s strict censorship made life tremendously difficult for writers. Most worried that they were being watched by the government’s secret police. Despite the harsh laws, small groups of writers dodged state censorship through an underground, or secret, publishing network that produced works called samizdat. The name samizdat came from the Russian words for “self ” and “publish.” For many writers, samizdat offered the only outlet for their intellectual and creative expression. To produce samizdat, an author passed a typed or handwritten text to a second person, who made a handwritten or typed copy. The original was returned to the author, while the copy was passed to other members of the network. The works were unsigned or signed with false names. 55 60 65 At first, samizdat focused mainly on literature, such as poetry and novels. By the late 1950s, samizdat circles were distributing political material, such as letters to the government, political essays, and trial Form A 70 59 transcripts. By the mid-1960s, the samizdat network produced sophisticated political news, debate, and analysis. The great Russian novelist Boris Pasternak had his work published as samizdat. Like other writers, he feared that an appearance of disloyalty to the Soviet state would bring a knock at his door in the middle of the night. His classic novel Doctor Zhivago was smuggled out of the Soviet Union for publication in Western countries in 1956; in Russia, it appeared only as samizdat. Pasternak won the Nobel Prize in Literature in 1958, but the government forced him to refuse the prize. Soviet authorities also blocked publication of the work of Anna Akhmatova, one of Russia’s greatest poets. Her work was banned until 1952 because censors thought she did not sufficiently praise the Soviet government. Akhmatova was kept out of public life and the official Writers’ Union. She composed her poetry in private, and her works were available only as samizdat. Through the 1960s and ’70s, Russian writers used samizdat networks to circulate banned or politically risky material. By the late 1980s, computers became available in scientific research facilities, and underground writers began using the computers to store and circulate texts. Censorship was officially abolished in 1989, shortly before the breakup of the Soviet Union, leading to a publishing boom. Works by previously banned authors were published, and the samizdat networks quickly faded into history. CONTINUE ON TO THE NEXT PAGE 52. Which of the following best tells what this passage is about? 55. Which event led to a change in how samizdat was created and shared? E. the Russian writers Boris Pasternak and Anna Akhmatova F. the poetry published in the Soviet Union during the twentieth century G. censorship in the Soviet Union and the underground system that arose in response H. the reasons for banning writers who criticized the Soviet government A. government authorities censoring Akhmatova’s work B. the increase in the distribution of political essays C. Pasternak winning the Nobel Prize D. the rise in the use of computers and computer networks 56. What is the most likely reason that samizdat materials were unsigned or signed with false names? 53. Which of the following is most likely an example of material circulated in the earliest phase of samizdat? E. to allow the materials to be smuggled out of the country F. to protect the writer from punishment G. to avoid breaking censorship laws H. to differentiate between original and copied works A. a letter protesting a writer’s imprisonment B. a short story C. a list of political prisoners D. an article from a foreign newspaper 57. Why did samizdat networks quickly disappear in 1989? 54. The phrase “a knock at his door in the middle of the night” (lines 44–45) most likely refers to a visit by E. F. G. H. Form A A. Samizdat networks were unnecessary after censorship was abolished. B. Samizdat networks became regular publishing companies. C. Samizdat networks were replaced by the use of computers. D. Samizdat networks went deeper underground than ever before. the secret police. a representative from a major publisher. a participant in the samizdat network. people from other countries. 60 CONTINUE ON TO THE NEXT PAGE PART 2 — MATHEMATICS Suggested Time — 90 Minutes 57 QUESTIONS IMPORTANT NOTES (1) Formulas and definitions of mathematical terms and symbols are not provided. (2) Diagrams other than graphs are not necessarily drawn to scale. Do not assume any relationship in a diagram unless it is specifically stated or can be figured out from the information given. (3) Assume that a diagram is in one plane unless the problem specifically states that it is not. (4) Graphs are drawn to scale. Unless stated otherwise, you can assume relationships according to appearance. For example, (on a graph) lines that appear to be parallel can be assumed to be parallel; likewise for concurrent lines, straight lines, collinear points, right angles, etc. (5) Reduce all fractions to lowest terms. GRID-IN PROBLEMS QUESTIONS 58–62 DIRECTIONS: Solve each problem. On the answer sheet, write your answer in the boxes at the top of the grid. Start on the left side of each grid. Print only one number or symbol in each box. DO NOT LEAVE A BOX BLANK IN THE MIDDLE OF AN ANSWER. Under each box, fill in the circle that matches the number or symbol you wrote above. DO NOT FILL IN A CIRCLE UNDER AN UNUSED BOX. 58. x° The owner of a tree farm plants pine trees and oak trees in a ratio of 8:3. How many oak trees are planted if 264 pine trees are planted? 60. For what value of w is 4w = 2w − 8 ? R Q 72° P 59. S In the figure above, PQRS is a parallelogram. What is the value of x ? Form A 61 CONTINUE ON TO THE NEXT PAGE 61. 62. A survey asked students what pets they have. Based on the results, the following statements are all true. • 20 students have cats. The sum of two consecutive integers is −15 . If 1 is added to the smaller integer and 2 is subtracted from the larger integer, what is the product of the two resulting integers? • 23 students have dogs. • 3 students have both dogs and cats. • 5 students have no dogs or cats. How many students were surveyed? Form A 62 CONTINUE ON TO THE NEXT PAGE MULTIPLE CHOICE PROBLEMS QUESTIONS 63–114 DIRECTIONS: Solve each problem. Select the best answer from the choices given. Mark the letter of your answer on the answer sheet. You can do your figuring in the test booklet or on paper provided by the proctor. DO NOT MAKE ANY MARKS ON YOUR ANSWER SHEET OTHER THAN FILLING IN YOUR ANSWER CHOICES. 63. A. B. C. D. 64. {3, 4, 5} {4, 5, 6} {8, 10, 12} {10, 14, 18} Jenny starts a game with twice as many marbles as Keiko. Jenny gives Keiko 5 marbles, but she still has 10 more than Keiko. How many marbles did Jenny have to start with? E. F. G. H. One bottle contains 500 milliliters of juice. How many liters of juice are there in 24 of these bottles? E. F. G. H. 65. 66. The set of possible values of m is {5, 7, 9}. What is the set of possible values of k if 2k= m + 3 ? 67. 12 L 120 L 1,200 L 12,000 L 25 30 35 40 In a scale diagram, 0.125 inch represents 125 feet. How many inches represent 1 foot? A. B. C. D. 0.001 in. 0.01 in. 0.1 in. 0.12 in. In a certain school, course grades range from 0 to 100. Adrianna took 4 courses and her average course grade was 90. Roberto took 5 courses. If both students have the same sum of course grades, what was Roberto’s average? A. B. C. D. Form A 72 80 90 92 63 CONTINUE ON TO THE NEXT PAGE 68. 69. PEOPLE PER VEHICLE AT CHECKPOINT Number of People Percent of in Vehicle Vehicles 1 40% 2 35% 3 15% R N P M 4 7% 5 or more 3% S Q A researcher recorded the number of people in each vehicle that passed through a checkpoint. The table above shows the percent distribution for the 420 vehicles that passed through the checkpoint yesterday morning. How many of the 420 vehicles contained at least 3 people? In the pyramid above, each triangular face has the same area, and the base MNPQ is a square that measures 8 centimeters on each side. If the length of RS  = 6 centimeters, what is the surface area of the pyramid excluding the base? E. F. G. H. A. B. C. D. 42 63 105 315 70. The perimeter of a rectangle is 510 centimeters. The ratio of the length to the width is 3:2. What are the dimensions of this rectangle? E. F. G. H. Form A 64 48 sq cm 96 sq cm 128 sq cm 160 sq cm 150 cm by 105 cm 153 cm by 102 cm 158 cm by 97 cm 165 cm by 90 cm CONTINUE ON TO THE NEXT PAGE 71. 74. Which number line below shows the solution to the inequality −4 < x < 2 ? 2 A. –10 –5 0 5 10 B. –10 –5 0 5 10 C. –10 –5 0 5 10 D. –10 –5 0 5 10 The sum of the numbers x , y , and z is 50. The ratio of x to y is 1:4, and the ratio of y to z is 4:5. What is the value of y ? E. F. G. H. 4 8 10 20 75. y 2 72. 1 dollar = 7 lorgs 1 dollar = 0.5 dalt 1 Kevin has 140 lorgs and 16 dalts. If he exchanges the lorgs and dalts for dollars according to the rates above, how many dollars will he receive? E. F. G. H. 73. O 1 2 x What is the area of the shaded region in the graph above? $28 $52 $182 $282 A. B. C. D. 0.25 square unit 0.5 square unit 1 square unit 1.5 square units A box of colored pencils contains exactly 6 red 76. pencils. The probability of choosing a red pencil from the box is 2 . How many of the 7 pencils in the box are not red? A. B. C. D. Form A 5 15 21 30 In Centerville, 45% of the population is female, and 60% of the population commutes to work daily. Of the total Centerville population, 21% are females who commute to work daily. What percentage of the total Centerville population are males who do not commute to work daily? E. F. G. H. 65 15% 16% 24% 39% CONTINUE ON TO THE NEXT PAGE 77. A. B. C. D. 78. 79. Mrs. Cranston bought five bottles of water for $0.90 each and 8 pounds of meat. She paid a total of $26.90 for these items, not including tax. What was the price per pound of the meat? A nation has five types of coins: sinds, dalts, lorgs, harps, and plunks. The relationship between the coins is shown above. Which coin is most valuable? $2.80 $3.25 $14.40 $22.40 A. B. C. D. In a sample of 10 cards, 4 are red and 6 are blue. If 2 cards are selected at random from the sample, one at a time without replacement, what is the probability that both cards are not blue? E. 2 15 F. 4 25 G. 3 10 1 sind = 4 lorgs 2 plunks = 5 dalts 5 sinds = 2 harps 1 plunk = 3 harps sind dalt harp plunk 80. SCORES ON MATH QUIZ H. 1 3 Score Number of Students 85 4 75 4 65 2 What is the mean score of the 10 students in the table above? E. F. G. H. Form A 66 22.5 75 77 85 CONTINUE ON TO THE NEXT PAGE 83. 81. HOW PEOPLE GET TO WORK IN CENTER CITY Bicycle 4% Car Pool 15% In a scale drawing of a triangular banner, one side measures 16 centimeters and the other two sides each measure 12 centimeters. On the actual banner, these two sides each measure 36 feet. What is the length of the remaining side of the actual banner? A. B. C. D. 16 ft 32 ft 40 ft 48 ft Walk 22% 84. Bus 10% Drive Alone 49% The faculty of a certain four-year college consists of 179 teachers. There are 663 first-year students. The student-to-faculty ratio for the entire college is 15 to 1. What is the total number of second-, third-, and fourth-year students? E. F. G. H. 1,989 2,022 2,652 2,685 Total number of people working in Center City = 15,000 How many more people in Center City walk to work than ride their bicycle to work? A. B. C. D. 82. What is the value of the expression shown above? A. 14 7 20 B. 14 2 5 Which of the following numbers has factors that include the smallest factor (other than 1) of 91? E. F. G. H. Form A 2,500 2,700 2,800 3,000 21 +3 3 + 4 2 +51 5 10 5 2 85. C. 15 7 20 D. 15 2 5 30 35 39 44 67 CONTINUE ON TO THE NEXT PAGE 86. 87. E. 55 i 5,280 1 F. 55 i 5,280 3,600 G. 55 i 3,600 5,280 H. 55 i 5,280 60 89. 91. 4 years old 6 years old 12 years old 16 years old 92. 12 15 30 36 For a presentation, Deion can create 5 slides in 20 minutes, working at a constant rate. Kyra can create 3 slides in 10 minutes, working at her own constant rate. What is the total number of slides the two of them can create in one hour? E. F. G. H. 0 1 2 12 $1,900 $4,600 $10,800 $11,800 There are 6 different cookies on a plate. Aiden will choose 2 of these cookies to pack in his lunch. How many different pairs of 2 cookies can he choose from the 6? A. B. C. D. How many positive even factors of 48 are greater than 24 and less than 48? E. F. G. H. Johan leased a car for three years. He paid a one-time fee of $1,000, and an additional $300 per month for the full three years. At the end of the three years, what is the total amount Johan paid for leasing this car? E. F. G. H. Today, Tien’s age is 1 of Jordan’s age. In 4 2 years, Tien’s age will be 1 of Jordan’s age. 3 How old is Jordan today? A. B. C. D. 88. 90. A car is traveling 55 miles per hour, and 1 mile = 5,280 feet. Which of the following calculations would give the car’s speed in feet per second? 16 30 33 55 The least of 5 consecutive integers is l, and the l+ g greatest is g. What is the value of  in 2 terms of l? A. 2l B. 3l C. l + 2 D. l + 5 Form A 68 CONTINUE ON TO THE NEXT PAGE 93. 95. L N Ryan must read 150 pages for school this weekend. It took him 30 minutes to read the first 20 pages. At this rate, how much additional time will it take him to finish the reading? A. 2 1 hr 6 B. 3 1 hr 4 On the number line above, LN = 1 . Point M 8 (not shown) is located between point L and C. 3 3 hr 4 point N. Which value below is a possible value D. 7 1 hr 2 for M? A. B. C. D. 94. 4.26 4.31 4.35 4.58 96. An unmarked straight stick will be laid end over end to measure a distance of exactly 72 feet. The same stick will be used in the same way to measure a distance of exactly 30 feet. What is the length of the longest possible stick that can be used for both measurements? E. F. G. H. 3 ft 4 ft 6 ft 8 ft 97. y Suppose M = w ,  N = , and w, x, y, and z z x do not equal 0. What is M in terms of w, x, y, N and z ? E. wx yz F. wy xz G. wz xy H. xy wz In the set of consecutive integers from 12 to 30, inclusive, there are four integers that are multiples of both 2 and 3. How many integers in this set are multiples of neither 2 nor 3? A. B. C. D. Form A 69 5 6 13 15 CONTINUE ON TO THE NEXT PAGE 98. 10 9 8 7 Number 6 5 of 4 Schools 3 2 1 0 In the equation above, w , x , y , and z are positive numbers. Which of these is equal to z ? NUMBER OF SCHOOLS IN FIVE CITIES M N P City Q R E. F. G. H. B. 9 100 C. 7 15 D. 49 100 Form A F. xy G. w xy H. xy w –10 X Y 0 Z 8 On the number line above, points W, X, Y, and Z are integers, and WX:XY:YZ = 4:2:3. What is the value of WY? A. B. C. D. 102. A box contains 5 strawberry candies, 3 banana candies, and 2 orange candies. If Braden selects 2 candies at random from this box, without replacement, what is the probability that both candies are not banana? 1 15 x W City M City P City Q City R A. E. 101. The graph above shows the number of schools per city for five small cities. Cities M and N each have 500 students per school. City P has 400 students per school. Cities Q and R each have 700 students per school. Which of the five cities has the greatest number of students? 99. w = y x z 100. A metal plate used in an electronic device must have a thickness of 0.02 inch, with an allowable error of 1 percent. What is the greatest allowable thickness of the metal plate? E. F. G. H. 70 8 11 12 18 0.0002 in. 0.02 in. 0.0202 in. 0.03 in. CONTINUE ON TO THE NEXT PAGE 103. 106. radius of each tire on the car is 1 foot. How SCORES ON BIOLOGY TEST Section Lowest Score Range I 65 28 II 62 25 III 67 22 many revolutions does one of these tires make in 1 minute? (Use the approximation 22  for 7 .) E. F. G. H. Mr. Blake’s biology class is divided into three sections. The same test was given to each section. The table above shows both the lowest score and the range of scores on this test for each section. What is the overall range of all scores in all three sections? A. B. C. D. 104. 107. 25 27 28 31 108. 2 3 4 5 10 = 0.769230 13 105. 700 1,925 13,828 15,400 100( 2 + 0.1)2 − 100 = A. B. C. D. If 3n  is a positive even number, how many odd numbers are in the range from 3n up to and including 3n + 5 ? E. F. G. H. A car travels at 4,400 feet per minute. The 101 200 301 341 A sports store has a container of handballs: 4 blue, 5 red, 8 yellow, 9 white, and 11 green. If one ball is picked from the container at random, what is the probability that it will be yellow? E. 1 37 F. 1 8 G. 8 37 H. 8 29 In the infinitely repeating decimal above, 7 is the first digit in the repeating pattern. What is the 391st digit? A. B. C. D. Form A 0 3 6 7 71 CONTINUE ON TO THE NEXT PAGE 109. Each week, Leon has fixed expenses of $1,250 at his furniture shop. It costs him $150 to make a chair in his shop, and he sells each chair for $275. What is Leon’s profit if he makes and sells 25 chairs in 1 week? A. B. C. D. 110. 111. J L M 3 8 $1,875 $2,500 $3,125 $4,375 On the number line above, JK = 3 1 , 2 JM = 9 3 , and LM = 1 1 . What is the 4 8 position of point L ? Using the approximation 2.54 centimeters = 1 inch, how many centimeters are in 4 feet 7 inches? E. F. G. H. K A. 5 1 8 21.65 cm 119.38 cm 121.92 cm 139.70 cm B. 5 1 4 C. 5 1 2 D. 6 1 4 112. If 4 x − 3 y = 12 , what is x  in terms of y ? E. = x F. 3 y + 12 4 x = − 3 y + 12 4 G. = x 3 y+3 4 H. x = − 3 y + 3 4 Form A 72 CONTINUE ON TO THE NEXT PAGE 114. 113. SERVINGS OF FRUITS AND VEGETABLES Number of Servings of Fruits and Vegetables Number of Students 0 5 1 7 2 3 3 4 4 0 5 1 A paste is made by mixing the following ingredients by weight: 4 parts powder, 3 parts water, 2 parts resin, and 1 part hardener. One billboard requires 30 pounds of this paste. How many total pounds of resin are required for 4 billboards? E. F. G. H. 6 lb 8 lb 24 lb 48 lb There are 20 students in a class. The frequency table above shows the number of students in this class who ate 0, 1, 2, 3, 4, or 5 servings of fruits and vegetables yesterday. What is the mean number of servings of fruits and vegetables eaten yesterday per student in this class? A. 1 1 2 B. 3 C. 3 1 3 D. 4 THIS IS THE END OF THE TEST. IF TIME REMAINS, YOU MAY CHECK YOUR ANSWERS. BE SURE THAT THERE ARE NO STRAY MARKS, PARTIALLY FILLED ANSWER CIRCLES, OR INCOMPLETE ERASURES ON YOUR ANSWER SHEET. j Form A 73 SAMPLE TEST – ENGLISH LANGUAGE ARTS EXPLANATIONS OF CORRECT ANSWERS AA FORM uses the pronoun “they” to correctly refer to both girls again. Sentence 4 (Option D) uses the pronoun “they” to correctly refer to “both girls.” Sentence 3 (Option C) is the only sentence where the pronoun is vague. The sentence uses the pronoun “she” near the beginning, but whether “she” refers to Eliza or Brianna is unclear. REVISING/EDITING PART A 1. (D) The question asks for the most precise revision for the words talked to some people who did the best in the contest. Option A and Option C do not precisely state how many people the reporter interviewed. Option B specifies the number of people interviewed but uses the imprecise phrase “who did well.” Option D is the only option that precisely states the reporter’s action (“interviewed”) as well as who exactly the reporter interviewed (“the top three contestants”). 6. (E) The sentence in the box needs a comma to separate coordinate adjectives. Option F incorrectly adds a comma between the subject and the verb. Option G incorrectly adds a comma between two adjectives, but they are not coordinate adjectives. Option H is incorrect because a comma is not needed to set off the phrase “to score 100 points in a single game.” Option E is the only option that places a comma where it is needed, between the coordinate adjectives “agile” and “athletic.” 2. (G) The question asks for the identification of a sentence with an inappropriate shift in verb tense. Option E, Option F, and Option H identify sentences that use past tense. Sentence 3 (Option C) demonstrates an incorrect shift into the present tense with the clause “as natural gas becomes more common,” which should be “as natural gas became more common.” REVISING/EDITING PART B Unlock, Ride, Return 3. (C) The sentence in the box needs a comma to separate items in a series. Option A would remove the necessary comma before the nonrestrictive clause “which is located on Midway Street.” Option B is incorrect because while the phrase that ends with the word “volunteer” can stand on its own as an independent clause, a comma is not needed before the explanation of what the volunteers would help do. Option D would delete a necessary comma between two items in a series. Option C is the only option that would place a necessary comma to separate items “walking dogs” and “cleaning kennels” in the series. 7. (A) The correct answer must state the topic of the passage and hint at some of the supporting ideas. Option B mentions the idea that bike sharing programs are successful because both residents and tourist can use them, but that detail is mentioned in only the second paragraph. Option C mentions that bike sharing provides a faster mode of transportation, but it is referring to a detail suggested in sentence 16. Option D might seem attractive because it states a specific benefit of bike sharing, but it does not encompass other key points of the passage. Option A is the only option that clearly states the topic of the passage and refers to multiple key points. 4. (E) The question asks for the best way to combine the sentences to clarify the relationship between the ideas. Option F is incorrect because the conjunction “although” suggests that scientists were allowed to collect data even though there were flyby missions, which is inaccurate. In Option G, the phrase “which have been happening since 1973” is incorrectly modifying “the planet and its moons.” In Option H, the conjunction “but” suggests an adverse relationship between ideas, which is also incorrect. Option E is the only option that accurately reflects the relationship between the ideas by using the nonrestrictive phrase “which allow scientists to collect data about the planet and its moons” to describe the purpose of the flyby missions. 8. (F) Because it will be placed after sentence 6, the correct answer needs to provide appropriate details that help the reader understand sentences 5 and 6. Sentences 5 and 6 begin to explain how bike sharing programs work, so the correct answer should provide additional general details that help the reader picture how people use the program. The detail stated in Option E may be true, but the detail relates to where bike sharing stations are located, not to how a bike sharing program works. Similarly, Option H includes details about major United States cities that have bike sharing programs and does not provide more information about how a bike sharing program works. Option G might seem attractive because it provides very specific details about how the bike stations keep bicycles secure. However, this level of detail is too specific to help the reader understand the program as a whole. Only Option F includes details that give the reader a clear idea of how people use bike sharing programs. 5. (C) The question asks the student to identify a vague pronoun in a sentence in the box. Sentence 1 (Option A) correctly uses the plural pronouns “their”/”they” to refer to both Eliza and Brianna. Sentence 2 (Option B) 74 SAMPLE TEST – ENGLISH LANGUAGE ARTS EXPLANATIONS OF CORRECT ANSWERS continued... 9. (D) The correct answer needs to have the same relationship between ideas as the one implied in sentences 8 and 9. Context tells the reader that there is a causal relationship between city bikers’ lack of responsibility and their preference for bike sharing. The subordinate clause in Option A, “Although they prefer bike sharing over ownership,” presents an opposing idea, giving the impression that city bikers prefer bike sharing even though they are not responsible for storage and maintenance, which is an inaccurate way to connect these ideas. In Option B, the linking word “and” connects the two ideas but does not clarify that one idea is the cause of the other. Option C presents a causal relationship, but it reverses the relationship between the ideas. The use of “since” in Option C suggests that city bikers’ preference for bike sharing is the reason they are not responsible for bike storage and maintenance. Only Option D accurately expresses the relationship that the original two sentences implied. The transition word “because” clarifies that avoiding the responsibility for storage and maintenance is the reason city bikers prefer bike sharing. Form A this option is incorrect because the paragraph focuses on how improved air quality is an effect of people using bike sharing programs. Option F indicates that participating in a bike sharing program is the principal way travelers can improve air quality, but that does not accurately connect the ideas in the paragraph. Option H suggests that bike sharing programs in some cities are more effective at improving air quality than bike sharing programs in other cities, but this option is incorrect because it does not relate to the main topic of the paragraph: there is a relationship between a city’s implementation of a bike sharing program and improved air quality in the city. Option G is correct because it makes a connection between the increasing use of bike sharing programs and the potential impact on air quality. 13. (B) The correct answer must be a sentence that does not relate to the topic of the third paragraph. Option A (sentence 12), Option C (sentence 15), and Option D (sentence 16) are relevant to the paragraph because they provide information about the bike sharing program in New York City. Option B (sentence 13) relates to a program in a city in China, which is not the focus of the third paragraph. 10. (G) The correct answer should use the most precise and specific details and language. Option E uses an imprecise word, “millions,” which does not provide a specific number. Option F specifies “14 million,” but it uses the imprecise “huge increase” rather than a specific number. Option H uses the imprecise phrase “several million” rather than the precise number of trips, and it uses the imprecise phrase “big increase” rather than the specific number for the increase. Option G is the only option that includes precise numbers and language. 14. (E) The correct answer should provide a logical conclusion based on the details about the transportation and environmental benefits of bike sharing programs. Option F makes a prediction about small- and mediumsized cities that is not supported by the passage, which mentions only large cities. Option G compares bike sharing with other transportation methods, but this option is an incorrect conclusion because the passage focuses on the benefits of bike sharing programs, not comparing bike sharing to other modes of transportation. Option H states that bike sharing is useful for tourists, which does not provide a conclusion for the details about transportation and the environmental impact of bike sharing programs. Only Option E could follow sentence 19 by presenting the conclusion that bike sharing programs will become more routine because of the benefits outlined in the passage. 11. (B) The correct answer must transition from the third paragraph and provide a topic sentence for the fourth paragraph. Option A presents information (“experts increasingly want to discuss”) that was not addressed in the passage. Option C might seem attractive because it mentions information from earlier paragraphs about the popularity of bike sharing programs, but it does not accurately preview the fourth paragraph. Option D restates information from the third paragraph but does not relate to the information in the fourth paragraph. Only Option B includes a transition from the previous paragraph and an accurate topic sentence for the fourth paragraph. Pursuing a Hobby 15. (A) The question asks for a sentence that should follow sentence 3 and state the main claim of the passage. Option B suggests that people should think carefully about selecting a hobby to pursue, which may be true, but it is only a minor detail implied in the last paragraph. Option C claims that people should pursue hobbies because hobbies are a productive way to spend 12. (G) The correct answer must relate to the topic of the fourth paragraph: the relationship between bike sharing programs and improved air quality in a city. Option E states that improving air quality is the main reason cities establish bike sharing programs, but 75 SAMPLE TEST – ENGLISH LANGUAGE ARTS EXPLANATIONS OF CORRECT ANSWERS continued... Form A 19. (A) The question asks for a sentence that follows sentence 17 and supports the ideas in the paragraph: hobbies can encourage positive social interaction. Pursuing a hobby with friends (Option B) may be enjoyable, but this does not explain the benefits of engaging in positive social interaction. Option C states that friends who enjoy one hobby may enjoy other hobbies, but this is not the idea that needs to be supported in sentence 17. Option D addresses the idea that forming relationships becomes more difficult as people grow up, which does not support the ideas in the paragraph. Option A is the only option that provides support for sentence 17 by making the point that having meaningful friendships stemming from interest in a hobby may be associated with a variety of positive outcomes. free time. This idea is implied in sentence 18, but it is not the main claim of the passage. Option D addresses the idea that having a hobby should be enjoyable, which is addressed in sentence 1, but this is a general description of hobbies, not the main claim of the passage. Option A is the only option that logically follows the statement in sentence 3 and presents the main claim that people should make time to pursue a hobby because hobbies have a variety of benefits. This claim is developed in sentences 4, 9, 13, and 17. 16. (G) Sentence 5 states the aspects of everyday life that can cause stress. Sentence 6 lists several physical symptoms that can result from stress. Option E incorrectly suggests that sentence 6 is an example of sentence 5. Option F suggests that sentence 6 provides confirmation of the point made in sentence 5, which is incorrect. Option H suggests that sentence 6 is the addition of examples similar to those presented in sentence 5, which is also incorrect. Option G is the only option that provides a transition to show that the symptoms in sentence 6 can happen as a result of the stressors listed in sentence 5. 20. (H) The question asks for a sentence that is irrelevant to the development of ideas in the third paragraph of the passage. Sentences 10 (Option E), 11 (Option F), and 13 (Option G) are essential to the paragraph in order to explain active leisure and flow. The idea in sentence 14 (Option H) relates to the hobbies of celebrities and businesspeople, which is not relevant to the description and benefits of engaging in active leisure. 17. (C) The sentence in the box names some active hobbies and states that these hobbies release endorphins to promote positive feelings and override some effects of stress. Option A places this sentence between sentences 6 and 7, which are both sentences that deal with symptoms of stress and not the hobbies themselves. Option B places the sentence in the box between sentences 7 and 8. Sentence 8 begins a list of some hobbies and how they can give the mind a break from stress. Since the sentence in the box uses the phrase “can also provide,” this sentence needs to go after sentence 8. The sentence in the box would not follow sentence 9 (Option D) because sentence 9 is used to end the discussion of how hobbies relieve stress. Option C states the only place where the ideas in the sentence fit into the paragraph. READING COMPREHENSION No Summer 21. (B) The passage is mostly about the strange, cold summer of 1816 and speculation around its cause, which is best stated in Option B. Option A is a detail in the passage about one theory regarding the cause of the weather. Option C is incorrect because the passage is about more than agriculture in New England. Option D is a detail in the passage mentioned only in the first paragraph. 22. (E) The second paragraph states that “farmers prepared to plow and plant” (lines 15–16), they “expected warm temperatures” (lines 16–17), and they were “optimistic” (line 18). This suggests that the farmers kept replanting their crops because they expected the weather to return to normal, which is reflected in Option E. Option F and Option H are incorrect because the cold weather and the snow actually worsened growing conditions. Option G is incorrect because the weather did not improve until the following year. 18. (F) Sentence 16 uses vague and imprecise language that needs to be more specific (“do something,” “get better,” “go to places”). Only Option F provides specific details about the social activities a hobbyist might do (“enroll in a course,” “attend a convention”). The language used in Option E (“learn more about,” “go to events”), Option G (“try to find new information,” “go to places”), and Option H (“want to expand his or her knowledge,” “do an activity”) does not offer more precision than the original sentence. 76 SAMPLE TEST – ENGLISH LANGUAGE ARTS EXPLANATIONS OF CORRECT ANSWERS continued... 23. (C) The winter of 1816–1817 followed the meager harvest of the summer of 1816. With many crops “stunted or destroyed” (lines 26–27), one would expect food shortages the following winter, which is Option C. There is no evidence in the passage that people experienced new weather events (Option A) or warmer temperatures (Option B). Although some famers did replant their crops, there is no evidence in the passage that they struggled to adjust to a different time line for farming (Option D). Form A plausible explanation”—Bessel’s—is described later in the passage. The ideas described in Option C and Option D are not included in the third paragraph. Dickens 28. (G) The issues presented in Option E and Option H are only briefly mentioned or hinted at in the passage. Option F states an important detail about Dickens’s childhood, but it is not a main topic. Option G correctly combines the information in the passage about Dickens’s childhood and the novel David Copperfield. 24. (H) The phrase “the global nature of weather” refers to how conditions in one part of the world can affect weather in another part of the world, which is Option H. Option E is incorrect because line 70 is about the effects of weather conditions around the world, not about making weather predictions. Option F is incorrect because it discusses the lasting impact on specific geographical areas, while the phrase “the global nature of weather” refers to events that affect the entire world. While weather events, like the unusually cold summer in New England in 1816, can be related to natural disasters such as a volcano eruption, there is no support for the idea that natural disasters tend to occur at the same time, which rules out Option G. 29. (A) The author includes the details about Dickens’s experiences as an adult in the fourth paragraph in order to highlight that his time spent in the factory continued to influence him as an adult, as evidenced in lines 43–44 (“As an adult, Dickens always remembered the shame and humiliation”), lines 46–47 (“he could not go near the sites of the factory and boardinghouse”), and lines 49–53 (“Dickens never told his wife and children about his childhood work experience”). This purpose is best stated in Option A. While Dickens was “miserable during the entire four months he spent working at the factory” (lines 26-27) and likely did not want to work in a factory ever again, the author does not include the details in the fourth paragraph to emphasize this idea, ruling out Option B. Option C is incorrect because the detail that Dickens did not share information about his childhood with others is used as a supporting sentence for the idea that childhood experiences affected his adult life. It is also reasonable to infer that Dickens did not want his children to suffer the way that he did, but the details in the fourth paragraph do not emphasize or support this inference, ruling out Option D. 25. (B) The details about the eruption are included in the fourth paragraph. They highlight the severity of the eruption and how it clouded the atmosphere and eventually encircled the world (Option B). The passage does not support the idea that the effects are still present today (Option A) or that other weather events caused the volcano to erupt (Option D). While the details may include information about what happens during an eruption (Option C), that is not why the author includes those details. 30. (H) Although Dickens disliked his job, there is no reason to think he could not perform his duties (Option E). Option F and Option G might be true, but the details presented in the second and third paragraphs primarily support Option H, the idea that Dickens disliked working in the factory and preferred attending school (“forced to quit school,” “relieved to be out of the factory,” “The father, however, now sided with his son, and the boy was sent back to school”). 26. (H) Researchers today believe that Bessel’s theory is the most logical and probable (lines 67−70). His ideas are summarized in lines 55–57, and Option H restates his theory. Option E and Option F were thought to be other possible causes at the time. Option G was an effect of Mount Pinatubo erupting in 1991. 27. (B) The third paragraph describes how nineteenthcentury religious and other leaders tried to account for the cooler weather in 1816. Some leaders thought it was “the end of the world” (line 42), “sunspot activity” (line 44), or a new invention (lines 44−46). This is best stated in Option B. The causes described in the third paragraph were not the most probable cause (Option A), as “the first 31. (B) The correct answer is found in lines 10–13 and in the fifth paragraph. Dickens wrote David Copperfield because he was unable to complete his autobiography and writing the novel helped him deal with difficult childhood memories (Option B). Dickens’s writings as an adult would not have helped pay his family’s 77 SAMPLE TEST – ENGLISH LANGUAGE ARTS EXPLANATIONS OF CORRECT ANSWERS continued... Form A 36. (F) The process of collecting aromas during food preparation is described in the third paragraph. Option E is not supported because the process of capturing aromas has been successful and only certain flavors present difficulties. The idea that most people cannot tell the difference between natural and synthetic flavors (Option G) is not a conclusion that can be made from the collection of aromas during food preparation. Option H is incorrect because aromas are collected during the cooking process to isolate essential chemicals that make up flavor (lines 35–37), not to enhance the natural flavor. Option F is the best answer: the aroma of food as it is being prepared can be captured and distilled to synthesize the food’s flavor (lines 35–41). debts (Option A). While it may be true that Dickens avoided telling his children about his job at the factory (Option C), that is not the reason Dickens wrote David Copperfield. Option D is incorrect because Dickens did not intend to share information about his own experiences. 32. (F) The passage says little about the relationship between Dickens and his mother, only that Dickens felt betrayed when his mother, anxious for the boy’s wages, got his job back for him (lines 33–36). Option F best expresses this information. Option E suggests that they did not have a positive relationship, but there is no evidence in the passage to support this inference. Dickens may have hoped his mother would understand why he wanted to go to school (Option G), but this idea is not clearly expressed in the passage. Dickens’s mother did negotiate with the factory boss on his behalf (Option H), but this action does not describe their relationship. 37. (C) Orange soda is mentioned in lines 52–57 to provide an example of a product that uses a synthetic flavor that some consumers prefer to its natural counterpart (Option C). The idea that consumer preferences for artificial or natural flavors vary could be true, but this is not suggested by the author’s discussion of one flavor (orange soda), ruling out Option A. Option B is incorrect because even though the passage states that natural flavors may be more expensive than artificial flavors, the author does not use the details about orange soda to make this point. The author states that some natural flavors may become scarce in the future (lines 59–61), but this is not exemplified by the discussion of orange soda in lines 52–57, ruling out Option D. 33. (D) The passage states, “The father, however, now sided with his son, and the boy was sent back to school” (lines 36–38). In other words, Dickens was able to return to school because his father supported the idea (Option D). The discussion between his mother and his boss (Option A) led to Dickens getting his factory job back. The argument between his father and his boss (Option B) led to Dickens’s dismissal from his job, not his return to school. Getting fired from the factory (Option C) occurred before Dickens returned to school but was not the direct reason for it. 38. (F) The author describes the role of the sense of smell to highlight that the aroma of a flavor, in addition to its taste, influences how a person experiences a flavor, as evidenced in lines 10–11 (“The sense of smell has a larger role in tasting flavors than most people realize”). This is best stated in Option F. Option E reflects the idea in lines 8–10, but the idea that it is easier for people to smell an aroma than to taste its flavor is not why the author includes the description about how smell impacts taste. Option G and Option H present inaccurate ideas regarding synthetic flavors and fail to explain why the author describes the role of the sense of smell at the beginning of the passage. Flavors 34. (G) Option E and Option F are too specific: the passage mentions some scientific aspects of taste and smell, but it concentrates on the development of flavors. Option H is mentioned in only the last paragraph. Option G is a good summary of the passage. It incorporates the main topics—the scientific analysis of flavors and how flavors are created. 35. (C) The third paragraph describes a technique for separating a food into its basic chemical constituents. Option C best summarizes the goal of this research. Option A and Option D are not supported by the passage as goals of the research. Option B is incorrect because the goal of the research is to capture and reproduce the flavor, not to develop food. 39. (D) The author describes the uses of synthetic flavors in items such as “mouthwashes, toothpastes, beverages” (lines 26–27) to demonstrate that synthetic flavors are found in many everyday household products. This is best stated in Option D. Option A may seem like an attractive option because the list of everyday items with synthetic flavors could give the impression that creating synthetic flavors is easy; however, while some synthetic flavors have 78 SAMPLE TEST – ENGLISH LANGUAGE ARTS EXPLANATIONS OF CORRECT ANSWERS continued... been successfully created (lines 17–20), efforts to duplicate other flavors have been unsuccessful (lines 62–71), ruling out Option A. The idea that the same synthetic flavor is used in many items (Option B) and the idea that synthetic flavors are healthier than natural flavors (Option C) cannot be concluded from the list of common products that use synthetic flavors in lines 26–28. Form A was abandoned and why,” has not been solved (lines 77–80). The presence of ivory and gold (Option F) and the reason that Europeans did not discover Great Zimbabwe until the 1870s (Option G) are not presented as mysteries. 43. (B) The Shona people are discussed in the fifth paragraph. Ancient Shona people lived in the African interior, not on the coast, and the passage does not explain where Shona people live in the present, which eliminates Option A. Option C and Option D confuse the histories of the Shona people and ancient Middle Eastern people. Option B is the best answer; lines 64–67 state that Great Zimbabwe was most likely built by ancestors of the present-day Shona people. Great Zimbabwe 40. (G) Only Option C represents the central idea that is developed, supported, and explained throughout the passage. The idea that there was much speculation about Great Zimbabwe is explained in lines 9 and 10 as well as in the second paragraph, and the details about how modern archaeologists determined its origins are explained in the fifth paragraph. Option E is incorrect because it does not encompass the facts revealed about Great Zimbabwe in the fifth paragraph. Option F is incorrect because it mainly focuses on the idea that archaeologists are still interested in the mysteries of Great Zimbabwe, which is mentioned only in the sixth paragraph. Option H is incorrect because the fact that early excavations of Great Zimbabwe caused the destruction of valuable evidence is a detail from lines 50–58, and the option does not fully explain the central idea developed in the passage. 44. (H) David Randall-MacIver and Gertrude CatonThompson’s conclusions were significant because their excavation of the ruins revealed that Great Zimbabwe was most likely built by the Shona people (lines 64–67) and discredited the long-standing idea that the structure was Middle Eastern in origin. This is stated in Option H. Option E is incorrect because Randall-MacIver and Caton-Thompson’s determined that the city was most likely built in the fourteenth or fifteenth century (line 66), which was later than earlier explorers had assumed. Lines 69–75 indicate that the Shona society was robust (Option G), and lines 77–80 pose the question of why the great city was abandoned (Option F), but these ideas are not the main reasons Randall-MacIver and CatonThompson’s conclusions were significant. 41. (B) Lines 45–47 explain that Mauch “jumped to the conclusion that Great Zimbabwe had been built by the Queen of Sheba.” This affected later investigations of the ruins because archaeologists worked under the assumption that Mauch’s conclusions were accurate, and the archaeologists discarded evidence that may have suggested otherwise, as detailed in lines 50–56. This is best stated in Option B. Archaeologists were interested in the area (Option A), but this was not the main effect of Mauch’s conclusions, as described in the passage. People searched for Great Zimbabwe because they already believed the stories told by Arab traders and historians like de Barro were true (lines 23–26), not because of Mauch’s conclusions, which rules out Option C. Although the city was considered impressive (lines 29–40; lines 69–75), Mauch’s conclusions did not influence whether people believed an ancient culture could have built it, ruling out Option D. 45. (A) Option A is correct; the Portuguese searched for “King Solomon’s gold,” which they associated with Great Zimbabwe but they never found the city (lines 23–26). Option B and Option D may be true based on the details in the second paragraph, but they are not the best descriptions of the Portuguese explorers’ overall relationship with Great Zimbabwe. Even though Portuguese explorers had little information about the precise location of Great Zimbabwe (lines 13–16), the passage does not support the idea that the explorers knew they would not find the stone city (Option C). Bats 46. (E) Option E is correct because it states the main idea of the passage: bats provide benefits for the environment and need to be protected. These benefits are explained and supported in the second and third paragraphs, and the idea that they need to be protected is supported in the fourth and fifth paragraphs. Option 42. (H) The fifth paragraph states that carbon-14 dating proved Randall-MacIver and Caton-Thompson’s conclusions that Great Zimbabwe was built by ancestors of the Shona people during the fourteenth or fifteenth century (lines 64–69). Option E, “when the settlement 79 SAMPLE TEST – ENGLISH LANGUAGE ARTS EXPLANATIONS OF CORRECT ANSWERS continued... F is incorrect because it focuses on the idea that people consider bats pests and does not explain how bats are helpful. The idea that bats help prevent the spread of disease (Option G) and pollinate rain forest plants (Option H) are details about some of the benefits bats provide, not the central idea of the passage. Form A 50. (G) The author describes the bat’s role as a keystone species in order to emphasize the drastic effects that a sudden change in the bat population would have on the pollination of plants and trees (lines 36–37), the ability of certain animals to find shelter (lines 37–38), and the levels of oxygen in the atmosphere (lines 40–43). This is best stated in Option G, which encompasses the idea that if bats could not perform these natural activities, the rain forest would be threatened. The ideas in Option E, Option F, and Option H are explained in the third paragraph as natural activities that bats support, but these are individual details about what bats do. The statements do not explain the significance of the role of bats in the survival of an entire rain forest ecosystem. 47. (D) The far-reaching impact of a keystone species is described in the third paragraph. The flying fox, a keystone species in the rain forest, pollinates plants and distributes seeds and thus helps provide food and shelter for many other plants and animals in its ecosystem. Rain forests in turn help maintain a balanced global atmosphere for living creatures everywhere. Option D best states that bats are important in maintaining a stable ecosystem because of the effects listed in lines 36–43. A keystone species can be threatened with extinction (Option A), but that does not explain the function of a keystone species. Option B and Option C are too limited in scope to represent the function of a keystone species. 51. (B) The phrase “ugliness is only skin deep” (line 69) is intended to mean that an unattractive outward appearance does not necessarily indicate inward ugliness. The intended meaning—that the ugly outward appearance of bats does not mean that they are bad—is best stated in Option B. While the passage describes different species of bats (“brown bat” in lines 5 and 8 and “flying foxes” in the second paragraph), the text focuses on their roles in the environment, not on a comparison of their appearance, which rules out Option A. Option C is incorrect; the passage states people try to remove bats because they ruin fruit trees and crops (lines 49–50), not because of their appearance. As mentioned in the fifth paragraph, conservation groups and government agencies are trying to overcome people’s negative perception of bats by educating people about the benefits bats provide; however, Option D is incorrect because the passage does not state or imply that the physical appearance of bats influences efforts to conserve them. 48. (G) The author discusses bat conservation at the end of the passage. The author acknowledges that many people think bats are a problem but then stresses that bats should be carefully managed and protected, which is best stated in Option G. Option E is incorrect because the author does not advocate for bats eating crops and cultivated trees. Option F is incorrect because, while bats do support the growth and survival of many species, the author never suggests relocating them to areas with struggling ecosystems as a conservation method. Option H is incorrect because the author understands that farmers and orchard owners need their crops and trees to survive in order to earn a livelihood (lines 62–65). Samizdat 49. (A) The iroko tree is mentioned in lines 20–23 as a valuable tree that depends entirely on flying foxes for pollination (Option A), and so without bats this plant would not exist. While genetic diversity can improve a plant’s ability to survive, it is not necessary for survival, ruling out Option B. Lines 46–50 explain why bats sometimes eat cultivated fruit, but the result of this is that crops are ruined, ruling out Option C. While bats do eat mosquitoes (lines 9 and 10), this detail best supports the idea that bats help limit the spread of disease, which rules out Option D. 52. (G) Option E mentions two important samizdat writers from the fourth paragraph but does not explain samizdat or the authors’ relationship to it. Option F refers to all poetry published in the Soviet Union, not just samizdat poetry, so it is too broad. Option H is mentioned only in the first paragraph. Option G is a good description of the topic of the passage, describing Soviet censorship and the samizdat response. 53. (B) The earliest phase of samizdat is described in lines 33 and 34: “At first, samizdat focused mainly on literature, such as poetry and novels.” Only Option B, “a short story,” fits in this category. Option A, Option C, and Option D are not representative of the earliest phase of samizdat. 80 SAMPLE TEST – ENGLISH LANGUAGE ARTS EXPLANATIONS OF CORRECT ANSWERS continued... 54. (E) The phrase “a knock at his door in the middle of the night” refers to the secret police. Pasternak, like other samizdat writers, feared being caught by authorities and accused of writing and distributing samizdat (lines 15–18). Option E is correct. The passage does not indicate that Pasternak would be concerned about “a representative from a major publisher” (Option F), “a participant in the samizdat network” (Option G), or “people from other countries” (Option H) knocking on his door at night. 55. (D) Option D is the best answer because storing and circulating texts via computers marked a significant change from hand-copying or typing paper copies of samizdat texts (lines 64–68). Option A, Option B, and Option C represent events that occurred during the peak of samizdat circulation, but those events did not directly lead to significant changes in the samizdat distribution process. 56. (F) Lines 1–3 indicate that people could be punished for writing about certain topics. Lines 26–32 explain the steps samizdat authors and distributors used to ensure that the network of authors was protected, including having authors leave their work unsigned or using fake names. This is stated in Option F. Samizdat works, like Pasternak’s Doctor Zhivago, were smuggled out of the country (lines 46–48), so using no names or fake names would not relate to this practice, ruling out Option E. Samizdat writers would be breaking censorship law regardless of whether they included their real names, which rules out Option G. While making copies of a work was part of the samizdat distribution process, the act of leaving work unsigned or writing under a false name would not differentiate between original and copied works, which rules out Option H. 57. (A) The answer is found in lines 68–73. The correct answer, Option A, makes the connection between the abolishment of censorship and subsequent freedom of the press, which eliminated the need for samizdat. The fifth paragraph explains that there was “a publishing boom” (lines 70–71) after censorship laws were abolished but does not suggest that samizdat networks ended because they became regular publishing companies, which rules out Option B. Option C is incorrect because the use of computers helped samizdat networks spread material (lines 65–68) while censorship laws were still in place. Option D is incorrect because going deeper underground would not be necessary after censorship was abolished. 81 Form A SAMPLE TEST – MATHEMATICS EXPLANATIONS OF CORRECT ANSWERS 58. (162) First, find the measure of angle PQR. The measure of angle PQR is equal to the measure of angle PSR. 62. (63) If x is the smaller consecutive integer, then x + 1 is the larger consecutive integer. Use their sum (−15) to find x: m∠PSR = 180 − 72 = 108. x + ( x + 1) = −15 2x + 1 = −15 2x = −16 x = −8 So, the measure of angle PQR is also 108. 108 + 90 + x = 360 198 + x = 360 x = 162 59. (99) Let x be the number of oak trees when 264 pine trees are planted. Set up a proportion and solve for x: x = 3 264 8 The two consecutive integers are −8 and −7. One is added to the smaller integer: −8 + 1 = −7, and 2 is subtracted from the larger integer: −7 − 2 = −9. Find the product: −7 × −9 = 63. 63. (B) 2k = m + 3 so k = m +3 . 2 Substitute each value of m to find the values of k: 8x = 762 x = 99 k = 5 +3 = 8 = 4 2 2 60. (– 4) 4w = 2w − 8 2w = −8 w = −4 k = 7+3 = 10 = 5 2 2 k = 9 +3 = 12 = 6 2 2 61. (45) Let x = number of students with only cats as pets. Let y = number of students with only dogs as pets. The set k is { 4, 5, 6}. 64. (E) First, convert 500 milliliters to liters by dividing by 1,000: 500 ÷ 1,000 = 0.500 Calculate x and y using the given information: There are 20 students who have cats, and of those 20 students, 3 have both cats and dogs. Thus, x = 20 − 3 = 17. There are 23 students who have dogs, and of those 23 students, 3 have both cats and dogs. Thus y = 23 − 3 = 20. AA FORM Now, multiply by 24 to find the solution: 0.500 × 24 = 12 L 65. (A) The sum of Adrianna’s course grades equals 4 times the average (mean) of her grades: 90 × 4 = 360. Roberto has the same sum (360) as Adrianna. Find the mean of his course grades: To find the total number of students surveyed, add the number of students who only have cats (x), the number of students who only have dogs (y), the number of students who have both (3), and the number of students who have neither (5): 3 + 5 + x + y = 8 + 17 + 20 = 45 360 ÷ 5 = 72 82 SAMPLE TEST – MATHEMATICS EXPLANATIONS OF CORRECT ANSWERS continued... 66. (H) Set up some equations. Jenny (J) has twice as many marbles as Keiko (K): J = 2K Jenny gives Keiko 5 marbles, so now they each have: J − 5 and K + 5 marbles. Jenny still has 10 more than Keiko: J − 5 = ( K + 5) + 10 Form 69. (B) Line segment RS is the altitude, or height, of triangle QRP. The length of QP is 8 cm. Use that information to find the area of triangle QRP: A = 1 bh = 1 (8)(6) = 24 sq cm. 2 2 To find how many marbles Jenny had to start with, solve J = 2K for K and substitute that into the second equation: In equation J = 2K , solve for K: K = J . 2 Substitute J in for K. 2 There are 4 congruent triangles in the pyramid, so the surface area is 4 × 24 = 96 sq cm. 70. (F) Let 2x = the width and 3x = the length. Draw the rectangle to help visualize. 3x J − 5 = ( K + 5) + 10 ( ) J − 5 = J + 5 + 10 2 J J −5 = + 15 2 J = J + 20 2 J = 20 2 J = 40 marbles 2x 2x 3x Since 2 times width + 2 times length = perimeter, we get 2( 2x) + 2(3x) = 510 4 x + 6x = 510 10x = 510 x = 51 2x = 102 cm and 3x = 153 cm 67. (A) Let x be the number of inches representing 1 foot. Set up a proportion and solve for x: x = 0.125 125 1 x = 0.001 in. 71. (D) Multiply each term by 2 to eliminate the fraction, and isolate x: 68. (G) First, add the percentage of cars containing 3 people, 4 people, and 5 or more people: ( ) −4( 2) < x ( 2) < 2( 2) 2 −8 < x < 4 15% + 7% + 3% = 25% A Therefore, x must be between −8 and 4. Thus, 25% of the cars contained at least 3 people, so use that to calculate the number of cars: 420 × 0.25 = 105 cars. 83 SAMPLE TEST – MATHEMATICS EXPLANATIONS OF CORRECT ANSWERS continued... 72. (F) Use proportions to make the conversions: Form 74. (H) Since both ratios have y in common, solve for x and z in terms of y in both equations. Lorgs to dollars: Using x: y = 1:4, solve for x in terms of y. 140 = 7 x 1 x = 1 y 4 7x = 140 x= 1 y 4 x = $20 Dalts to dollars: 16 = 0.5 x 1 Using the ratio y: z = 4:5, solve for z in terms of y: y = 4 z 5 0.5x = 16 z= 5 y 4 x = $32 Total dollars = 20 + 32 = $52 73. (B) Let x be the total number of colored pencils in the box. Set up a proportion to find x: The question states x + y + z = 50. Substitute from the two equations above and solve for y. 1 y + y + 5 y = 50 4 4 2 = 6 7 x A 2x = 42 10 y = 50 4 x = 21 10 y = 200 If there are 6 red pencils, then the number of pencils that are not read is 21 − 6 = 15. y = 20 75. (B) The shaded region is a right triangle. Each leg is 1 unit in length. So the area is = A 1= bh 2 1 (1)(1) = 1 or 0.5 sq unit 2 2 76. (F) Create a table with the information provided in the problem and use subtraction to fill in the rest of the table: Female Commutes to work 21% Does not 24% commute to (45 – 21) work TOTAL 84 45% Male TOTAL 39% (60 – 21) 60% 16% (40 – 24) 40% (100 – 60) 100% 16% of the population is male and does not commute to work. SAMPLE TEST – MATHEMATICS EXPLANATIONS OF CORRECT ANSWERS continued... 77. (A) Let x be the price per pound for the meat. Set up an equation to show what Mrs. Cranston spent: Form A 81. (B) According to the chart, 22% of people walk to work and 4% ride a bicycle. Subtract to find the percentage of how many more people walk than bicycle: 5(0.90) + 8x = 26.90 4.50 + 8x = 26.90 8x = 22.40 x = 2.80 22% − 4% = 18% The price per pound for the meat is $2.80. To find the exact number of people, multiply 18% (0.18) by the number of people working in Center City (15,000): 15,000 × 0.18 = 2,700 78. (E) The probability that both cards are not blue is the same as the probability that both cards 82. (F) To find the smallest factor of 91, list the factors: 1, 7, 13, and 91. are red. There are 4 red cards out of the 10, The smallest factor (other than 1) is 7. so the probability of the first card being red is 4 . Now there are 9 cards left, and 3 of those 10 are red, so the probability of the second card being red is 3 . Multiply the two probabilities 9 to find the probability that both cards are red Of the options listed (30, 35, 39, and 44), only 35 is a multiple of 7. 83. (D) Let x be the remaining side of the actual banner. Set up a proportion: x = 36 12 16 (not blue): x = 48 ft 4 × 3 = 12 = 2 10 9 90 15 84. (F) Let x be the number of second-, third-, and fourth-year students. Then the total number of students in the college is 663 + x. Set up a proportion and solve for x: 79. (D) 1 sind = 4 lorgs, so 1 sind > 1 lorg. 2 harps = 5 sinds, so 1 harp > 1 sind. 15 = 663+ x 1 179 663 + x = 179(15) 1 plunk = 3 harps, so 1 plunk > 1 harp, meaning that 1 plunk > 1 sind and 1 lorg. 2 plunks = 5 dalts, so 1 plunk > 1 dalt. 663 + x = 2,685 Therefore, the plunk is the most valuable. x = 2,022 85. (D) 2 1 + 3 3 + 4 2 + 5 1 5 10 5 2 80. (G) For each row, multiply the number of students by the score. Then add those together and divide by the total number of students to find the mean (average) of the 10 students. 85( 4 )+75( 4 )+65( 2) = 340 +300 +130 10 10 Convert all the fractions to a common denominator (10): 2 2 +3 3 +4 4 +5 5 10 10 10 10 + 2 3 + 4 +5 = ( 2 + 3 + 4 + 5) + 10 = 14 + 1 4 = 15 2 10 5 ( = 770 = 77 10 85 ) SAMPLE TEST – MATHEMATICS EXPLANATIONS OF CORRECT ANSWERS continued... 86. (F) Divide the rate by the number of seconds in an hour. (Since there are 60 minutes in an hour and 60 seconds in a minute, multiply 60 × 60 = 3,600 seconds in an hour): 55 miles per second 3,600 l+ g = l + l + 4 = 2l + 4 = l + 2 2 2 2 Multiply by the number of feet in a mile (5,280): 55 ⋅ 5,280 feet per second 3,600 90. (H) Three years is 36 months (12 × 3). Set up an expression to find the total amount Johan paid: 1,000 + 300(36) = $11,800 91. (B) Create a list of the possible pairs. Let the cookies be named A, B, C, D, E, and F. Two years from now, Tien’s age will be T + 2, and Jordan’s age will be J + 2. Set up an equation about the relationship between Tien’s age and Jordan’s age in two years: AB, AC, AD, AE, AF BC, BD, BE, BF CD, CE, CF DE, DF EF There are a total of 15 possible pairs of cookies that Aiden can choose. T + 2 = 1 ( J + 2) 3 Solve the above equation for T: T = 1 ( J + 2) − 2 3 92. (G) Set up proportions to figure out how many slides Deion and Kyra can create in 1 hour: Now set the two equations equal to each other and solve for J: 1 J = 1 (J + 2) − 2 4 3 20x = 300 − 1 J =−4 12 3 ( Deion 5 = x 20 60 1J = 1J− 4 4 3 3 J = − 4 − 12 3 1 x = 15 ) Deion can create 15 slides in 1 hour. Kyra 3 = x 10 60 J = 16 88. (E) List the factors of 48: 10x = 180 x = 18 A Substitute l + 4 for g and simplify: T = 1J 4 Form 89. (C) The first integer is l, so the second is l + 1, the third is l + 2, then l + 3, and finally l + 4. Since g is the fifth and greatest of the integers, g = l + 4. 87. (D) First, set up an equation to express Tien’s age (T) and Jordan’s age (J) today: 1 and 48, 2 and 24, 3 and 16, 4 and 12, 6 and 8 There are no factors greater than 24 and less than 48. 86 Kyra can create 18 slides in 1 hour. Add Deion and Kyra to figure out how many slides they can create together in 1 hour: 15 + 18 = 33. SAMPLE TEST – MATHEMATICS EXPLANATIONS OF CORRECT ANSWERS continued... Form A 97. (B) The question asks for integers from 12 to 30 that are not divisible by 2 or 3. 93. (C) Since LN = 1 , point N is located at 8 4 5 + 1 = 4 7 . So M must be between 16 8 16 point L, 4 5 , and point N, 4 7 . Point L can 16 16 also be written as 4.3125, and point N can be written as 4.4375. The only option given that The set of consecutive integers is {12, 13, 14, 15, 16, 17, 18, 19, 20, 21, 22, 23, 24, 25, 26, 27, 28, 29, 30}. Since all even numbers are divisible by 2, eliminate all even numbers, leaving the odd numbers in the set: {13, 15, 17, 19, 21, 23, 25, 27, 29}. Eliminate those integers that are multiples of 3 (15, 21, and 27). The remaining integers are: {13, 17, 19, 23, 25, 29}. Therefore, there are 6 numbers in the set that are multiples of neither 2 nor 3. lies between those two points is 4.35. 94. (G) The length of the stick must be the greatest common factor of 72 and 30. The factors of 30 are 1, 2, 3, 5, 6, 10, 15, and 30. Of those, only 1, 2, 3, and 6 are also factors of 72. The greatest of these is 6. 98. (G) Take each city’s number of schools and multiply by the number of students. It is not necessary to calculate all 5 of these. Cities M and N have the same number of students, so just calculate the number of students in City M because it has more schools than City N. The same goes for Q and R — only Q needs to be calculated because it has more schools than R. 95. (B) Ryan has 130 pages left to read (150 − 20). He read 20 pages in 30 minutes, which means he read at a rate of 40 pages per 1 hour. To find out how much longer it will take him to finish the assignment, divide the total number of pages remaining (130) by the number of pages he is able to read per hour (40): 130 = 3 1 40 4 M = 8 × 500 = 4,000 P = 9 × 400 = 3,600 96. (G) It is easier to rewrite M as M ÷ N since N they are both fractions. Q = 6 × 700 = 4,200 y M÷N = w ÷ = w ⋅ z = wz x z x y xy City Q has the greatest number of students. 99. (C) The total number of candies in the box is 5 + 3 + 2 = 10. The number of candies that are not banana is 5 + 2 = 7. The probability of the first candy not being banana is 7 . Now, out of 9 candies, there 10 are 6 candies left that are not banana. The probability of the second candy not being banana is. 6 . Multiply these two 9 probabilities to get the solution: 7 × 6 = 42 = 7 10 9 90 15 87 SAMPLE TEST – MATHEMATICS EXPLANATIONS OF CORRECT ANSWERS continued... Form A 100. (H) Solve the equation for z: 105. (D) There are 6 digits in the repeating decimal (769230), so 7 would be the first, seventh, thirteenth digit and so on. To find the 391st digit, divide 391 by 6. w = y x z wz = xy z= 391 ÷ 6 = 65 R1 xy w 101. (C) Convert the ratios into fractions of WZ. Use the sum of the ratios for the denominator. 106. (E) One revolution is equal to the circumference of the tire: WX:XY:YZ = 4:2:3 WX = 4 = 4 4 + 2 +3 9 ( ) C = 2πr = 2(1) 22 = 44 ft 7 7 2 XY = = 2 4 + 2 +3 9 The part of WZ that is WY is the sum of those fractions: Find the length of WZ: WZ = 8 − (−10) = 18 The value of WY is 2 (18) = 12. 3 107. (D) 100( 2 + 0.1)2 − 100 = 100( 2.1 2 ) − 100 = 100( 4.41) − 100 = 441 − 100 = 341 102. (G) Find 1% of 0.02: 0.02 × 1 = 0.0002 100 The car travels at 4,400 ft per minute. To calculate the number of revolutions, divide the speed by the circumference: 4,400 ÷ 44 = 4,400 × 7 = 700 revolutions. 7 44 WY = 4 + 2 = 6 = 2 9 9 9 3 Since the remainder is 1, that means the 391st digit is the same as the 1st digit, which is 7. 108. (G) The total number of handballs in the container is 4 + 5 + 8 + 9 + 11 = 37. The greatest allowable thickness would be 0.02 + 0.0002 = 0.0202 inch. Since there are 8 yellow handballs, the probability of selecting a yellow handball is 8 . 37 103. (D) First, calculate the highest score for each section by adding the lowest score to the range: Section I: 65 + 28 = 93 Section II: 62 + 25 = 87 109. (A) Each chair costs Leon $150 to make, and he sells the chair for $275. His profit is found by subtracting the cost from the price: Section III: 67 + 22 = 89 $275 − $150 = $125 per chair The overall highest score is 93, and the overall lowest score is 62. Thus the overall range is 93 − 62 = 31. If Leon makes and sells 25 chairs in a week, his initial profit is 25 × $125 = $3,125. However, Leon has additional fixed expenses of $1,250 per week, so this cost must also be subtracted to arrive at the profit. His final profit is $3,125 − $1,250 = $1,875. 104. (F) Since 3n is even, then 3 n + 1 must be odd. Thus 3 n + 3 and 3 n + 5 are also odd. So there are a total of 3 numbers in this range that are odd. 88 SAMPLE TEST – MATHEMATICS EXPLANATIONS OF CORRECT ANSWERS continued... 110. (H) Convert 4 ft 7 in. to inches. 4(12) + 7 = 55 inches Multiply that by the conversion 254 cm = 1 in. 55 × 2.54 = 139.70 cm 111. (C) First, use JK = 3 1 to find the location of J: 2 3 −J = 31 8 2 3 1 J= − 3 = −3 1 8 2 8 Now, use JM = 9 3 to find the location of M: 4 ( ) A Number of Servings of Fruits and Vegetables Number of Students Number of Servings × Number of Students 0 5 0 1 7 7 2 3 6 3 4 12 4 0 0 5 1 5 Total: 30 M − −3 1 = 9 3 8 4 1 3 M+3 = 9 8 4 3 M = 9 −31 = 6 5 4 8 8 Form 113. (A) First, determine the total number of servings of fruits and vegetables that the students ate by multiplying the number of servings by the number of students in each row of the table. Then add that column to get the total number of servings: Since 12 in. = 1 ft : Calculate the mean by dividing the total number of servings of fruits and vegetables by the total number of students: 30 = 1 1 20 2 Finally, use LM = 1 1 to find the location of 8 L: 114. (G) The ratio is 4:3:2:1, so the total parts is 10. 65 −L =11 8 8 Since there are two parts resin, the fraction of L = 6 5 −1 1 = 5 4 = 5 1 8 8 8 2 resin is 2 = 1 . 10 5 112. (G) 4 x − 3 y = 12 So the amount of resin in 30 lb of paste (for 1 billboard) is 1 × 30 = 6 lb. For 4 billboards, 5 4 x = 3 y + 12 that would be 6 × 4 = 24 lb. x = 3 y + 12 4 4 x = 3 y+3 4 89 SAMPLE TEST – MATHEMATICS EXPLANATIONS OF CORRECT ANSWERS continued... Form Answer Key for Sample Form A 1. D 14. E 27. B 40. G 53. B 66. H 79. D 92. G 105. D 2. G 15. A 28. G 41. B 54. E 67. A 80. G 93. C 106. E 3. C 16. G 29. A 42. H 55. D 68. G 81. B 94. G 107. D 4. E 17. C 30. H 43. B 56. F 69. B 82. F 95. B 108. G 5. C 18. F 31. B 44. H 57. A 70. F 83. D 96. G 109. A 6. E 19. A 32. F 45. A 58. 162 71. D 84. F 97. B 110. H 7. A 20. H 33. D 46. E 59. 99 72. F 85. D 98. G 111. C 8. F 21. B 34. G 47. D 60. –4 73. B 86. F 99. C 112. G 9. D 22. E 35. C 48. G 61. 45 74. H 87. D 100. H 113. A 10. G 23. C 36. F 49. A 62. 63 75. B 88. E 101. C 114. G 11. B 24. H 37. C 50. G 63. B 76. F 89. C 102. G 12. G 25. B 38. F 51. B 64. E 77. A 90. H 103. D 13. B 26. H 39. D 52. G 65. A 78. E 91. B 104. F 90 A SIDE 2 NEW YORK CITY PUBLIC SCHOOLS 2017 SPECIALIZED HIGH SCHOOLS ADMISSIONS TEST GRADE 8 Test Booklet Letter Student’s First Name (please print) A \ E 2\ A 3\ E 4\ A 5\ E 6\ A 7\ E 8\ A 9\ E 10 \ A 11 \ E 12 \ A 13 \ E 14 \ A 15 \ 1 B \ F \ B \ F \ B \ F \ B \ F \ B \ F \ B \ F \ B \ F \ B \ C \ G \ C \ G \ C \ G \ C \ G \ C \ G \ C \ G \ C \ G \ C \ Student’s Last Name (please print) E \ A 17 \ E 18 \ A 19 \ E 20 \ A 21 \ E 22 \ A 23 \ E 24 \ A 25 \ E 26 \ A 27 \ E 28 \ A 29 \ E 30 \ D \ H \ D \ H \ D \ H \ D \ H \ D \ H \ D \ H \ D \ H \ D \ 16 PART 1 ENGLISH LANGUAGE ARTS F \ B \ F \ B \ F \ B \ F \ B \ F \ B \ F \ B \ F \ B \ F \ H \ D \ H \ D \ H \ D \ H \ D \ H \ D \ H \ D \ H \ D \ H \ G \ C \ G \ C \ G \ C \ G \ C \ G \ C \ G \ C \ G \ C \ G \ A \ E 32 \ A 33 \ E 34 \ A 35 \ E 36 \ A 37 \ E 38 \ A 39 \ E 40 \ A 41 \ E 42 \ A 43 \ E 44 \ A 45 \ 31 PART 2 58 59 – \\\\ \ 0 \ 0 \ 0 \ 0 \ 1 \ 1 \ 1 \ 1 \ 2 \ 2 \ 2 \ 2 \ 3 \ 3 \ 3 \ 3 \ 4 \ 4 \ 4 \ 4 \ 5 5 5 5 \\\\ 6 \ 6 \ 6 \ 6 \ 7 \ 7 \ 7 \ 7 \ 8 \ 8 \ 8 \ 8 \ 9 \ 9 \ 9 \ 9 \ • A \ E \ A \ E \ A 67 \ E 68 \ A 69 \ E 70 \ A 71 \ E 72 \ A 73 \ E 74 \ A 75 \ 63 64 65 66 B \ F \ B \ F \ B \ F \ B \ F \ B \ F \ B \ F \ B \ • • C \ G \ C \ G \ C \ G \ C \ G \ C \ G \ C \ G \ C \ • D \ H \ D \ H \ D \ H \ D \ H \ D \ H \ D \ H \ D \ • E \ A \ E \ A \ E \ A \ E 82 \ A 83 \ E 84 \ A 85 \ E 86 \ A 87 \ E 88 \ 76 77 78 79 80 81 • • G \ C \ G \ C \ G \ C \ G \ C \ G \ C \ G \ C \ G \ C \ G \ C \ G \ C \ G \ C \ G \ C \ G \ C \ G \ C \ G \ C \ D \ H \ D \ H \ D \ H \ D \ H \ D \ H \ D \ H \ D \ H \ D \ E \ A 47 \ E 48 \ A 49 \ E 50 \ A 51 \ E 52 \ A 53 \ E 54 \ A 55 \ E 56 \ A 57 \ 46 61 F \ B \ F \ B \ F \ B \ F \ B \ F \ B \ F \ B \ G \ C \ G \ C \ G \ C \ G \ C \ G \ C \ G \ C \ H \ D \ H \ D \ H \ D \ H \ D \ H \ D \ H \ D \ • H \ D \ H \ D \ H \ D \ H \ D \ H \ D \ H \ D \ H \ • • A \ E \ A \ E \ A \ E \ A 95 \ E 96 \ A 97 \ E 98 \ A 99 \ E 100 \ A 101 \ 89 90 91 92 93 94 62 – \\\\ \ 0 \ 0 \ 0 \ 0 \ 1 \ 1 \ 1 \ 1 \ 2 \ 2 \ 2 \ 2 \ 3 \ 3 \ 3 \ 3 \ 4 \ 4 \ 4 \ 4 \ 5 5 5 5 \\\\ 6 \ 6 \ 6 \ 6 \ 7 \ 7 \ 7 \ 7 \ 8 \ 8 \ 8 \ 8 \ 9 \ 9 \ 9 \ 9 \ – \\\\ \ 0 \ 0 \ 0 \ 0 \ 1 \ 1 \ 1 \ 1 \ 2 \ 2 \ 2 \ 2 \ 3 \ 3 \ 3 \ 3 \ 4 \ 4 \ 4 \ 4 \ 5 5 5 5 \\\\ 6 \ 6 \ 6 \ 6 \ 7 \ 7 \ 7 \ 7 \ 8 \ 8 \ 8 \ 8 \ 9 \ 9 \ 9 \ 9 \ • F \ B \ F \ B \ F \ B \ F \ B \ F \ B \ F \ B \ F \ B \ F \ B \ F \ B \ F \ B \ F \ B \ F \ B \ F \ B \ F \ B \ MATHEMATICS 60 – \\\\ \ 0 \ 0 \ 0 \ 0 \ 1 \ 1 \ 1 \ 1 \ 2 \ 2 \ 2 \ 2 \ 3 \ 3 \ 3 \ 3 \ 4 \ 4 \ 4 \ 4 \ 5 5 5 5 \\\\ 6 \ 6 \ 6 \ 6 \ 7 \ 7 \ 7 \ 7 \ 8 \ 8 \ 8 \ 8 \ 9 \ 9 \ 9 \ 9 \ • Test Booklet Number • B \ F \ B \ F \ B \ F \ B \ F \ B \ F \ B \ F \ B \ C \ G \ C \ G \ C \ G \ C \ G \ C \ G \ C \ G \ C \ • • D \ H \ D \ H \ D \ H \ D \ H \ D \ H \ D \ H \ D \ Copyright © 2017 NCS Pearson, Inc. All Rights Reserved. Reprinted by Permission—Restricted Edition—Not for Distribution 91 – \\\\ \ 0 \ 0 \ 0 \ 0 \ 1 \ 1 \ 1 \ 1 \ 2 \ 2 \ 2 \ 2 \ 3 \ 3 \ 3 \ 3 \ 4 \ 4 \ 4 \ 4 \ 5 5 5 5 \\\\ 6 \ 6 \ 6 \ 6 \ 7 \ 7 \ 7 \ 7 \ 8 \ 8 \ 8 \ 8 \ 9 \ 9 \ 9 \ 9 \ • • E \ A \ E \ A \ E \ A \ E 108 \ A 109 \ E 110 \ A 111 \ E 112 \ A 113 \ E 114 \ 102 103 104 105 106 107 F \ B \ F \ B \ F \ B \ F \ B \ F \ B \ F \ B \ F \ • • G \ C \ G \ C \ G \ C \ G \ C \ G \ C \ G \ C \ G \ • H \ D \ H \ D \ H \ D \ H \ D \ H \ D \ H \ D \ H \ T T T T T T T T T T T T T T T T T T T T T T T T T T T T T T T T T T T T T T T T T T T T T T T T T T T T T T T T T T T T T T T SAMPLE TEST, FORM B PART 1 — ENGLISH LANGUAGE ARTS Suggested Time — 90 Minutes 57 QUESTIONS REVISING/EDITING QUESTIONS 1–20 IMPORTANT NOTE The Revising/Editing section (Questions 1-20) is in two parts: Part A and Part B. REVISING/EDITING Part A DIRECTIONS: Read and answer each of the following questions. You will be asked to recognize and correct errors in sentences or short paragraphs. Mark the best answer for each question. 1. Read this paragraph. (1) In September 2016, the National Museum of African American History and Culture opened, as part of the Smithsonian Institution, the museum is already the Smithsonian’s third most popular site. (2) Experts say that they expect this newest Smithsonian facility to welcome nearly 4 million visitors a year. (3) The museum features more than 30,000 objects, including Muhammad Ali’s boxing gloves and a dress sewn by Rosa Parks. (4) A commemorative copy of the Emancipation Proclamation, which was written in 1863 during the presidency of Abraham Lincoln, is also available for viewing at the museum. Which sentence should be revised to correct a run-on? A. sentence 1 B. sentence 2 C. sentence 3 D. sentence 4 Form B 92 CONTINUE ON TO THE NEXT PAGE 2. Read this sentence. To promote their club, a bake sale will be sponsored by members of the debate team on Wednesday. How should this sentence be revised? E. To promote their club, on Wednesday a bake sale will be sponsored by members of the debate team. F. On Wednesday to promote their club, a bake sale will be sponsored by members of the debate team. G. To promote their club, members of the debate team will sponsor a bake sale on Wednesday. H. Members of the debate team, on Wednesday to promote their club, will sponsor a bake sale. 3. Read this paragraph. (1) Devon spent several hours preparing for an upcoming audition. (2) First, he did vocal exercises to practice his diction and projection. (3) Then, he studies the text of the monologue to better understand the emotions and motivations of the character he plans to portray. (4) Finally, he recited his monologue in front of a mirror many times, making slight adjustments and improvements to his performance each time. Which sentence should be revised to correct an inappropriate shift in verb tense? A. sentence 1 B. sentence 2 C. sentence 3 D. sentence 4 4. Read this sentence. The Colosseum in Rome, Italy which is considered one of the New Seven Wonders of the World, is the largest amphitheater ever built and could hold more than 50,000 spectators. Which edit should be made to correct this sentence? E. delete the comma after Rome F. insert a comma after Italy G. delete the comma after World H. insert a comma after built Form B 93 CONTINUE ON TO THE NEXT PAGE 5. Read this sentence. The engineers tried some other things in the hope of finding a more effective insulation for the compartment. What is the most precise revision for the words The engineers tried some other things? A. The engineers did experiments with several new materials B. The engineers tested foam and fiberglass C. The engineers did tests with two new materials D. The engineers worked with foam and fiberglass Form B 94 CONTINUE ON TO THE NEXT PAGE REVISING/EDITING Part B DIRECTIONS: Read the passage below and answer the questions following it. You will be asked to improve the writing quality of the passage and to correct errors so that the passage follows the conventions of standard written English. You may reread the passage if you need to. Mark the best answer for each question. The Local Library (1) According to a 2015 survey, more than two-thirds of Americans own a smartphone, which means that obtaining information or communicating with people is easier than ever before. (2) With the swipe of a finger or the tap of an icon, people have instant access to articles, blogs, news, and social networking websites. (3) Even with all these immediate sources, one of the best resources also happens to be one of the oldest. (4) The public library is a great place for people to get information. (5) The public library serves a truly critical role in promoting community. (6) There has been an explosion of digital media in recent years. (7) This now allows people in different places to communicate almost instantaneously. (8) And yet many people complain of feeling more isolated and alone than ever before. (9) The library stands in a unique position to help community members meet this universal need for human connection and companionship. (10) Furthermore, libraries offer free assistance, training programs, and Internet access to all people. (11) A student who does not have a computer at home can research and type a paper for school. (12) An unemployed adult without access to the Internet can make use of library resources to find job opportunities. (13) A library’s selection of materials, known as its collection, can include classic novels, reference books, magazines, periodicals, CDs, and DVDs. (14) The stuff that is found at the library can help people get a lot done. (15) Public libraries also offer a variety of resources to community members. (16) One important resource is the local librarian, who does far more than check out books and collect fines. (17) Other library resources include free or low-cost tutoring and training programs. (18) Additionally, lectures, book groups, and town meetings promote critical thinking and community engagement. (19) A typical librarian holds a master’s degree and can help library patrons navigate through the flood of information available in print or on the Internet. (20) The hallmark of a public library is that its materials and services are accessible to all. (21) The library connects people to a network of information and resources and is an important part of a community. 6. Which sentence should replace sentence 4 to best introduce the main claim of the passage? E. The public library is a valuable resource that all members of a community should use. F. Going to the public library to gather information can be a valuable learning experience. G. The resources available at a public library are most useful for students. H. A public library offers many services to those who are interested in improving their communities. Form B 95 CONTINUE ON TO THE NEXT PAGE 7. What is the best way to combine sentences 6 and 7 to clarify the relationship between ideas? A. There has been an explosion of digital media in recent years, since people in different places can now communicate almost instantaneously. B. There has been an explosion of digital media in recent years, although it now allows people in different places to communicate almost instantaneously. C. In addition to the explosion of digital media in recent years, people in different places can communicate almost instantaneously. D. With the explosion of digital media in recent years, people in different places can now communicate almost instantaneously. 8. Which sentence can best follow and support sentence 9? E. Libraries, which have been around for centuries and are found throughout the world today, help preserve the history of a community. F. The public library is often used as a place for community leaders and organizers to host community events, such as hearings or town halls. G. Libraries rely on a combination of local, state, and federal dollars to provide the kinds of services and programs that community members have come to expect. H. At a public library, people can interact with others through a variety of programs, including teen book clubs, toddler story times, and senior-citizen exercise classes. 9. Which transition word or phrase should be added to the beginning of sentence 12? A. For example B. Similarly C. Indeed D. As a result 10. Which revision of sentence 14 best maintains the formal style established in the passage? E. The resources available at the library can help people accomplish many tasks. F. All the materials you can get at the library can help you do many different tasks. G. Everything in the library can help people work on a lot of different things. H. The resources you can find at the library can help you do whatever you need to do. 11. Where should sentence 19 be moved to improve the organization of the fourth paragraph (sentences 15–19)? A. to the beginning of the paragraph (before sentence 15) B. between sentences 15 and 16 C. between sentences 16 and 17 D. between sentences 17 and 18 Form B 96 CONTINUE ON TO THE NEXT PAGE 12. Which sentence is irrelevant to the argument presented in the passage and should be deleted? E. sentence 3 F. sentence 11 G. sentence 13 H. sentence 20 13. Which concluding sentence should be added after sentence 21 to support the argument presented in the passage? A. Finally, patrons of the public library can grow in both their interpersonal life and their level of community engagement. B. Clearly, it is in the best interest of community members to maintain, support, and use their public library. C. After all, the public library has been a part of American communities since the first one was founded in 1833. D. In the end, community leaders must work together to find ways to generate more involvement in public library events. Form B 97 CONTINUE ON TO THE NEXT PAGE Moving Through Mountains (1) An age-old proverb says that necessity is the mother of invention. (2) Centuries of human ingenuity in the face of obstacles prove this to be true. (3) For many years the Swiss Alps, a mountain range spanning southern Switzerland and northern Italy, were such an obstacle. (4) Roads and railways had to navigate around the mountains or through winding tunnels inside the mountains. (5) Transportation of people and goods was difficult and time consuming. (6) In 2016 these burdens were eased with the completion of the Gotthard Base Tunnel. (7) Construction of the high-speed railway tunnel began in 1996. (8) The tunnel was created through the use of tunnel-boring machines, which are giant drills with a rotating head. (9) Each of the tunnel-boring machines used during the construction of the tunnel was about the length of four football fields arranged end-to-end. (10) During the seventeen-year construction period, 28 million tons of rock were removed, enough to rebuild the Great Pyramid at Giza five times. (11) This massive construction project is reported to have cost $12 billion. (12) After that, 4 million cubic meters of concrete, or enough concrete to build eighty-four Empire State Buildings, were used to construct and support the tunnel. (13) By 2020 the high-speed railway will carry more than 250 freight trains and fifty-five passenger trains a day, with most traveling at speeds of around 100 to 125 miles an hour. (14) It will be faster for people to travel between northern and southern Europe. (15) The travel time between the European cities of Zurich, Switzerland, and Milan, Italy, will be reduced by an hour. (16) Many European leaders compare the Gotthard Base Tunnel to the Channel Tunnel, a 33-mile underwater tunnel that connects the United Kingdom and France. (17) While there is no roadway in the Channel Tunnel, people can drive their cars onto special trains that will carry vehicles through to the other side. (18) The Gotthard Base Tunnel project was successful, so now there is renewed interest in solving other problems associated with traveling to and from certain places. (19) Just as traffic congestion in major cities led to the construction of underground local transportation, natural formations, such as mountain ranges, have also sent people underground for faster, easier, and cheaper methods of transportation across larger areas. 14. What is the best way to combine sentences 4 and 5 to clarify the relationship between ideas? E. Roads and railways had to navigate around the mountains or through winding tunnels inside the mountains, making the transportation of people and goods difficult and time consuming. F. Even though roads and railways had to navigate around the mountains or through winding tunnels inside the mountains, the transportation of people and goods was difficult and time consuming. G. Roads and railways had to navigate around the mountains or through winding tunnels inside the mountains, emphasizing that the transportation of people and goods was difficult and time consuming. H. Roads and railways had to navigate around the mountains or through winding tunnels inside the mountains, since the transportation of people and goods was difficult and time consuming. Form B 98 CONTINUE ON TO THE NEXT PAGE 15. Which sentence should follow sentence 6 to most clearly introduce the topic of the passage? A. The Gotthard Base Tunnel was approved by Swiss voters in 1992 and was funded by tolls, fuel taxes, and government loans. B. Leaders from several European countries attended the opening ceremonies for the Gotthard Base Tunnel. C. The Gotthard Base Tunnel is the world’s longest and deepest railway tunnel, stretching 35.5 miles straight through the base of the Swiss Alps. D. The construction of the Gotthard Base Tunnel continues to help reduce the number of freight trucks on the roadways. 16. Which sentence could be added to follow and support sentence 8? E. The tunnel-boring machine is helpful to tunnel builders in the modern era and has been an improvement over dynamite. F. These enormous tunnel-boring machines function somewhat like a cheese grater, grinding slowly through rock and stone. G. Engineers had considered making a tunnel under the mountains for many years, but it was impossible to do without modern tunnel-boring machines. H. Different types of tunnel-boring machines are used depending on the geology of the area where the tunnel is being created. 17. Where should sentence 12 be moved to improve the organization of the second paragraph (sentences 7–12)? A. to the beginning of the paragraph (before sentence 7) B. between sentences 7 and 8 C. between sentences 9 and 10 D. between sentences 10 and 11 18. Which transition word or phrase should be added to the beginning of sentence 15? E. Even so F. Additionally G. For example H. Therefore Form B 99 CONTINUE ON TO THE NEXT PAGE 19. Which sentence is irrelevant to the topic presented in the passage and should be removed? A. sentence 3 B. sentence 11 C. sentence 13 D. sentence 17 20. Which concluding sentence should be added after sentence 19 to support the topic presented in the passage? E. There is proof that underground tunnels like the Gotthard Base Tunnel are beneficial to the economy of the surrounding areas. F. The Gotthard Base Tunnel is an extraordinary example of how human ingenuity and persistence can overcome great obstacles. G. The completion of the Gotthard Base Tunnel shows that people can work together to achieve important goals. H. The Swiss government is confident that the economic impact of the Gotthard Base Tunnel will be worth its construction cost. Form B 100 CONTINUE ON TO THE NEXT PAGE READING COMPREHENSION QUESTIONS 21–57 DIRECTIONS: Read the passage below and answer the questions following it. Base your answers on information contained only in the passage. You may reread a passage if you need to. Mark the best answer for each question. 5 10 15 20 25 30 35 On Monday evening, September 26, 1960, seventy million Americans turned on their TV sets to view the first televised political debate in a campaign for the presidency of the United States. As of that date, it was by far the largest number ever to witness a political discussion. The novelty of the event drew even those with little or no interest in politics. 40 45 The candidates, Republican Vice President Richard M. Nixon and Democratic Senator John F. Kennedy, had agreed to face each other and the nation in four one-hour sessions that the press dubbed the “Great Debates.” Many expected Vice President Nixon to win the debates easily. He was ahead in the newspaper polls, he was an experienced public speaker, and he had served as vice president for nearly eight years. Senator Kennedy was less wellknown and, at forty-three, was the youngest man ever to run for president. Throughout the presidential race, his opponents criticized him for his relative youth and inexperience. 50 55 60 By mutual agreement, the first session was limited to domestic issues within the United States. Each candidate was given eight minutes to make his opening remarks. During the remainder of the hour, the candidates took turns responding to questions posed by selected reporters. Both Kennedy and Nixon dealt with the issues calmly and carefully. Viewers who expected to see a free-for-all were disappointed. The way the two men appeared on the television screen, however, may have been as important as what they said. Kennedy looked at the Form B 65 101 camera while answering questions, appearing to speak directly to his viewers and give them straight answers. Nixon was recovering from a severe bout of influenza, and he appeared tense and tired. He looked at the reporters who asked the questions instead of at the camera, giving some viewers the impression that he avoided eye contact with his audience and thus suggesting that he was not trustworthy. Most commentators agreed that Kennedy gained from the encounter: many viewers who had previously thought he lacked the maturity necessary to be president were won over by his charm, poise, and confident manner. While far fewer people watched the three later sessions, much discussion ensued regarding the influence of the Great Debates on the outcome of the 1960 presidential election. Some feared that the better TV performer was bound to come across as being the better candidate. “Is this a good way to judge a person’s ability to serve as president of the United States?” they asked. Kennedy ultimately won the election, but it was by the narrowest popular vote margin in more than eighty years. Some observers concluded that, had the Great Debates been broadcast on radio and not on television, Nixon would have won. CONTINUE ON TO THE NEXT PAGE 24. How did Kennedy benefit from the debates? 21. Which of the following best tells what this passage is about? E. He was able to prove that he knew more about domestic issues than Nixon did. F. He was able to display favorable personal characteristics despite his inexperience. G. He was able to show that he had a more positive relationship with reporters than Nixon did. H. He was able to persuade viewers to agree with his positions on domestic issues. A. the reasons Nixon was expected to defeat Kennedy in the 1960 election B. the discussion of domestic issues in the 1960 presidential debates C. the events related to the first televised presidential debate D. the qualifications of Nixon and Kennedy for the role of president 22. Which of the following would have been the most likely result if the candidates had not debated on television in 1960? 25. What evidence does the author provide to support the last sentence of the passage? A. Kennedy and Nixon drew much smaller audiences for their later debates. B. Kennedy and Nixon responded to questions calmly and carefully. C. Nixon participated in the debate despite having been recently ill. D. Nixon was more experienced and wellknown than Kennedy. E. Kennedy would have won the election anyway. F. Nixon would have had a better chance of winning the election. G. The election results would have been much closer. H. The debates would not have become a tradition. 26. Why did people who were not normally interested in politics tune in to the first of the Great Debates? 23. Which of the following did critics in 1960 think could be an undesirable consequence of televised presidential debates? E. Vice President Nixon was a popular politician. F. Television had never before been used in this way. G. They had heard that Kennedy was young and charismatic. H. They wanted to see whether the newspaper polls were correct. A. Candidates who are less well-known would have to debate experienced politicians. B. Candidates might have difficulty overcoming the pressure of being on live television. C. Candidates would be evaluated based on their performance rather than their positions. D. Candidates would need to participate in multiple televised debates to fully cover important issues. Form B 102 CONTINUE ON TO THE NEXT PAGE 5 10 15 20 25 30 35 If you have ever watched someone fall on the ice, you’ve seen slipperiness at work. But have you wondered what makes ice slippery, or why skates or skis glide across ice so easily? The answer might seem obvious: ice is smooth. Yet smoothness in itself does not explain slipperiness. Imagine, for example, skating on a smooth surface of glass or sheet metal. 40 45 Surprisingly, scientists do not fully understand why ice is slippery. Past explanations of slipperiness have focused on friction and pressure. According to the friction theory, a skate blade rubs across the ice, causing friction. The friction produces heat, melting the ice and creating a slippery, microscopically thin layer of water for the skate to glide on. The friction theory, however, cannot explain why ice is slippery even when someone stands completely motionless, creating no friction. 50 55 The pressure theory claims that pressure from a skate blade melts the ice surface, creating a slippery layer of water. The water refreezes when the pressure is lifted. Science textbooks typically cite this explanation, but many scientists disagree, claiming that the pressure effect is not great enough to melt the ice. Nor can the pressure theory explain why someone wearing flat-bottomed shoes— which have a greater surface area than skate blades and thus exert less pressure per square inch—can glide across the ice or even go sprawling. 60 65 70 During the 1990s, another theory found acceptance: the thin top layer of ice is liquid, Form B 103 or liquid-like, regardless of friction or pressure. This notion was first proposed more than 150 years ago by physicist Michael Faraday. Faraday’s simple experiment illustrates this property: two ice cubes held against each other will fuse together. This happens, Faraday explained, because liquid on the cubes’ surfaces freezes solid when the surfaces make contact. Faraday’s hypothesis was overlooked, in part because scientists did not have the means to detect molecular structures. However, technological advances during recent decades have allowed scientists to measure the thin layer on the surface of ice. For example, in 1996, a chemist at Lawrence Berkeley Laboratory shot electrons at an ice surface and recorded how they rebounded. The data suggested that the ice surface remained liquid-like, even at temperatures far below freezing. Scientists speculate that water molecules on the ice surface are always in motion because there is nothing above them to hold them in place. The vibration creates a slippery layer of molecules. According to this interpretation of the Lawrence Berkeley Laboratory experiments, the molecules move only up and down; if they also moved side to side, they would constitute a true liquid. Thus it could be said that people are skating on wildly vibrating molecules! The phenomenon of a slippery, liquid-like surface is not limited to ice, although ice is the most common example. Lead crystals and even diamond crystals, which are made of carbon, also show this property under certain temperature and pressure conditions. CONTINUE ON TO THE NEXT PAGE 31. According to researchers at the Lawrence Berkeley Laboratory, why is the surface of ice liquid-like rather than a true liquid? 27. Which of the following best tells what this passage is about? A. the circumstances that allow ice to melt B. the theories behind what makes ice slippery C. the discoveries of Michael Faraday D. the processes of freezing and melting A. because electrons rebound from the ice surface B. because the ice surface molecules vibrate only up and down C. because the ice surface is a different temperature D. because the ice surface is under pressure 28. What is the most likely reason that the author mentions lead and diamond crystals in the last paragraph? 32. According to the passage, which of the following undermines the friction theory of slipperiness? E. to point out that solids other than ice have slippery surfaces F. to suggest that ice, lead, and diamonds are composed of the same materials G. to cast doubt on Faraday’s theory of slipperiness H. to illustrate the effects of temperature and pressure on slipperiness E. a person wearing flat-bottomed shoes gliding across an icy surface F. two ice cubes fusing together G. electrons bouncing off the surface of ice H. a person sliding while standing immobile on an icy surface 29. According to Faraday, why do two ice cubes fuse when held together? 33. The author includes details about the pressure theory in the third paragraph in order to A. Friction causes the surfaces of the ice to melt and refreeze. B. The applied pressure forces the surfaces of the cubes to stick to each other. C. Liquid layers on the surfaces freeze together when the surfaces make contact. D. The liquid layers on the surfaces of the ice are perfectly smooth. A. highlight that there are different ideas regarding this theory about the slipperiness of ice. B. describe the most recently accepted theory for the slipperiness of ice. C. emphasize that this theory has been tested by scientists who study the slipperiness of ice. D. show that there is scientific support for a plausible theory about the slipperiness of ice. 30. What is the most likely reason that the author mentions the 1996 experiment at Lawrence Berkeley Laboratory? E. to provide evidence about the surface of ice F. to illustrate that studying ice molecules is difficult G. to show how experiments on ice are inconclusive H. to explain why ice cubes freeze together Form B 104 CONTINUE ON TO THE NEXT PAGE 5 10 15 20 25 30 35 40 45 people may use and enjoy, but not destroy, the riches of the earth. One of the books that has done the most to alert the world to the dangers of environmental degradation was George Perkins Marsh’s Man and Nature. Its message—that Western society was in the process of causing irreparable harm to the environment—greatly influenced ecologists during the beginning of the modern environmentalist movement in the 1960s. Marsh was not, however, part of this movement. Surprisingly, Man and Nature was first published in 1864. 50 55 Marsh first observed the environmentally destructive effects of human activities while growing up in Vermont in the early nineteenth century. The heavy demand for firewood had depleted the forests, and extensive sheep grazing had stripped the land. The result was flooding and soil erosion. Furthermore, streams were fouled by wastes dumped from numerous mills and dye houses. 60 65 Much later in his life, after careers in law, business, farming, and politics, Marsh served as ambassador to Italy. There he noticed land abuse similar to what he had seen in Vermont. Overgrazing and forest mismanagement had rendered desolate areas that had been productive farmland since the days of the Roman Empire. Marsh attributed this to what he called “man’s ignorant disregard for the laws of nature.” 70 75 In Italy, Marsh began to organize his observations and theories. He wrote in a way intended to educate readers about the impact of industrial and agricultural practices on the environment. In Man and Nature, he evaluated the important relationships between animals and plants, discussed forestry practices in great detail, and analyzed the ways natural water supplies are affected by human use. 80 85 Furthermore, he asserted that everything in nature is significant and that even the tiniest organism affects the fragile environmental balance. His belief that drastic alteration of this balance would be dangerous is now accepted as a fundamental principle of modern environmental science. Although he pointed out environmental damage caused by irresponsible human activities, Marsh did not oppose every human alteration to the environment. To him, the goal was proper management, not a return to wilderness conditions. People should consider the consequences of their actions, he wrote, and become “co-worker[s] with nature.” Marsh praised the Suez Canal, the humanmade waterway between the Mediterranean Sea and the Gulf of Aden, as “the greatest and most truly cosmopolite physical improvement ever undertaken by man.” He believed that the advantages of the canal— improved transportation and commerce— would outweigh any environmental damage. Yet he also warned of possible unintended consequences, such as destructive plants and animals spreading from one body of water to the other. Marsh was considered a radical thinker during his lifetime. By the late nineteenth century, however, his writings, along with those of John Muir, Henry David Thoreau, and others, had inspired what became known as the conservation movement. The conservationists of that time sought to educate the public that wilderness areas were worth preserving, and they were responsible for creating the National Park Service and the National Forest Service. Man and Nature challenged the popular belief that nature could heal any damage that people inflict upon it. Marsh argued that Form B 105 CONTINUE ON TO THE NEXT PAGE 34. Which statement best describes the central idea of the passage? 36. What is the most likely reason the author uses the word “surprisingly” in line 11? E. Marsh’s experience growing up on a farm allowed him to witness firsthand how human demands on nature can lead to problems, and as an adult he wrote one of the first books about conservation. F. Marsh challenged the notion that nature could repair the damage people cause to it, but he also supported human-made modifications to nature that improve transportation and commerce. G. Marsh’s ideas about the environment were considered radical in his lifetime, but they later gained popularity during the environmental movement in the twentieth century. H. Marsh was a radical thinker who believed that people’s actions could dramatically affect nature, and his writings are considered foundational to the conservation movement. E. to argue that Marsh’s ideas are more applicable in the present than they were during his lifetime F. to show that Marsh introduced ideas a century before they became widely accepted G. to emphasize that Marsh was unaware that his ideas would help begin a conservation movement H. to prove that there would be fewer issues with the environment today if people had accepted Marsh’s ideas earlier 37. Which evidence supports the idea that Marsh’s theories about nature were accurate? A. the details about Marsh’s observations of environmental degradation B. the details about how Marsh’s writing inspired a conservation movement C. the details about how Marsh’s ideas are essential to modern environmental science D. the details about Marsh’s opinion on human alterations to the environment 35. Marsh believed that the people of his time caused harm to the environment mostly because A. they assumed future generations would solve any environmental problems. B. they thought industrial progress was more important than protecting nature. C. they were unwilling to change farming and waste-disposal practices. D. they lacked knowledge of nature and natural processes. Form B 106 CONTINUE ON TO THE NEXT PAGE 38. Which detail about Marsh provides support for the author’s statement in lines 55–58? 39. Which of Marsh’s ideas most influenced the environmental movement of the 1960s? E. his reputation as a radical thinker F. his contribution to the conservation movement G. his experience working as an ambassador H. his approval of beneficial human-made projects in nature Form B A. Some human alterations to the environment are necessary. B. People lack an understanding of the environment. C. Human activities could damage the environment. D. Environmental degradation has been occurring for many years. 107 CONTINUE ON TO THE NEXT PAGE 5 Anyone who has watched TV news coverage of a hurricane has seen how destructive wind energy can be. But the power of the wind can also be put to constructive use. From sailboats to old-fashioned windmills to hightech wind turbines, people have devised ways to harness wind energy for thousands of years. 40 45 10 15 20 25 30 35 The first known attempt to use wind power was the sailboat. Ancient shipbuilders understood how to use forces like lift and momentum, even if they could not explain those forces scientifically. The principles behind sailing led to the development of the windmill. The first known windmills originated in Persia, an area that is now Iran, as early as A.D. 500. They were created to help with the demanding chores of grinding grain and pumping water. By the tenth century, windmills were used throughout central Asia; they were used in China as early as the thirteenth century. 50 55 In Europe, windmills came into widespread use during the twelfth century. As in other parts of the world, they were used for milling grain and pumping water. Windmills replaced the water wheel, which was turned by the movement of running water over paddles mounted around a wheel. The windmill was more adaptable and efficient than the water wheel and quickly became popular. For example, Holland, famous for its windmills, used the machines to pump seawater away from low-lying coastal bogs. This allowed the Dutch to reclaim large areas of land from the sea. Windmills eventually became sophisticated enough for use in a broad range Form B 60 65 70 108 of work, from sawmills and drainage pumping to processing goods, such as dyes, tobacco, cocoa, and spices. In the 1700s, as steam engines gained in popularity, the use of wind machines for many types of work declined. However, windmills still played an essential role in pumping water on farms throughout the American West and Midwest. Between 1850 and 1970, over six million small windmills were installed on American farms for watering livestock and meeting other water needs. In many remote areas even today, livestock production would be impossible without the use of windmills to provide water. Beginning in the late nineteenth century, windmills were adapted to generate electricity. During the 1930s and ’40s, thinbladed windmills provided electricity for hundreds of thousands of farms across the United States. By the 1950s, however, power lines connected almost every household in America to a central power source, such as a utility company. After that, there was little need for wind turbines until the energy crisis of the 1970s. At that time, interest in wind turbines was renewed due to rising energy costs and concern about the future availability of fossil fuels, such as oil, coal, and natural gas. The last several decades have seen the development of “wind farms,” clusters of wind turbines that generate electricity. Efficient, clean, and fairly inexpensive to operate, wind farms may prove to be as important in the future as earlier windmills were in the past. CONTINUE ON TO THE NEXT PAGE 40. Which of the following best tells what this passage is about? 43. Why were fewer American farms dependent on wind machines for electrical power after the 1950s? E. the ways people have harnessed wind power throughout history F. reasons for developing wind farms to generate electricity G. how windmills are used in the United States H. why windmills were modified to generate electricity A. The energy crisis had prompted interest in other fuel sources. B. Coal and natural gas could generate more power than wind turbines. C. Centralized power systems connected most places. D. Wind farms had reduced the need for individual windmills. 41. What evidence best supports the idea that windmills were important resources in the western United States? 44. Which of the following best expresses the author’s opinion regarding the future use of wind energy? A. the details about the development and use of wind farms B. the details about how windmills provided water for farms and livestock C. the details about the role of wind energy during the energy crisis of the 1970s D. the details about how steam engines influenced the use of windmills E. Wind farming methods are unlikely to increase electrical output. F. Wind farms will most likely be used to provide energy in rural areas. G. Wind farming output will likely supplement other sources of energy. H. Wind farms may become an important source of electricity. 42. The author describes the tasks windmills were used for in lines 36–40 in order to 45. According to the passage, the people of Holland most notably used windmills to E. emphasize that windmills served a variety of purposes. F. highlight that windmills are more efficient than water wheels. G. emphasize that windmills were used for many years. H. convey that some countries used windmills more than other countries. Form B A. B. C. D. 109 pump water to remote locations. turn bogs into usable land. make the country famous. process a variety of goods. CONTINUE ON TO THE NEXT PAGE 5 10 15 20 25 30 35 Archaeologists first succeeded in using tree-ring dating while excavating ancient Pueblo Native American villages in the southwestern United States during the 1920s. At that time, no one knew when the villages had been occupied, or for how long, but the logs used in the buildings provided a clue. Scientists had long known that trees add a new growth ring to their circumferences during each growing season. Drought or early frost results in little growth and narrow rings. Good growing years result in wide rings. Archaeologists knew that by matching identical patterns of wide and narrow rings in sections of two different logs, they could determine which log was older. For example, a log with a certain pattern of rings near its outside edge would indicate a specific series of good and bad growing seasons. This log would have been cut down before a log of comparable size that shows the identical pattern near its center. 40 45 50 55 But how could these ring patterns help determine the actual dates for the abandoned Pueblo villages? Archaeologists had already used the ring patterns of trees with overlapping lifetimes to establish a tree-ring chronology for the southwestern United States that went back to A.D. 1260. That work had been done in a Hopi village called Oraibi. Oraibi had been continuously inhabited since before the arrival of the first Spanish explorers in 1540. 60 65 logs used in the buildings. With this floating chronology, the archaeologists could tell which logs were older and which were more recent. None could be precisely dated, since no log had a pattern of tree rings that matched any part of the established chronology. It was clear from this evidence, however, that the buildings must have been constructed before 1260. Finally, continued excavations turned up a “key” beam. The outer ring pattern of the key beam overlapped the earliest rings in the established chronology. Furthermore, its inner ring pattern matched the pattern formed by the most recent rings of the floating chronology. Thus, the chronology for the abandoned Pueblo villages could be known with certainty. Counting backward from the present, the archaeologists estimated that the villages had been occupied between 900 and 1300. The tree rings also suggested why the villages had been abandoned. The rings for the years 1276 to 1299 were very thin, indicating a severe drought that lasted for twenty-three years. Most likely the villagers had left their homes to search for a more hospitable climate. That same team of archaeologists also developed a relative, or floating, chronology for the abandoned Pueblo villages by matching up the ring patterns of the various Form B 110 CONTINUE ON TO THE NEXT PAGE 46. Which of the following best tells what this passage is about? 49. Which evidence best supports the idea that tree-ring dating helped archaeologists understand why the Pueblo villages were abandoned? E. how variations in weather conditions affect tree growth F. recent breakthroughs in understanding Native American cultures G. why the Pueblo villages were abandoned H. how tree-ring dating can establish the age of archaeological findings A. the details about the Hopi village Oraibi B. the details about the twenty-three-year drought C. the details about the discovery of the Pueblo villages D. the details about the floating chronology 47. What was the importance of the key beam described in the fourth paragraph? 50. Why did the archaeologists conclude that the buildings in the abandoned Pueblo villages were constructed before A.D. 1260? A. It proved that trees of the same age would have identical tree-ring patterns. B. It helped archaeologists determine why the villages had been built at that time. C. It revealed a new floating chronology for archaeologists to investigate. D. It connected the floating chronology to the established chronology. E. The logs in the Pueblo buildings did not share any ring patterns with the established chronology from the Hopi village. F. The logs in the Pueblo buildings had fewer rings than the logs from the buildings in the Hopi village. G. The Pueblo villages were abandoned before the Hopi village was established. H. The Pueblo villages and the Hopi village were constructed from logs of similar size. 48. The author explains the causes of wide and narrow tree rings in lines 8–13 in order to E. emphasize that tree-ring dating is highly accurate. F. highlight how tree rings help determine past weather patterns. G. emphasize that a tree’s ring patterns grow in a unique way. H. highlight the importance of studying tree rings. Form B 51. Why did archaeologists compare the logs from buildings in the abandoned Pueblo villages with logs from the Hopi village? A. to find the oldest and most recent logs used in the Pueblo villages B. to find evidence that would help explain what caused people to leave the Pueblo villages C. to find a log from the Pueblo villages that had tree rings that matched logs from the Hopi village D. to find evidence that the Pueblo villages were constructed before the Hopi village 111 CONTINUE ON TO THE NEXT PAGE 5 10 15 20 25 30 35 The decade that began with the stock market crash in 1929 and ended with the declaration of war in Europe in 1939 was a turning point for art in the United States. Rejecting European trends, such as abstract art, American painters searched for a style that was distinctly American. It was a time of great social change—a society based on rural and small-town life was rapidly being replaced by a society focused on city life and values. Although members of various groups are all referred to as “American Scene” painters, different groups painted their images of the United States in very different ways. 40 45 50 One group, sometimes called the Regionalists, included Thomas Hart Benton, Grant Wood, and John Steuart Curry, all from the Midwest. Their art was intensely patriotic and frequently glorified an older, simpler United States. Their subject matter included church steeples, New England fishing villages, and Midwestern cornfields. Grant Wood’s most famous canvas is probably American Gothic, which shows a stiff and proper farm couple, the husband holding a pitchfork. The Regionalists were often muralists as well, painting local scenes on walls of state capitols and other public buildings. Enormously popular during the 1930s, Regionalist art is still treasured by many as a fond memory of times gone by. 55 60 65 While the Regionalists remembered the past, other American Scene artists painted the drab realities of the contemporary urban environment, testifying to its loneliness and anonymity. The Urban Realists, including Form B 70 112 Reginald Marsh, Isabel Bishop, and the Soyer brothers, were associated with the Art Students League in New York. These painters showed the high price paid by individual men and women struggling to survive the Depression. The names of some of their works illustrate the style: Office Girls, Waiting, The Bowery. For various reasons, their work has been largely forgotten today. Edward Hopper was an artist who was associated with the American Scene but otherwise escaped further classification. Like the Urban Realists, he painted the tired dinginess of the urban streets during the Depression. Yet Hopper often found beauty in the midst of the city’s monotony. For example, one of Hopper’s best-known paintings, Nighthawks, shows several people sitting like robots in a brightly lit coffee shop at night, each apparently unaware of the others. Hopper was not interested in a return to the past. He presented what he saw without apology or sentimentality. The American Scene art movement of the 1930s was characterized by realistic paintings that expressed the traditions and interests of people in the United States at that time. Because the paintings presented common images and mirrored the lives of many people, the general public readily identified with the subjects of the paintings. With the onset of World War II, a new spirit of internationalism swept through the art of the United States, and the American Scene painters became out of date. Although the movement did not last, it had reflected its own time with profound understanding. CONTINUE ON TO THE NEXT PAGE 52. Why did ordinary people in the 1930s identify with the art of the American Scene painters? 55. How does the fourth paragraph contribute to the passage? A. It describes the end of the American Scene movement. B. It highlights Edward Hopper as a unique American Scene painter. C. It explains why Edward Hopper’s work has been forgotten. D. It contrasts the American Scene and Urban Realist styles. E. The artists were primarily concerned with painting farm life. F. People appreciated the beauty reflected in the paintings. G. People wanted paintings to show social and cultural change. H. The paintings reflected the times in ways that were familiar to most viewers. 56. The author uses the phrase “without apology” (lines 59–60) to explain that Hopper did not think he needed to justify 53. Which of the following subjects would an Urban Realist painter be most likely to represent? E. F. G. H. A. a scene with factory workers going home from work B. a scene with children playing games in a street C. a scene of a bustling European city D. a scene showing the stores along the main street in a city 57. What is the most likely reason that Regionalist art has retained some of its popularity while Urban Realist art has not? A. Regionalist art depicts modern life as well as life in the past, while Urban Realist art depicts only the past. B. Regionalist art more accurately portrays the time in which it was painted than Urban Realist art does. C. Regionalist art shows American life as people wish to remember it, while Urban Realist art does not. D. Regionalist art depicts Americans overcoming the Depression, while Urban Realist art depicts life during World War II. 54. Hopper’s paintings contrast with the work of the Urban Realist painters by E. showing the ugliness of a city environment. F. illustrating the move toward an international style. G. revealing the ways that dull urban life can include beauty. H. focusing on the portrayal of people rather than on the setting. Form B how he portrayed his subjects. painting scenes of real places. why his paintings became popular. the classification of his art style. 113 CONTINUE ON TO THE NEXT PAGE PART 2 — MATHEMATICS Suggested Time — 90 Minutes 57 QUESTIONS IMPORTANT NOTES (1) Formulas and definitions of mathematical terms and symbols are not provided. (2) Diagrams other than graphs are not necessarily drawn to scale. Do not assume any relationship in a diagram unless it is specifically stated or can be figured out from the information given. (3) Assume that a diagram is in one plane unless the problem specifically states that it is not. (4) Graphs are drawn to scale. Unless stated otherwise, you can assume relationships according to appearance. For example, (on a graph) lines that appear to be parallel can be assumed to be parallel; likewise for concurrent lines, straight lines, collinear points, right angles, etc. (5) Reduce all fractions to lowest terms. GRID-IN PROBLEMS QUESTIONS 58–62 DIRECTIONS: Solve each problem. On the answer sheet, write your answer in the boxes at the top of the grid. Start on the left side of each grid. Print only one number or symbol in each box. DO NOT LEAVE A BOX BLANK IN THE MIDDLE OF AN ANSWER. Under each box, fill in the circle that matches the number or symbol you wrote above. DO NOT FILL IN A CIRCLE UNDER AN UNUSED BOX. 58. How many 5-digit numbers can be created using the digits 2, 3, 5, 7, and 8 without repeating any digits within that 5-digit number? 60. (−6) − (−5) + 4.2  −  3 − 9.6  = 147− x = 12 12 59. What is the value of x in the equation shown above? Form B 114 CONTINUE ON TO THE NEXT PAGE 61. 62. Tyler has completed 60 pages in his French workbook. This is 20% of the total number of pages in the workbook. How many pages are in the workbook? 30° y° 60° P 25° Four straight lines intersect at point P as shown above. What is the value of y ? Form B 115 CONTINUE ON TO THE NEXT PAGE MULTIPLE CHOICE PROBLEMS QUESTIONS 63–114 DIRECTIONS: Solve each problem. Select the best answer from the choices given. Mark the letter of your answer on the answer sheet. You can do your figuring in the test booklet or on paper provided by the proctor. DO NOT MAKE ANY MARKS ON YOUR ANSWER SHEET OTHER THAN FILLING IN YOUR ANSWER CHOICES. 63. If  x = 9 and y = −7 , what is the value of x ( x − 2 y) ? A. B. C. D. M = 3N = P = Q + 5 = R > 0 4 7 65. Based on the statement above, which variable has the greatest value? 18 45 144 207 A. M B. N C. 64. P D. R Q R 66. 50° 70° P T S E. F. G. H. In the figure above, PQRS is a parallelogram. The measure of ∠PQT is 50° , and the measure of ∠PTQ is 70° . What is the measure of ∠QRS ? 67. E. 60° F. 70° G. 80° H. 120° Form B A roofing contractor uses shingles at a rate of 3 bundles for every 96 square feet of roof covered. At this rate, how many bundles of shingles will he need in order to cover a roof that is 416 square feet? To make party invitations, Macie could buy a package of paper for $10.50, or she could buy x individual sheets of the same paper for $0.15 each. What is the largest value of x that would make buying the individual sheets less expensive than buying the package? A. B. C. D. 116 5 12 13 14 60 65 69 70 CONTINUE ON TO THE NEXT PAGE 68. 70. At 1:00 p.m. one day, the temperature was 8 degrees above zero. During the rest of the day, the temperature fell 3 degrees per hour. What was the temperature at 7:00 p.m. that day? DESSERT CHOICES E. −13° F. Dessert Number of Times Ordered Cookies 42 Pie 23 Cake 47 Ice Cream 48 −10° G. −7° H. 5° 69. A bag contains 75 marbles that are red, blue, or green. The ratio of red to blue marbles is 15:7, and the ratio of blue to green marbles is 7:3. If 2 blue marbles are removed and replaced with 2 green marbles, what will be the new ratio of red to green marbles? The table above shows the number of times that different desserts were ordered at a restaurant. Based on this information, what is the probability of a customer ordering ice cream as a dessert? A. B. C. D. E. F. G. H. 3:1 5:1 7:5 45:11 71. What is the least common multiple of 24, 6, and 18? A. B. C. D. 72. 117 36 48 72 144 One day, the Early Bird Restaurant used 15 dozen eggs for 200 breakfast customers. At this rate, approximately how many dozen eggs are needed for 300 breakfast customers? E. F. G. H. Form B 25% 30% 40% 48% 20 23 25 30 CONTINUE ON TO THE NEXT PAGE 73. 74. 75. A cooler contains three types of beverages: 5 bottles of apple juice, 3 bottles of grape juice, and 6 bottles of fruit punch. What is the probability that a bottle chosen at random from this cooler is not apple juice? PRICES FOR AD SPACE Space Price A. 1 9 1 4 200 B. 5 14 1 2 350 C. 9 14 full page 600 D. 2 3 The table above shows prices for newspaper advertising. A store purchased 1 pages, 4 1 pages, and full pages of space in equal 2 numbers for a total of $11,500. What is the A large circular dinner plate has a radius of 20 centimeters. A smaller circular plate with a circumference of 20  centimeters is placed in the center of the larger dinner plate. What is the area of the part of the larger dinner plate that is not covered by the smaller plate? total amount of page space the store purchased? A. 1 3 pages 4 E. 20  sq cm F. 100  sq cm B. 10  pages G. 200  sq cm C. 16 1 pages 2 H. 300  sq cm D. 17 1 pages 2 Form B 118 CONTINUE ON TO THE NEXT PAGE 76. 79. y R the total cost of the cloth? A. B. C. D. S T V Bryana bought 1 3 yards of cloth at $8.00 per 4 yard. If there was an 8% sales tax, what was x U $12.96 $14.08 $15.12 $16.08 80. For which point on the graph above is the value of the y-coordinate greater than the value of the x -coordinate? E. F. G. H. 77. On the number line above, MN = 5 5 . What 6 is the position of point M ? E. −7 1 6 3 4 9 12 F. H. 7 1 6 Y  is between X and Z . Length  YZ = 3 XY , 5 and length XY = 20 centimeters. What is the 81. length of XZ ? Form B −4 1 2 G. 4 1 2 Points X , Y, and Z are on a straight line, and E. F. G. H. N 11 3 R T U V If 36 = 4 x , what is the value of x  when y y=3? A. B. C. D. 78. M 12 cm 24 cm 30 cm 32 cm A United States presidential coin is made from an alloy of four metals—copper, zinc, manganese, and nickel—with weights in the ratio of 177:12:7:4, respectively. The coin weighs a total of 8 grams. What is the weight of the zinc in this coin?  A. B. C. D. 119 0.28 g 0.48 g 0.96 g 48 g CONTINUE ON TO THE NEXT PAGE 82. E. F. G. H. 83. 86. Jack scored an average of 15 points per game in his first 3 basketball games. In his 4th game, he scored 27 points. What is his average score for the first 4 games? E. w + 1 15 17 18 21 F. H. 2w + 1 87. when it is completely full. Currently, the drum is 1  full of oil. How many kiloliters of oil 3 need to be added in order to fill the drum completely? 84. 1.44 kL 2.88 kL 4.32 kL 14.10 kL Three students stand at the starting line of a running track and begin running laps at the same time. Ann completes 1 lap every 2 minutes, Jack completes 1 lap every 3 minutes, and Lee completes 1 lap every 4 minutes. How many laps does Ann complete before all three runners are once again at the starting line at the same time? A. B. C. D. Nicole’s age now is three times Carmen’s age. If Carmen will be 17 in two years, how old was Nicole 5 years ago? E. F. G. H. 88. 4 ( 7 − 3x ) − ( 5 − x ) 38 yr 40 yr 45 yr 50 yr E. 23 − 4 x Form B 23 − 11x G. 28 − 4 x A chemical decays in such a way that the amount left at the end of each week is 20% less than the amount at the beginning of that same week. What percent of the original amount is left after two weeks? A. B. C. D. 4 6 12 20 Simplify this expression: F. 85. 2w − 1 G. 2w − 2 A cylindrical oil drum can hold 4,320 liters A. B. C. D. If w − 1 is an odd integer, which one of the following must be an even integer? H. 28 − 12x 40% 60% 64% 80% 120 CONTINUE ON TO THE NEXT PAGE 5x 3 + 3x + 9 + 12 x 91. 89. If x = 10 , what is the value of the expression above? PET SURVEY Number of Pets Number of Students 0 12 1 16 2 7 3 or more 5 A. B. C. D. 2,539.01 5,039.01 5,039.1 5,139 92. N S P Amy surveyed students at her school about the number of pets they have. What is the probability that a student who participated in the survey has at least 2 pets? 90. A. 7 40 B. 1 12 C. 1 8 D. 3 10 M E. F. G. H. 14 15 26 30 93. Q 121 9 sq cm 12 sq cm 18 sq cm 36 sq cm The Chens spend $5 of every $8 they earn on planned expenses. If the family earns $29,600 in one year, how much will they spend on planned expenses that year? A. B. C. D. Form B T R, S, and T are midpoints of the sides of square MNPQ, as shown above. What is the sum of the areas of the shaded triangles? A large container is partially filled with n liters of water. Ito adds 10 liters of water to the container, making it 60% full. If Ito adds 6 more liters of water, the container will be 75% full. What is the value of n ? E. F. G. H. 6 cm R $1,850 $3,700 $5,920 $18,500 CONTINUE ON TO THE NEXT PAGE 94. 97. A pizza shop offers a choice of 3 sizes (small, medium, and large) and 7 different toppings. Different pizzas can be created by changing the size and/or the choice of toppings. If Cody wants to order a pizza with exactly 2 different toppings, how many different pizzas can he create? E. F. G H. A. B. C. D. 6 21 63 126 98. 95. SURVEY OF CATS PER FAMILY Number of Cats Number of Families 0 15 1 42 2 35 3 or more 8 99. 96. 1 kph 5 kph 7 kph 9 kph Points P, Q, R, and S represent −3, −1, 0, and 2, respectively, on a number line. How many units is the midpoint of PQ  from the midpoint of RS ? E. F. G. H. 1 2 3 4 There are 1,000 cubic centimeters in 1 liter, and 1,000 cubic millimeters in 1 milliliter. How many cubic millimeters are there in 1,000 cubic centimeters? A. B. C. D. The table above shows the number of cats per family in 100 households in the Blaine neighborhood. By what percentage is the number of families with 1 cat greater than the number of families with 2 cats? A. B. C. D. On a bike trip, Rajiv traveled 65 kilometers in 5 hours, while Shaina traveled 72 kilometers in 4 hours. How much less was Rajiv’s mean speed, in kilometers per hour (kph), than Shaina’s? 1,000 10,000 100,000 1,000,000 7% 10% 17% 20% A wooden box has a square base. The height of this box is 3 times the length of one side of the base. If one side of the base is 3 feet long, what is the volume of this box? E. F. G. H. Form B 9 cu ft 27 cu ft 36 cu ft 81 cu ft 122 CONTINUE ON TO THE NEXT PAGE 101. 100. M −1 4 1 2 In the quarter circle above, what is y  in terms of x ? 102. E. x −1 F. x +1 G. x +1 2 H. Form B T 5 8 The hash marks on the number line above are evenly spaced. What is the coordinate of point R? x+ y R A. 7 40 B. 9 40 C. 11 40 D. 21 40 Phan chose an Internet service that charges $18.00 per month plus $0.024 per minute. Deion chose an Internet service that charges $30.00 per month for unlimited usage. At the end of the month, Phan’s and Deion’s charges were identical. For how many minutes did Phan use the Internet service that month? E. F. G. H. ( x +1)2 2 123 50 min 60 min 100 min 500 min CONTINUE ON TO THE NEXT PAGE 103. 105. In a sample of 50 cars at a local dealership, there are 12 red cars and 10 cars with backup cameras. Of the 12 red cars, 4 have backup cameras. If a car is selected at random from the given sample, what is the probability that both of the following are true: the car is not red and does not have a backup camera? y 4 104. A. 3 5 B. 16 25 C. 19 25 D. 4 5 O m n x What is the area of the shaded triangle shown above? A. m + n The decimal 0.06 can be written as the B. n − m fraction x . What is the value of x ? 50 E. F. G. H. Form B C. 2( n − m) 3 6 12 30 D. 4 ( n − m) 124 CONTINUE ON TO THE NEXT PAGE 106. 14 = p 21 7 108. ANIMAL CARDS In the equation above, what is the value of p ? Number of Cards Picture on Card 8 cat E. 6 dog F. 3 5 bird G. 14 3 4 fish 1 horse H. 14 109. The cards in the table above are mixed in a box. Which animal pictured on a card has exactly a 1 in 4 chance of being picked at random from the box? E. F. G. H. 107. cat dog fish horse Which number line below shows the solution set for 2x − 2 ≤ y ≤ 4 x + 10  when y = 1 ? A. –3 –2 –1 0 1 2 3 B. –3 –2 –1 0 1 2 3 C. –3 –2 –1 0 1 2 3 D. –3 –2 –1 0 1 2 3 Form B 2 3 110. A ball is selected at random from a box that contains 7 black balls, 14 green balls, and 21 red balls. What is the probability that the ball selected is black? A. 1 6 B. 1 5 C. 1 3 D. 5 6 At North High School, a survey asked two questions, Question A and Question B. For each question, students could answer either “yes” or “no.” Of the 800 students who responded to the survey, 720 answered “yes” to Question A, and 640 answered “yes” to Question B. What is the least possible number of these students who could have answered “yes” to both questions? E. F. G. H. 125 80 160 560 640 CONTINUE ON TO THE NEXT PAGE 111. 113. Raoul is at least 3 years older than Vahn. Which of the following inequalities gives the relationship between Raoul’s age ( r ) and Vahn’s age ( v )? A. B. C. D. A. r − v ≥ 3 B. r − v ≤ 3 C. 3 − v ≤ r D. 3 − r ≤ v 112. 114. 1 sind = 5.6 ricks 1 sind = 12.88 dalts Using the conversion above, how many dalts are equal to 1 rick? E. F. G. H. There are now  x  cans stacked on a shelf that holds 36 cans when full. If 4 of these cans were removed, the shelf would be half full. What is the value of x ? Carlos tossed a paper cup in the air 50 times and found that the probability of it landing on its side was 72%. If he tosses the cup in the air 150 more times, what is the total number of times he can expect the cup to land on its side? E. F. G. H. 0.43 dalt 2.30 dalts 7.28 dalts 18.48 dalts 14 16 18 22 72 108 144 158 THIS IS THE END OF THE TEST. IF TIME REMAINS, YOU MAY CHECK YOUR ANSWERS. BE SURE THAT THERE ARE NO STRAY MARKS, PARTIALLY FILLED ANSWER CIRCLES, OR INCOMPLETE ERASURES ON YOUR ANSWER SHEET. j Form B 126 SAMPLE TEST – ENGLISH LANGUAGE ARTS EXPLANATIONS OF CORRECT ANSWERS REVISING/EDITING PART A FORM BB REVISING/EDITING PART B 1. (A) The question asks for the identification of a run-on sentence that needs to be corrected. Option B, Option C, and Option D identify complete sentences that are not run-ons. Option A identifies a sentence made up of two independent clauses that need to be more definitively separated between “opened” and “as,” using either a semicolon or a period. The Local Library 6. (E) The question asks for a sentence that should replace sentence 4 to introduce the main claim in the passage. Option F states that going to the library can be a learning experience. This idea is implied in sentences 17 and 18, but it is not the main claim of the passage. Option G states that the resources at a public library are most useful for students, which is discussed in the third and fourth paragraphs. The idea that the library is useful for students is offered as evidence for the main claim. Option H states the fact that local libraries provide services to help improve communities, which is explained in sentences 15–18, but this statement does not present an argument. Option E best presents the argument that the public library is a valuable resource and should be used by community members. 2. (G) The sentence in the box demonstrates the use of a misplaced modifier. In Option E and Option F, the phrase “to promote their club” incorrectly modifies “a bake sale.” While Option H makes it clearer that “to promote their club” refers to the “members of the debate team,” the rest of the sentence is poorly written. Option G is the only option in which the phrase “to promote their club” clearly modifies “members of the debate team” and that clarifies that the bake sale is on Wednesday. 7. (D) This question asks for the best way to combine sentences 6 and 7 to clarify the relationship between ideas. Option A makes an incorrect connection that the explosion of digital media happened “since” people can communicate instantaneously. Option B makes an incorrect connection because the use of the conjunction “although” suggests that the explosion of digital media in recent years led to something different before allowing people to communicate instantaneously, which does not make sense. Option C suggests that instantaneous communication is in addition to the explosion of digital media, rather than an effect. Option D is the only option that accurately connects the ideas in sentences 6 and 7 to show that the explosion of digital media has contributed to the ability of people to instantaneously communicate. 3. (C) All the sentences should have a main verb in the past tense. For Option A (sentence 1), Option B (sentence 2), and Option D (sentence 4), the main verbs in the sentences are in the past tense (“spent,” “did,” “recited”). For Option C (sentence 3), the main verb in the sentence inappropriately shifts to the present tense (“studies”) and should be revised to the past tense (“studied”). 4. (F) The sentence in the box needs a comma to set off the nonrestrictive clause “which is considered one of the New Seven Wonders of the World.” Option E would remove the comma between a city and country, which would be incorrect. Option G would incorrectly remove the comma at the end of a nonrestrictive clause, which is set off by commas at the beginning and the end. Option H is incorrect because a comma is not necessary before the conjunction “and” to connect a dependent clause. Option F is the only option that places a comma where it is needed, after “Italy,” to set off the nonrestrictive clause that follows. 8. (H) Sentence 9 expresses the idea that the library helps community members meet the need for human connection and companionship. Option E references the history of the library, which does not support the ideas in sentence 9 or the main argument in the passage. Option F introduces the idea of community meetings, but it does not support the idea in sentence 9. Option G mentions an expectation of the community, but it does not support the ideas in sentence 9, and it brings up a new idea unrelated to the main claim in the passage. Option H is the only option that provides support for sentence 9 and strengthens the connection between the information in sentence 9 and the main argument in the passage by listing examples of how people can connect at the library. 5. (B) The question asks for the most precise revision for the words The engineers tried some other things. Option A and Option C use the word “materials” rather than precisely identifying what the engineers used. Option D identifies the materials, but the imprecise verb “worked with” does not specify what the engineers were doing. Option B is the only option that revises the words to be precise by using the specific words “tested” for the verb and “foam and fiberglass” for the materials. 127 SAMPLE TEST – ENGLISH LANGUAGE ARTS EXPLANATIONS OF CORRECT ANSWERS continued... 9. (B) Sentence 11 provides an example of a way that a person can use the library, and sentence 12 offers another way. Option A suggests that sentence 12 is an example of the idea in sentence 11. Option C suggests that sentence 12 confirms the point made in sentence 11. Option D suggests that sentence 12 is a result of sentence 11. Option B is the only option that demonstrates that sentence 12 provides a similar example of a person who can use the library. Form B to the argument of the passage, which is that the library promotes a sense of community and offers assistance. This sentence should be deleted. 13. (B) The question asks for a concluding sentence that would follow sentence 21 to support the argument presented in the passage. Option A does not provide a logical conclusion to support the argument that the public library is a valuable resource because the sentence addresses the patrons of the library more than the function of the library. Option C presents the idea that the library has been in existence since 1833, but it does not logically support the main claim of the passage as a concluding sentence because it focuses only on this one idea. Option D states a new claim, that community leaders must work together to support library events, instead of a claim supporting the argument of the passage. Option B is correct because it logically follows sentence 21 and supports the argument that the public library is a valuable part of a community by urging community members to use, maintain, and support their local public library. 10. (E) The question asks for a revision of sentence 14 that best maintains the formal style established in the passage. Both Option F and Option H use the second person and directly address the reader, which is more informal. Also, Option H uses the informal phrase “whatever you need to do.” Option G does not use second person, but it uses informal language like “work on” and “things.” Option E is the only option that maintains the formal style by using “people” instead of “you” and the more formal language “resources available” and “accomplish many tasks.” 11. (C) The question asks for where sentence 19 should be moved in order to improve the organization of the fourth paragraph. Sentence 19 describes a typical librarian and the direction he or she provides in the library. Option A places this sentence before the main idea of the paragraph, which states that public libraries offer a variety of resources. Sentence 19 is an example of one of the resources, so it would not be placed at the beginning of the paragraph before the main idea. Option B places the sentence before the librarian is introduced in the paragraph. Both sentences 17 and 18 (Option D) have moved on from the topic of the librarian, so sentence 19 would not logically fit between them. Option C correctly places sentence 19 after the librarian is introduced in sentence 16, with an example of what else the librarian does. Moving Through Mountains 14. (E) Sentence 4 is the reason for the situation described in sentence 5. In Option F, the phrase “even though” changes the relationship between ideas so that the sentence illogically implies that needing to navigate around and through the mountains should have made transportation easy. Option G is incorrect because it states that traveling around the mountains or through winding tunnels emphasized that transporting people and goods was difficult rather than clarifying that this difficulty was an effect of having to travel around the mountains or through winding tunnels. Option H reverses the relationship between the ideas. Only Option E makes it clear that the transportation of people and goods was difficult because roads and railways had to navigate around and through the mountains. 12. (G) The question asks for a sentence that is irrelevant to the argument presented in the passage. Option E is relevant because sentence 3 makes reference to the library being one of the oldest yet best resources. Option F is relevant because sentence 11 provides an example of how a person can get assistance at the library, the main idea of the third paragraph. Option H is relevant because sentence 20 emphasizes the important idea that libraries are accessible. Option G, while related to the topic of the library because it states the selection of materials available, does not contribute 15. (C) The correct answer should state the main topic of the passage. Option A offers overly detailed information about the funding to build the Gotthard Base Tunnel, but it does not explain what or where it is. Option B gives details about the opening of the tunnel but does not provide a description of the tunnel. Option D offers a result of the Gotthard Base Tunnel, rather than a description of what and where it is. Only Option C specifically states and clarifies the Gotthard Base Tunnel as the main topic. 128 SAMPLE TEST – ENGLISH LANGUAGE ARTS EXPLANATIONS OF CORRECT ANSWERS continued... 16. (F) The correct answer needs to provide details about the tunnel-boring machines used to build the Gotthard Base Tunnel to supplement the general description of the machines in sentence 8. Option E offers information about how tunnel-boring machines were an improvement, which does not explain how the machines work. Option G is incorrect because it describes how the tunnel could not be completed until advances were made in tunnel-boring machine technology and does not describe the machines used for this tunnel. Option H explains that there are different types of machines for different geologies, but the geology of the tunnel area is not discussed in sentence 8 or the rest of the paragraph and the sentence does not explain how the machines work. Only Option F gives specific details about the tunnel-boring machines used to create the Gotthard Base Tunnel. Form B 19. (D) The correct answer must be a sentence that is irrelevant to the topic. Sentence 3 (Option A) introduces the obstacle that the Gotthard Base Tunnel was built to overcome, so sentence 3 should not be removed. Sentence 11 (Option B) should not be removed because the sentence helps the reader understand that building the tunnel was a massive job. Sentence 13 (Option C) helps the reader understand how the tunnel will improve transportation through the Alps, so it should not be removed. However, the information about the Channel Tunnel in sentence 17 (Option D) does not help the reader understand the Gotthard Base Tunnel, so sentence 17 should be removed. 20. (F) The correct answer needs to support key points from earlier in the passage. Option E would not be an effective concluding sentence because the economies of surrounding areas were never mentioned in the passage. Option G might seem attractive because building the Gotthard Base Tunnel appears to have required many people to work together. However, the passage does not explicitly mention people or groups working together. Option H would not be an effective concluding sentence because it focuses on the cost of the Gotthard Base Tunnel, which is referred to in only sentence 11 of the passage. Only Option F, which supports key elements from the introductory paragraph, would make an effective concluding sentence. 17. (D) The correct answer must be a location in which the sentence completes the detailed steps of how the tunnel was built. Option A, placing the sentence at the beginning of the paragraph (before sentence 7), would not make sense because the topic has not been introduced. Similarly, Option B, placing the sentence after sentence 7, would not make sense because adding concrete would have to happen after the rock was broken down and removed from the tunnel. Similarly, Option C, placing the sentence between sentences 9 and 10, would not make sense because the use of concrete did not take place before the removal of “28 million tons of rock.” Option D, placing the sentence between sentences 10 and 11, is correct because placing the sentence there helps the reader understand the full sequence of steps in constructing the tunnel before the cost of the project is introduced. READING COMPREHENSION Debates 21. (C) The passage is primarily about the participants in the first televised presidential debate (lines 10−24), details about how both Nixon and Kennedy acted during the debate (lines 25−47), and the impact of the debate (lines 47−61). All of these details are best stated by Option C. The reasons Nixon was expected to win (Option A), the discussion of domestic issues (Option B), and the qualifications of each candidate (Option D) are important details related to the debates but are not the overall topic. 18. (G) The correct answer must be a transition that clarifies the relationship between the ideas in sentences 14 and 15. Option E, “even so,” suggests that travel between Zurich and Milan would not be affected by the tunnel. Option F may seem attractive because “additionally” is used to show that sentence 15 gives a new idea related to the earlier one in sentence 14. However, sentence 15 doesn’t add an idea related to faster travel time. Option H, “therefore,” is used to show that one idea or event is the result of another. However, the one-hour reduction in travel time from Zurich to Milan mentioned in sentence 15 is not a result of travel being faster from northern to southern Europe but a detail to show one trip that will be faster. Only Option G, “for example,” correctly shows that the reduced travel time from Zurich to Milan is an example of faster travel between northern and southern Europe. 22. (F) Some observers suggested that had the debate not been on television, it is more likely that Nixon, not Kennedy, would have won the election (lines 64−67), which is Option F. The passage states that Nixon was well known, ahead in the polls, and more experienced than Kennedy (lines 16−20), so it is unlikely that Kennedy would have won anyway (Option E) or that the 129 SAMPLE TEST – ENGLISH LANGUAGE ARTS EXPLANATIONS OF CORRECT ANSWERS continued... Form B 26. (F) The answer is given in lines 3–9, which explains that the debates of 1960 were the first time presidential debates were broadcast on television and reached a greater audience than ever before, including people who were not interested in politics. This is summarized in Option F. Option E is a true statement, but it does not explain why people not interested in politics would have watched the debates. The text does not indicate that Kennedy’s youth and charisma were the main reasons people watched the debates, ruling out Option G. The idea that people wanted to verify the newspaper polls (Option H) is not supported by the passage. election results would have been closer (Option G). The idea that the debates would not have become a tradition (Option H) is not supported by the passage. 23. (C) The answer to this question can be found in the fourth paragraph. Lines 57–59 state that “Some feared that the better TV performer was bound to come across as being the better candidate.” This concern is best reflected in Option C. The less well-known candidates have to encounter more experienced candidates whether the political debates are televised or not, which rules out Option A. While certain candidates may be uncomfortable on live television, this was not the main concern described in the fourth paragraph, ruling out Option B. Multiple debates were planned before the first debate took place, so the idea in Option D would not be considered an undesirable consequence of televised debates. Ice 27. (B) The passage begins by asking why ice is slippery (lines 2–5) and then reviews several theories of slipperiness: smoothness, friction, pressure, and Faraday’s theory. Option B best states the topic of the passage. The circumstances that allow ice to melt (Option A), the discoveries of Michael Faraday (Option C), and the processes of freezing and melting (Option D) are details related to the theories of ice slipperiness, but they do not express the main idea of the passage. 24. (F) Details in lines 49−52 show that the televised debates benefitted Kennedy by allowing him to display his “charm, poise, and confident manner,” which were favorable characteristics that convinced viewers he had the maturity to be president. This benefit is summarized by Option F. Option E is incorrect because it can be inferred that both candidates appeared to have an equal understanding of domestic issues since they are described as having “dealt with the issues calmly and carefully” (lines 31–33). Both Nixon and Kennedy took questions from reporters (lines 29−31), but there is no indication that Kennedy had a better relationship with reporters than Nixon did, which rules out Option G. The debate may have persuaded some viewers to agree with Kennedy’s positons on domestic issues (Option H), but the passage does not support that this was an advantage for Kennedy because of the debates being televised. 28. (E) The most likely reason that the author mentions lead and diamond crystals (lines 70–71) is to illustrate that solids other than ice have slippery surfaces and indicate that other substances can contain this property under the right circumstances (lines 72−73). This is best stated in Option E. Option F cannot be correct, because these crystals are not made of frozen water. The properties of lead and diamond crystals are not related to Faraday’s theory, ruling out Option G. While lead and diamond crystals may demonstrate the slipperiness property under certain temperature and pressure conditions (Option H), the author does not list these substances in order to show the effects of temperature and pressure on their slipperiness. 25. (D) The detail in Option D supports the last sentence of the passage: Nixon’s greater experience (lines 16–20) may have been more apparent to radio listeners who would not have been distracted by his poor television appearance. While lines 53−54 state that fewer people watched the later debates (Option A), this does not support the idea that Nixon would have won had the debates not been televised. The idea that both Nixon and Kennedy responded to questions calmly and carefully (lines 31−33) also does not indicate that Nixon would have won, ruling out Option B. While Nixon was ill just before the debate (lines 40−41), this does not support the idea that Nixon was the favorite to win before the debate, ruling out Option C. 29. (C) According to Faraday, the liquid on the ice cubes’ surfaces freezes solid when the surfaces make contact (lines 42–45). This information is restated in Option C. Option A is incorrect because Faraday’s explanation does not include the concept of friction. Neither the pressure theory, described in the third paragraph, nor Faraday’s theory, described in lines 35–42, support the idea that applied pressure forces ice cubes to stick together, ruling out Option B. Smoothness (Option D) was a discredited explanation for slipperiness, not for why two ice cubes fuse when held together. 130 SAMPLE TEST – ENGLISH LANGUAGE ARTS EXPLANATIONS OF CORRECT ANSWERS continued... Form B Marsh 30. (E) The experiment at Lawrence Berkeley Laboratory is mentioned in lines 51–57. The author most likely mentions the experiment because the data provide evidence that the ice surface remains liquid-like, creating a slippery layer of molecules on the ice surface (Option E). In the past, studying ice molecules was impossible (lines 46−48), but the experiment at Lawrence Berkeley Laboratory does not indicate the difficulty of studying ice molecules presently, ruling out Option F. Option G is incorrect because the experiment described in the fifth paragraph did lead to the conclusion that the molecules vibrated only up and down (lines 63−64). Ice cubes freezing together refers to Faraday’s observations in lines 41−42 (Option H), but the experiment at the lab in 1996 is included to present observations about ice on a molecular level. 34. (H) The second through fifth paragraphs of the passage primarily describe Marsh’s personal experiences and his ideas about nature, and the final paragraph explains how his ideas are the basis for the conservation movement. This is best stated in Option H. Option E focuses mainly on Marsh’s early life and does not address his influence. Option F describes specific details about Marsh’s beliefs but does not explain who he was or how he affected the conservation movement. Option G emphasizes that Marsh’s ideas were radical and influential but does not provide any information about what Marsh’s beliefs were. 35. (D) Marsh attributed people’s practices to “the popular belief that nature could heal any damage that people inflict upon it” (lines 43–45), which suggests a lack of understanding, or ignorance, of nature (lines 31–32). This is best stated in Option D. Although future generations are working to solve environmental problems, there is no indication in the passage that Marsh believed that people in his time expected future generations to solve environmental problems, ruling out Option A. While the people of Marsh’s time made advances in industry, Marsh did not indicate that he believed that people thought industrial progress outweighed efforts to protect the environment (Option B). Option C is incorrect because there is also no evidence in the passage to suggest that Marsh believed that people were unwilling to change their practices. 31. (B) The distinction between the two terms is made in lines 63–65. The surface of ice is liquid-like because the surface molecules move only up and down, which is Option B. Option A describes the results of the experiment, not the ice surface itself. The passage does not state that the liquid-like state is related to temperature (Option C) or pressure (Option D). 32. (H) The friction theory of slipperiness is explained in the second paragraph, which concludes that the theory cannot explain why ice is slippery for someone who stands motionless, essentially creating no friction. Something that a theory cannot explain can be said to weaken, or undermine, the theory. Option H, “a person slipping while standing immobile on an icy surface,” is the best answer. Option E undermines the pressure theory, not the friction theory. Option F and Option G neither support nor undermine the friction theory. 36. (F) Lines 1–9 lead the reader to expect that Marsh was part of the modern environmental movement that began in the 1960s. Therefore, the fact that Marsh’s influential book was published 100 years earlier is surprising (Option F). While Marsh’s ideas have had a resurgence in popularity since the 1960s (lines 7–9), Marsh’s observations about deforestation in Vermont (lines 16–19) and land mismanagement in Italy (lines 25–30) indicate that his ideas were just as applicable in his time as they are today, ruling out Option E. While Marsh could not have been aware that his ideas would lead to the start of a conservation movement (Option G), line 12 indicates that the surprising part is that Marsh had these ideas 100 years before the conservation movement became popular. While it’s possible a greater awareness of human impact on the environment during Marsh’s time could have prevented certain environmental issues today (Option H), this is conjecture and does not explain the author’s purpose for using the word “surprisingly.” 33. (A) The author includes the information about the pressure theory to highlight that there are different perspectives on the validity of the pressure theory between what is presented in textbooks and what researchers believe to be true (lines 25−29). This is best stated in Option A. Option B is incorrect because the liquid-like theory, not the pressure theory, has gained more acceptance recently. Even if scientists have tested the pressure theory (Option C), the pressure theory still raises questions about why ice is still slippery when less pressure is applied (lines 29−34). There is scientific support for a plausible theory (Option D), but that support is for Faraday’s theory, not the pressure theory. 131 SAMPLE TEST – ENGLISH LANGUAGE ARTS EXPLANATIONS OF CORRECT ANSWERS continued... 37. (C) The concept that Marsh’s theories about nature were accurate is best supported by the statement that ideas from his book are now considered basic knowledge in the field of environmental science (lines 51–54), which is Option C. While Marsh made observations of environmental degradation (lines 13–22, lines 25–27), this does not provide evidence that his theories were accurate, ruling out Option A. While Marsh’s writing did inspire a conservation movement (lines 77–80), these details do not call attention to the accuracy of his ideas, which rules out Option B. Option D is incorrect because it refers to personal opinions (lines 58–59, lines 67–70), not his theories, which are considered true today. Form B electricity is mentioned in lines 54–56, but this detail is explained only at the end of the passage, ruling out Option H. 41. (B) The idea that windmills were an important resource in the western United States is best supported by the information about the use of windmills to pump water for farms and livestock in lines 44–45 (“essential role in pumping water”) and lines 47–50 (“over six million small windmills were installed . . . for watering livestock and meeting other water needs”). This is best stated in Option B. The details about the development and use of wind farms in lines 68–71 (Option A) support the idea of wind being an alternative energy source but are not limited to a specific region. The energy crisis of 1970 (Option C) was not limited to the western United States, and steam power (Option D) led to a decline in the use of windmills. 38. (H) Details about Marsh’s approval of the Suez Canal (lines 63–67) show that Marsh did not oppose certain human activates because the advantages—improved transportation and commerce—improved human life and outweighed negative environmental damage (lines 67–70). Option H best summarizes that idea. Option E is not relevant to the statement in question. Option F acknowledges Marsh’s contributions to the environmental movement but does not relate to his attitudes about certain alterations to the environment. Option G is incorrect because it relates to a time when Marsh observed environmental degradation in a foreign country. 42. (E) The author describes the different tasks windmills were used for mainly to emphasize that windmills and wind energy can be used to serve a variety of purposes and were important in the production of goods before steam power was harnessed. This purpose is best stated in Option E. While windmills did replace water wheels (lines 26–31), this is not the idea the author is emphasizing by including a list of the tasks windmills were adapted to perform, ruling out Option F. The passage does explain that windmills had been used for many years (lines 15–23), but this idea is not highlighted by the list of tasks, which rules out Option G. Option H is incorrect because it relates to the idea that Holland was famous for its windmills (line 32), which is not proved or emphasized by the tasks listed in lines 36–40. 39. (C) Marsh’s main contribution to the modern environmental movement is given in lines 5–7—the idea that Western society was causing irreparable harm to the environment. Option C restates that idea. While Marsh did believe that some human alterations to the environment are necessary (lines 67–70; Option A), that people lacked an understanding of nature (lines 30–32; Option B), and that environmental degradation had been occurring for many years (lines 27–30; Option D), it is clear from the information in the first paragraph that the impact of human activity was his most influential idea. 43. (C) The need for wind machines to produce electricity on American farms before the 1950s is discussed in lines 54–59. The next two sentences explain that the need for windmills decreased in the 1950s because most homes were connected to an electric utility and no longer depended on windmills for electrical power (Option C). The energy crisis happened in the 1970s and prompted a renewed interest in wind energy, which rules out Option A. The idea that coal and natural gas generate more power than wind turbines (Option B) is not supported in the passage as the main reason for the decline in the use of wind machines in the 1950s. Electrical connectivity, not wind farms, reduced the need for individual windmills (Option D). Wind Energy 40. (E) Option E best describes what the passage is about, which is how wind energy has been used in a variety of ways from ancient sailboats to medieval windmills to modern turbines. Wind farms are a more modern development, and they are mentioned only in lines 68–74, which rules out Option F as the main topic of the passage. Option G is incorrect because the second and third paragraphs explain in detail about windmills that were used in other areas of the world, not just the United States. Developing windmills to generate 132 SAMPLE TEST – ENGLISH LANGUAGE ARTS EXPLANATIONS OF CORRECT ANSWERS continued... 44. (H) The author’s opinion regarding the future use of wind energy is discussed in the last paragraph: “wind farms may prove to be as important in the future as earlier windmills were in the past” (lines 72–74). Option H best conveys the author’s optimism that wind farms will be a major source of electricity in the future. Option E is incorrect, given the author’s optimism. Option F and Option G relate to the present state of wind farming more than the author’s vision of the future. Form B 49. (B) Tree-ring dating helped reveal that the Pueblo villages were likely abandoned during a long drought (Option B). Option C first led archaeologists to realize that the villages had been abandoned. Option A and Option D contributed to establishing the chronology but do not explain why the villages were abandoned. 50. (E) The third paragraph describes the development of a floating chronology that did not overlap the established chronology that went back to  1260. This implies that the years of the floating chronology preceded the years of the established chronology, which is stated in Option E. Option F is incorrect because the pattern of tree-rings in the logs reveals more about the chronology than the number of tree-rings does. Archaeologists knew that the Pueblo villages were abandoned before the Hopi villages were established because the Hopi villages had been continuously inhabited (lines 31–33), ruling out Option G. Option H is incorrect because the size of the logs does not provide information about their connection to the established chronology. 45. (B) The country of Holland most notably used windmills to pump seawater away from bogs and reclaim large areas of land (lines 32–36), as stated in Option B. The pumping of water to remote locations (Option A) was more important in the United States than in Holland. While Holland is famous for its windmills (line 32), the passage clearly states that windmills were most notably used for practical purposes, such as clearing bogs (lines 33–34) or processing goods (line 39), which rules out Option C. The use of windmills to process goods was not limited to or most notably done by the Dutch, which rules out Option D. 51. (C) Lines 41–44 suggest that archaeologists compared samples from both villages in hopes of finding a beam where the patterns matched (Option C). The discovery of the key beam is described later in the passage. Option A, Option B, and Option D are related to information that can be gathered from examining the logs, but the options do not accurately explain the archaeologists’ purpose of finding a key beam. Pueblo 46. (H) Option E cannot be the topic of the passage because only the first paragraph discusses how weather conditions affect tree growth. Option F is too broad to be a good statement of the topic. The reason for the abandonment of the Pueblo villages (Option G) is mentioned only in the last paragraph, so it is not the main topic. Option H offers the best statement of the topic of the passage. American Scene 52. (H) The correct answer is found in lines 65–68; the general public identified with American Scene art because the paintings presented common images and mirrored the lives of many people, which is best stated in Option H. While farm life was depicted in a subset of American Scene art (Regionalist art; lines 16−32), this is not an explanation of why people identified with the art, which rules out Option E. Option G is incorrect because American Scene art primarily focused on painting the changing United States as the artists saw it (lines 11−15). Option F may seem true, but the emphasis of the passage is on the relatability of the scenes depicted in American Scene art, not the beauty. 47. (D) Option A is true but can be proved without a key beam. Option B is not related to the key beam. Option C misrepresents the importance of the key beam. Option D is correct. The key beam, with its overlapping ring patterns from the established and floating chronologies, allowed archaeologists to connect the two chronologies. 48. (F) The size of the tree rings provides information about the health of the tree and insight about environmental factors and weather patterns (Option F). The fact that archaeologists rely on tree-ring dating suggests that it is accurate (Option E), but this does not explain why the author describes ring widths. Option G is incorrect because the passage explains that trees that live in a specific place at the same time will have the same tree-ring pattern. Option H is incorrect because it does not offer an explanation about the conclusions that can be made from the size of tree rings. 53. (A) According to the third paragraph, Urban Realists “painted the drab realities of the contemporary urban environment” (lines 34–36) and depicted “the high price paid by individual men and women struggling to survive the Depression” (lines 41–43). The subject that 133 SAMPLE TEST – ENGLISH LANGUAGE ARTS EXPLANATIONS OF CORRECT ANSWERS continued... best matches that description is Option A. Option B and Option D may be set in an urban environment, but they do not fit the description of Urban Realist art given in the passage. Option C is incorrect because it does not describe an American city scene. 54. (G) Lines 49–53 provide the correct answer. Edward Hopper, the painter of Nighthawks, portrayed dingy urban streets; however, he often found beauty in the midst of a city’s drab surroundings, which is expressed in Option G. Option E and Option H do not state clear contrasts to the Urban Realist style. The international style had not yet developed, ruling out Option F. 55. (B) The fourth paragraph is about Edward Hopper and his association with American Scene art. The paragraph highlights that Hopper’s work was unique and hard to classify because of the way he found beauty in otherwise drab urban settings (lines 49–53). This is best stated in Option B. The end of the American Scene movement is described in the fifth paragraph, not the fourth, ruling out Option A. Option C is incorrect because Hopper’s work has been remembered. Option D is incorrect because the fourth paragraph focuses on Hopper’s contribution to the art style of the time. Additionally, Urban Realist style is a subcategory of American Scene style, and most typically contrasted with Regionalist art (lines 33–37). 56. (E) The phrase “without apology” (lines 59−60) refers to how Hopper saw and portrayed his subjects. In his art, he presented people and places as he saw them, as in his painting Nighthawks (lines 55−57). This is described in Option E. While Hopper did paint scenes of real places (Option F), the phrase is about how he chose to portray those scenes in his work. Hopper’s works became well known (Option G), but the phrase does not relate to the popularity of his work. The passage says that Hopper “escaped further classification” (line 49), but the phrase does not relate to the classification of his art style, which rules out Option H. 57. (C) Regionalist art, with its pleasant and familiar subjects, retained some of its popularity because it showed American life as people wished to remember it (lines 8−11, lines 19−23). This is best stated in Option C. It did not retain some of its popularity because it portrayed modern life (Option A) or the time in which it was painted (Option B). Regionalist art did not depict Americans overcoming the Depression or life during World War II, ruling out Option D. 134 Form B SAMPLE TEST – MATHEMATICS EXPLANATIONS OF CORRECT ANSWERS 58. (120) There are 5 choices for the first digit, 4 choices for the second digit, 3 choices for the third digit, 2 choices for the fourth digit, and 1 choice for the final digit. The total number of possibilities is 5 × 4 × 3 × 2 × 1 = 120. In parallelogram PQRS, angle QPT is congruent to angle QRS, so the measure of angle QRS is also 60°. 65. (D) Break the equations apart to each equal M: 147 − x = 144 M = 3N x=3 M = P 4 60. (–3.4) (−6) − (−5) + 4.2 − 3 − 9.6 = 3.2 − −6.6 = 3.2 − 6.6 = −3.4 M = Q+5 M = R 7 61. (300) Let x be the total number of pages in the workbook. Then, 20% of x is 60. Set up a proportion and solve for x: 20 = 60 100 x Pick a number to substitute into the equations, and solve the equations to find the values of M, N, P, Q, and R. Let M = 2. Since all the equations are equal to 2, substitute 2 to find each variable. 20x = 6,000 6,000 = 20 BB 64. (E) Find the missing angle, angle QPT, of triangle PQT: 180° − 70° − 50° = 60° 59. (3) 147− x = 12 12 x = FORM 300 pages M = 3N 2 = 3N 62. (65) Call the missing angle in the top half of the figure x. The sum of the four angles on the top of the figure is equal to 180°. x + y + 30 + 60 = 180 2 = N 3 M = P 4 Since x is a vertical angle with the 25° angle, then x is also 25°. Use that to solve for y. 2= P 4 25 + y + 30 + 60 = 180 8= P y + 115 = 180 M = Q+5 y = 65 2= Q+5 −3 = Q 63. (D) x( x − 2 y) = 9[ 9 − 2(−7)] = 9(9 + 14 ) = 9(23) = 207 M = R 7 R 2= 7 14 = R Variable R has the greatest value. 135 SAMPLE TEST – MATHEMATICS EXPLANATIONS OF CORRECT ANSWERS continued... 66. (G) Set up a proportion: 96x = 1,248 3 = 10 30%. 71. (C) Since 18 and 24 are both multiples of 6, find the least common multiple of only 18 and 24. 67. (C) Set up an inequality to compare the costs: 0.15x ≤ 10.50 Multiples of 18: 18, 36, 54, 72… x ≤ 70 Multiples of 24: 24, 48, 72… Therefore, 70 individual sheets of paper would cost $10.50, so 69 is the greatest number of individual sheets of paper that Macie can buy that would be less expensive than the package. The least common multiple of 6, 18, and 24 is 72. 72. (F) Let x be the number of dozens of eggs for 300 customers. Set up a proportion: x = 15 300 200 68. (F) 7:00 p.m. is 6 hours after 1:00 p.m. Calculate the number of degrees the temperature dropped in 6 hours: 3 × 6 = 18  degrees. 200x = 4500 x = 22.5 dozen eggs. Subtract that from the starting point (8 degrees) to find the solution: 8 − 18 = −10 degrees. Round up to 23 because you can't have half an egg. 73. (C) The total number of bottles of juice in the cooler is 5 + 3 + 6 = 14. 69. (D) The ratio of red to blue to green is 15:7:3. Find the proportion of blue marbles. Add the The number of bottles of juice that are not apple juice (grape juice and fruit punch) is 3 + 6 = 9. So the probability is 9 . 14 numbers of the ratio and use the total sum as 7 = 7 . Find the 15+7+3 25 3 proportion of green marbles: . Since there 25 are a total of 75 marbles, the number of blue the denominator: marbles is 7 × 75 = 21. The number of 25 green marbles is 3 × 75 = 9. The number 25 of red marbles is 75 − 21 − 9 = 45. B The probability that ice cream was chosen is 48 = 160 x = 13 bundles Form 70. (F) The total number of desserts ordered is 42 + 23 + 47 + 48 = 160. x = 3 416 96 If 2 blue marbles are removed and replaced with 2 green marbles, the number of blue marbles is now 19 and the number of green marbles is now 11. The ratio of red marbles to green marbles is 45:11. 136 SAMPLE TEST – MATHEMATICS EXPLANATIONS OF CORRECT ANSWERS continued... 74. (H) The radius of the large plate is 20 cm. Use that to find the area of the large plate: 36 = 4 x 3 The circumference of the smaller plate is 20 π cm. Use that to find the radius, and then the area, of the smaller plate: 12 = 4 x C = 2πr 3= x 20π = 2πr 78. (H) Since XY = 20 cm, use that to find YZ : r = 10 YZ = 3 XY = 3 ( 20) = 12 cm 5 5 XZ = XY + YZ = 20 + 12 = 32 cm 2 2 A = πr = π(10 ) = 100π sq cm Subtract the area of the small plate from the area of the large plate: 400π − 100π = 300π sq cm 79. (C) Calculate the cost of the cloth before tax: 1 3 × 8 = 7 × 8 = $14 4 4 75. (D) The question says that an equal number (x) of each type of space was purchased. To find the number of each type of space that was purchased, multiply the price per type by x and set it equal to the total amount spent, then solve for x: Now find the tax for $14 worth of cloth: 14 × 8% = 14 × 8 = $1.12 100 200x + 350x + 600x = 11,500 Finally, add the cost of the fabric and the tax: $14 + $1.12 = $15.12 1,500x = 11,500 x = 10 B 36 = 4 x y A = πr = π( 20 2 ) = 400π sq cm Form 77. (A) Substitute 3 for y and solve for x: 2 80. (F) To find M, subtract N − M and set it equal to the length: Thus, the store purchased 10 units of each type of space. To find the total amount of page space purchased, multiply each type of space by 10, and add: 11 −M = 55 3 6 5 −M = 5 − 1 1 6 3 5 −M = 5 − 1 2 6 6 3 −M = 4 6 1 M = −4 2 (10 × 14 page) + (10 × 12 page) + (10 × 1 page) = 17 1 pages 2 76. (E) In the second quadrant, where point R is located, the x-values are negative and the y-values are positive. Any point in the second quadrant will have a y-value greater than the x-value. So, the answer is point R. 137 SAMPLE TEST – MATHEMATICS EXPLANATIONS OF CORRECT ANSWERS continued... 81. (B) Add the four values in the ratio (177 + 12 + 7 + 4 = 200) and use the sum as the denominator. Use that to find the fraction of zinc in one of the coins. Then reduce the fraction: Form B 85. (C) Let x be the original amount of the chemical. It loses 20% after each week, which means 80% of the chemical remains at the end of each week. 12 = 3 200 50 Multiply this fraction by 8 to find the number of grams of zinc in decimal form: End of first week: 0.80x At the end of the second week, 80% of the amount left at the end of the first week remains. End of second week: 0.80(0.80x) = 0.64 x or 64% 3 × 8 = 24 = 0.48 g 50 50 86. (G) One more than an odd integer must be even. 82. (G) Jack scored a mean of 15 points per game in each of the first 3 games, so he earned a total of 45 points for the first 3 games. Use that information to calculate the mean over the four games: Therefore, 2w − 2 must be even. 45 + 27 = 72 = 18 4 4 87. (B) Find the least common multiple of 2, 3, and 4 — which is 12. So, it takes 12 minutes before all three are back at the starting line. Ann completes 1 lap every 2 minutes, so in 12 minutes she has completed 6 laps. 83. (B) Find the number of liters that need to be added. Since 1 of the oil drum is full, 2 of 3 3 the drum remains empty: 88. (F) 4( 7 − 3x) − ( 5 − x) = 28 − 12x − 5 + x = 23 − 11x 2 × 4,320 = 2,880 liters 3 Use the conversion 1 kL = 1,000 L to find the number of kL: 89. (D) First, add the number of students for each category to find out how many total students were in the survey: 12 + 16 + 7 + 5 = 40 2,880 = 2.88 kL 1,000 The number of students who had at least 2 pets are the ones who have 2 pets (7) plus the ones who have 3 or more (5). The total number of students with at least 2 pets is 7 + 5 = 12. The probability of a student in the survey having at least two pets is: 84. (F) First, find out how old Nicole and Carmen are now. Let N = Nicole’s age now. Let C = Carmen’s age now. C + 2 = 17 12 = 3 40 10 , C = 15 (Carmen s age now) N = 3C = N 3(15) = One more than w − 1 is w, therefore w must be even. Two times an even integer must be even, therefore 2w is even. An even integer decreased by 2 must be even. 45 (Nicole’s age now) N − 5 = 45 − 5 = 40 (Nicole’s age 5 years ago) 138 SAMPLE TEST – MATHEMATICS EXPLANATIONS OF CORRECT ANSWERS continued... 90. (E) Let x be the total number of liters the container can hold. First, solve each equation for x: Equation 1: TM, MR, RN, and NS are all equal to 3 cm. Triangles TMR and RNS are both right triangles, so the area of one of the triangles is 1 × 3 × 3 = 9 . The triangles are congruent, 2 2 so the sum of the areas is 9 + 9 = 9 sq cm. 2 2 n +10 = 3 x 5 3x = 5( n + 10) x = 5n +50 3 93. (D) Let x be the amount spent on planned expense in one year: x = 5 29,600 8 5 x = ( 29,600) = 18,500 8 Equation 2: n +16 = 75 x 100 n +16 = 3 x 4 3x = 4( n + 16) x = 4n +64 3 B cm. Since R, S, and T are midpoints, then n +10 = 60 x 100 Form 92. (E) The length of one side of the square is 6 n +10 = 60% and n +16 = 75% x x 94. (G) First, figure out how many different topping pairs are possible. Use 1, 2, 3, 4, 5, 6, 7 to represent the toppings and create a list of possible pairs: Now, set the two equations equal to each other and solve for n. 1,2; 1,3; 1,4; 1,5; 1,6; 1,7 2,3; 2,4; 2,5; 2,6; 2,7 3,4; 3,5; 3,6; 3,7 4,5; 4,6; 4,7 5,6; 5,7 6,7 So there are 21 different topping combinations for one pizza. Since there are 3 pizza sizes, multiply the total number of combinations by 3 to get the total number of different pizzas Cody can create: 3 × 21 = 63. 5n +50 = 4n +64 3 3 5n + 50 = 4n + 64 n + 50 = 64 n = 14 liters 3 91. (B) 5x + 3x + 9 + 1 2 x = 5(10 3 ) + 3(10) + 9 + 1 2 10 = 5,000 + 30 + 9 + 1 = 5,039.01 100 95. (D) To find by what percent the number of families with 1 cat is greater than the number of families with 2 cats, calculate the difference between the two numbers and divide by the number of families with 2 cats: 42−35 = 7 = 1 = 0.20 or 20%. 35 35 5 139 SAMPLE TEST – MATHEMATICS EXPLANATIONS OF CORRECT ANSWERS continued... 96. (H) One side of the square base is 3 ft long. Since the height of the box is 3 times the length, then the height is 3 × 3 = 9 ft. The volume of a rectangular prism is length × height × width. The volume of the wooden box is Form B 100. (E) Both x + 1 and y + 2 are radii. So, set them equal to each other and solve for y. y + 2 = x +1 y = x −1 V = 3 × 3 × 9 = 81 cu ft. 101. (C) There are 5 sections between M and T. To find the length of one of these sections, find the distance between M and T and divide by 5: 97. (B) Calculate each mean speed: = R 65 = 5 13 kph = S 72 = 4 18 kph ( 85 − (− 14 )) ÷ 15 = ( 85 + 82 ) ÷ 15 = Then calculate the difference of both mean speeds: 7 ÷ 5 = 7×1 = 7 8 1 8 5 40 S − R = 18 − 13 = 5 kph ( ) − 1 + 3 7 = − 10 + 21 = 11 4 40 40 40 40 98. (G) R is located at 11 . 40 102. (H) Let x be the number of minutes Phan used his internet service in the month. Phan’s monthly charges were 18 + 0.024 x. Since Deion’s charges were the same as Phan’s, set the expression equal to 30 and solve for x: Find the midpoint of PQ and RS: −1 − (−3) Midpoint of PQ = = 2 = 1 unit. 2 2 The midpoint of PQ is located 1 unit from each endpoint, so the midpoint is at −2. Midpoint of RS = 2 − 0 = 2 = 1 unit. 2 2 The midpoint of RS is located 1 unit from each endpoint, so the midpoint is at 1. 18 + 0.024 x = 30 0.024 x = 12 x = 500 The distance between the two midpoints is 1 − (−2) = 3 units. = = cu cm, then 1 L 1,000 mL. 99. (D) If 1 L 1,000 Set up a proportion, letting x = the amount of cubic millimeters in 1,000 cubic centimeters. 1,000 cu mm x cu mm = 1L 1,000 cu mm R is 3 sections away from M, so add: Solve for x: 1,000,000 cubic millimeters are in 1,000 cubic centimeters. 140 Phan used his service for 500 minutes. SAMPLE TEST – MATHEMATICS EXPLANATIONS OF CORRECT ANSWERS continued... 103. (B) Create a chart using the given information and use subtraction to figure out how many cars are not red and do not have a back-up camera: Back-up Camera Red Not Red TOTAL 4 6 (10-4) 10 TOTAL 38 (50-12) 12 Substitute y = 1 into each inequality and solve for x: 2x − 2 ≤ 1 2x ≤ 3 x≤ 3 2 50 1 ≤ 4 x + 10 −9 ≤ 4 x −9 ≤ x 4 both conditions from the total of 50 cars at the dealership is 32 = 16 . 50 25 6 . Simplify the fraction to find the 100  answer: 104. (E) 0.06 = 3 so, x = 50 B 2x − 2 ≤ y and y ≤ 4 x + 10 The probability of selecting a car that meet 6 = 100 Form 107. (C) Separate the compound inequality into two pieces: 32 (38-6) No back-up Camera The solution is the number line that shows that x is greater than or equal to −2 1 and 4 less than or equal to 1 1 . 2 108. (G) 14 = p 21 7 3. 21 p = 7(14) 105. (C) The height of the triangle is 4 units. The length of the base is n − m. So the area is 21 p = 98 p = A = 1 ( n − m)( 4 ) = 2( n − m). 2 98 = 21 14 3 109. (A) The total number of balls in the box is 7 + 14 + 21 = 42. 106. (F) The total number of cards in the box is 8 + 6 + 5 + 4 + 1 = 24. Set up a proportion The probability that the ball is black is 7 = 1. 42 6 to figure out which card has exactly a 1 in 4 chance of being picked at random. x = 1 24 4 or x = 6. The dog card has a 1 in 4 chance of being randomly selected. 141 SAMPLE TEST – MATHEMATICS EXPLANATIONS OF CORRECT ANSWERS continued... Form 113. (D) The shelf, when full, holds 36 cans. When the shelf is half full, it holds 18 cans. 110. (G) None of the 80 students (800 − 720) who answered “no” to Question A (800 − 720) could have answered “yes” to both questions. Therefore, the least possible number of students who could have answered “yes” to both questions, can be found by subtracting the 80 who answered “no” to Question A from the 640 who answered “yes” to Question B or 640 − 80 = 560. x − 4 = 18 x = 22 114. (G) The probability of the cup landing on its side is 72%. Carlos tossed the cup a total of 200 times (50 + 150). The number of times the cup lands on its side is 72% of 200: 0.72 × 200 = 144. 111. (A) Raoul is at least 3 years older than Vahn, which can be written as r ≥v+3 B Rewrite this inequality to match the answer options: r−v≥3 112. (F) Since 5.6 ricks and 12.88 dalts are both equal to 1 sind, then 5.6 ricks = 12.88 dalts. To calculate the number of dalts (d) in 1 rick, set up a proportion: 5.6 = 1 12.88 d 5.6d = 12.88 d = 2.3 Answer Key for Sample Form B 1. A 13. B 25. D 37. C 49. B 61. 300 73. C 85. C 97. B 109. A 2. G 14. E 26. F 38. H 50. E 62. 65 74. H 86. G 98. G 110. G 3. C 15. C 27. B 39. C 51. C 63. D 75. D 87. B 99. D 111. A 4. F 16. F 28. E 40. E 52. H 64. E 76. E 88. F 100. E 112. F 5. B 17. D 29. C 41. B 53. A 65. D 77. A 89. D 101. C 113. D 6. E 18. G 30. E 42. E 54. G 66. G 78. H 90. E 102. H 114. G 7. D 19. D 31. B 43. C 55. B 67. C 79. C 91. B 103. B 8. H 20. F 32. H 44. H 56. E 68. F 80. F 92. E 104. E 9. B 21. C 33. A 45. B 57. C 69. D 81. B 93. D 105. C 10. E 22. F 34. H 46. H 58. 120 70. F 82. G 94. G 106. F 11. C 23. C 35. D 47. D 59. 3 71. C 83. B 95. D 107. C 12. G 24. F 36. F 48. F 60. –3.4 72. F 84. F 96. H 108. G 142 SAMPLE PROBLEMS FOR GRADE 9 MATHEMATICS GRADE 99 DIRECTIONS: This section provides sample mathematics problems for the Grade 9 test forms. General direc- tions for how to answer math questions are located on pages 61 and 114. There is no sample answer sheet for this section; mark your answers directly on this page or on a separate piece of paper. 1. 4. Assume S( x ) equals the sum of all positive even integers less than or equal to x . What is the value of S( 7) ? 2. Simplify: p12  p0 − p 4 16  196 = E. 0 F. p G. p8 H. p16 −3 y 3. 5. 12 ft 8 6 M 4 ft 4 Water is pumped into a tank that is shaped like the right inverted cone shown above. The cone has a base diameter of 12 feet and a height of 4 feet. What is the volume, in cubic feet, of the water in the tank when the height of the water is 2 feet? 2 N O 2 4 6 8 x If MN is translated 1 unit to the left to produce M′ N′, what is the area of parallelogram NMM′ N′ ? A. B. C. D. A. 6  cu ft B. 18  cu ft 3 square units 4 square units 5 square units 6 square units C. 24  cu ft D. 48  cu ft 143 SAMPLE PROBLEMS FOR GRADE y 6. 9 MATHEMATICS 7. l (2a, 2b) continued... The graph shows the wolf population in Yellowstone National Park since 2000. A student drew a line of best fit to model the data. y (a, b) Number of Wolves O x Straight line l passes through the origin, as shown in the figure above. What is the slope of line l in terms of a  and b ? E. ​a ​ F. ​ 2b ​ G. ​2a ​ H. ​b ​ 180 170 160 150 140 130 120 110 100 90 80 0 b a Wolf Population 2 4 6 8 10 12 14 Years Since 2000 x Which statement best describes the line of best fit that the student drew? b A. The line of best fit is not a strong model for the data, because the points are not close to the line. B. The line of best fit is not a strong model for the data, because it does not pass through any of the data points. C. The line of best fit is a strong model for the data, because both the line and the data show a negative trend. D. The line of best fit is a strong model for the data, because about half the data points are on each side of the line. a 144 SAMPLE PROBLEMS FOR GRADE 8. 9 MATHEMATICS To determine the price of servicing a car, a mechanic charges a fixed fee plus an hourly rate for each hour he works. If his price for 4 hours of service is $270, and his price for 7 hours of work is $420, what is the fixed fee that the mechanic charges? E. F. G. H. continued... 11. Which of the following expressions is negative in value? A. 4 − B. 3 − 9 $50 $60 $70 $120 C. 12 − 4 D. 36 − 9 ​ y 9. 12. 4 Q 3 2 1 O P N R m 5c x S P 1 2 3 4 10 cm M x R  In the figure above, MPR is similar to NPQ . If the length of NQ is 10 centimeters, what is the length of MR in terms of x ?  Rectangle PQRS above is rotated 180° about the origin to form rectangle P′Q′ R′S′. What are the coordinates of R′ ? E. 2x A. ( 4, −3) F. 2x + 10 B. (−4, 3) C. (−4, 1) G. x + 5 D. (−4, −3) H. ​1 x + 5 2 −8 15.3 × 10 10. 1.5 × 10 Q 4 What is the quotient of the expression above, expressed in scientific notation? −13 E. 1.02 × 10 −11 F. 1.02 × 10 −4 G. 1.02 × 10 H. 1.02 × 1012 145 ​ SAMPLE PROBLEMS FOR GRADE 9 MATHEMATICS 13. 20 x y In the right triangle above, x = 2 y . What is the value of y ? A. 2 B. 10 C. 80 D. 200 146 continued... EXPLANATION OF CORRECT ANSWERS GRADE 9 MATHEMATICS 1. (12) S(x) is the sum of all positive even integers less than or equal to x. 1, 2, 3, 4, 5, and 6 are GRADE 6. (H) Use the slope formula to figure out the slope of line l. all integers less than 7. Take the positive integers from the list and find the sum: 3. (B) 16 = 196 • • 14 4= Slope of line l = 7. (A) The line of best fit should be close to as many points as possible. In this case, very few of the points are on or next to the line. So, this is not a strong model for the data, because most of the points are not close to the line. 56 When MN is translated 1 unit left, the distance between M′ and M is 1 unit, which is the base of the parallelogram. The height of the parallelogram is the vertical distance from M to N. Since M is at y = 5 and N is at y = 1, the height is 5 − 1 = 4 units. The area of a parallelogram is base × height, so the area is 1 × 4 = 4 square units. y2−y1 2b−b = = b x2− x1 2 a−a a S(7) = 2 + 4 + 6 = 12 2. (56) 99 8. (G) Set up the two equations and subtract them from one another to find the price per hour: y + 7x = 420 − y + 4 x = 270 3x = 150 4. (H) 12 p • 0 p −4 p = ( p12 • p0 ) 4 p 1 x = 50 = p 12+0 +4 = ( ) p 12+4 = p16 ( 5. (A) ) To find the fixed fee, use one of the equations ( y + 7x = 420 or y + 4 x = 270) and solve for y, using x = 50. First, find the radius when the depth of the water is 2 ft. Set up two similar right triangles as shown below: y + 4x = y + 4( 50) = y + 200 = y= 12 ft x ft 2 ft 270 270 270 70 9. (D) Point R is at (4, 3). If (x, y) is rotated 180° about the origin: R( x, y) → (− x, − y). Therefore, R( 4, 3) → (−4, −3). 4 ft 10. (F) Use a proportion to find x. Since the diameter of the right inverted cone is 12 ft, the radius is 6 ft: −8 15.3 × 10 1.5 × 10 4 x = 2 4 6 ( Now, find the volume of the cone with a radius of 3 ft and a height of 2 ft: Then use the rule of exponents to simplify. 10.2 × 10 = 10.2 × 10 Rewrite the answer so that it is standard scientific notation form. −11 1.02 × 10 V = 1 r 2πh = 1 (32 ) π( 2) = 3π(2) = 6π 3 3 147 ) (−8−4) x = 3 ft − − 8 8 = 15.3 × 10 4 = 10.2 × 10 4 1.5 10 10 −12 EXPLANATION OF CORRECT ANSWERS GRADE 9 MATHEMATICS continued... 13. (C) Use the Pythagorean Theorem: 11. (C) Substitute the approximation π = 3.14 into each expression and solve to find which expression results in a negative value: x 2 + y 2 = 20 2 4 − π = 0.86 Substitute x = 2 y into the equation and solve for y. 3π − 9 = 0.42 12 − 4π = −0.56 36 − 9π = 7.74 ( 2 y)2 + y 2 = 20 2 4 y 2 + y 2 = 400 So, the answer is 12 − 4π . 5 y 2 = 400 y 2 = 80 12. (F) Triangles NPQ and MPR are similar, so corresponding sides of the triangles are proportional. Set up a proportion to find MR. y= 80 MR = NQ MP NP MR = 10 x +5 5 5(MR) = 10(x + 5) 5(MR) = 10x + 50 MR = 2x + 10 Answer Key for Grade 9 Mathematics 148 1. 12 6. H 11. C 2. 56 7. A 12. F 3. B 8. G 13. C 4. H 9. D 5. A 10. F SAMPLE PROBLEMS FOR GRID-IN ITEMS GRADE 88 DIRECTIONS: Solve each problem. On the answer sheet, write your answer in the boxes at the top of the grid. Start on the left side of each grid. Print only one number or symbol in each box. DO NOT LEAVE A BOX BLANK IN THE MIDDLE OF AN ANSWER. Under each box, fill in the circle that matches the number or symbol you wrote above. DO NOT FILL IN A CIRCLE UNDER AN UNUSED BOX. 1. 2. On a practice test, there are 3 essay questions for every 7 multiple-choice questions. If there are a total of 420 questions on this test, how many of those are essay questions? 0.21  =  x 1.10 0.33 3. What is the solution to the equation above? 4. 19 − 21  +  1.9 − 2.1  −  x = 10 P In the equation above, what is the value of x? –6 –4 –2 R 0 2 4 6 Point Q is to be placed on the number line one-third of the way from Point P to Point R . What number will be at the midpoint of segment PQ ? 149 SAMPLE PROBLEMS FOR 5. GRID-IN ITEMS continued... How many ways can the letters in the word RAIN be arranged horizontally so that the vowels (A and I) are always immediately next to each other (either AI or IA)? 7. A box contains 11 marbles: 7 red and 4 green. Five of these marbles are removed at random. If the probability of drawing a green marble is now 0.5, how many red marbles were removed from the box? 8. If a = 20 and a = 8 , what is the value of b 2 3b + a ? 6. A –8 –6 –4 –2 B 0 2 C 4 6 8 On the number line above, D (not shown) is the midpoint of AB , and E  (not shown) is the midpoint of BC . What is the midpoint of DE ? 150 SAMPLE PROBLEMS FOR 9. GRID-IN ITEMS continued... Maria is now 16 years old. In 6 years, she will be twice as old as her brother will be at that time. How old is her brother now? 10. 151 4.5 × 0.22 = 0.1 EXPLANATION OF CORRECT ANSWERS GRID-IN ITEMS GRADE 88 DIRECTIONS: When filling out the answer grid, remember that each column should contain ONLY one symbol/one digit. When filling out the circles, only fill in one circle to MATCH the symbol or digit at the top of the column. 1. (126) There are 3 essay questions for every 7 2. (– 7.8) 19 − 21 + 1.9 − 2.1 − x = 10 multiple-choice questions, for a total of 10 −2 + −0.2 − x = 10 questions. The proportion of essay questions is 3 . Multiply the fraction of essay 10 questions by 420 to find the total number of essay questions: 420 × 3 = 1260 = 126 10 10 2 + 0.2 − x = 10 x = −7.8 Since the answer is a positive whole number, skip the first column and begin inputting the digits to the answer in the second column on the left-hand side. 152 Since the answer is a negative, begin filling out the grid with the negative sign. The answer contains a negative sign, a whole number, a decimal point, and a digit in the tenths place. Each part of the answer, including the decimal point, should be placed in a separate column, with no blank spaces between them. EXPLANATION OF CORRECT ANSWERS GRID-IN ITEMS continued... 4. (– 2) First, find the length of PR : 4 − (−5) = 9 units 3. (0.7) 0.21 = x 0.33 1.10 Point Q is located 1 of the way from P to R, 3 so calculate where that point would be: Multiply the numerators and denominators of all the fractions by 100 to eliminate the decimals: 9 × 1 = 3 units 3 21 = 100x 33 110 Simplify the fractions: 7 = 10x 11 11 7 = 10x 7 = 10 So, point Q is located at 4 − 3 = 1. Finally, calculate the midpoint of PQ : Midpoint PQ = 7(11) = 10x(11) x = 0.7 Since the answer is a positive decimal, skip the first column and place the zero in the second column on the left-hand side. 153 −5+1 = −2 2 Since the answer is a negative single-digit, fill in the negative sign and the second column contains the digit, 2. EXPLANATION OF CORRECT ANSWERS GRID-IN ITEMS continued... 5. (12) There are three positions for the letters AI in the four letter combinations: 6. (1.25) Calculate the midpoints of AB and BC to find the locations of D and E, respectively: AI _ _ , _ AI _ _ , and _ _ AI For each of those positions of A and I, there are two combinations of the letters R and N: AIRN, AINR, RAIN, NAIR, RNAI, NRAI. Thus, for the letters AI (in that order), there is a total of 6 combinations. The question indicates that IA is also possible, so there are also + combinations with the letters in the order IA. The total number of combinations is 6 + 6 = 12. Find the midpoint for AB (Point D): D= −8 +3 −5 = = −2.5 2 2 Find the midpoint for BC (Point E): E = 3 +7 = 5 2 Now, find the midpoint of DE : −2.5+5 = 2.5 = 5 = 1.25 2 4 2 Since the answer is a positive whole number, skip the first column and begin inputting your answer in the second column. 154 Since the answer is a positive decimal, skip the first column. The response begins inon the second column on the left-hand side. EXPLANATION OF CORRECT ANSWERS 7. (4) GRID-IN ITEMS continued... 8. (65.2) Use the given equation and value of a to find the value of b: There were 11 marbles in the box. After 5 marbles were removed, the total number of a = b marbles in the box is now 6. The probability of drawing a green marble is now 1 , which 2 is equivalent to 3 , thus, 3 green marbles 6 remain in the box. Originally, there were 7 red marbles in the box. Since there are now 6 total marbles, there are now 3 red marbles. Meaning 4 red marbles were removed from the box. Since the answer is a positive single-digit whole number, skip the first column and the response begins on the second left-hand column. 20 = and a 8,= so 8 b 20 = or b 8 . 20 Now, calculate the expression: ( ) 3b + a2 = 3 8 + 8 2 = 1.2 + 64 = 65.2 20 155 Since the answer is a positive whole number with a decimal, skip the first column and begin inputting the answer starting at the second column at the left-hand side. EXPLANATION OF CORRECT ANSWERS 9. (5) GRID-IN ITEMS continued... 10. (9.9) In order to solve this problem, first convert 4.5 to a whole number by multiplying the 0.1 numerator and denominator by 10 to get 45 1 which is 45. When Maria is 22, she will be twice as old as her brother. Let x = the age of Maria’s brother when Maria is 22. 2x = 22 x = 11 To find Maria’s brother’s current age, subtract 11 − 6 = 5. Maria’s brother is currently 5 years old. Since the answer is a positive single-digit whole number, skip the first column and the response begins in the second left-hand column. Multiply: 45 × 0.22 = 9.9 Since the answer is a positive whole number with a decimal, skip the first column and begin inputting the answer starting in the second column on the left-hand side. Answer Key for Grid-In Mathematics 156 1. 126 6. 1.25 2. –7.8 7. 4 3. 0.7 8. 65.2 4. –2 9. 5 5. 12 10. 9.9